Вы находитесь на странице: 1из 144

Table of Contents

1. General and Cellular Basis of Medical Physiology 6


2. Physiology of Nerve and Muscle Cells 13
3. Functions of the Nervous System 24
4. Endocrinology and Reproduction 39
5. Gastrointestinal Physiology 48

in
6. Circulating Body Fluids 51
7. Cardiovascular Physiology 56
8. Pulmonary Physiology 68
9. Renal and Acid-Base Physiology 82

s.
10. Critical Care Physiology 90

Self Scorers
Self-scorer 1: General Physiology 95
iim
Self-scorer 2: Physiology of Nerve and Muscle Cells
Self-scorer 3: Functions of the CNS
Self-scorer 4: Endocrinology and Reproduction
Self-scorer 5: Gastrointestinal Physiology
97
99
102
105
Self-scorer 6: Cardiovascular Physiology 107
Self-scorer 7: Pulmonary Physiology 110
Self-scorer 8: Renal and Acid-Base Physiology 112
4a
A Blast from the Past 114

More Questions for Self-study 129


m
Ai

5
1. GENERAL PHYSIOLOGY; D. highly variable
CELLULAR BASIS OF MEDICAL Osmosis; osmolality of body fluids;
PHYSIOLOGY physiologic solutions
7. Plasma Na = 135 mmol/L, plasma glucose =
Body fluid compartments 400 mg/dL and blood urea = 120 mg/dL. What
1. The volume of distribution of intravenously is the osmolality of plasma?
administered sucrose in a healthy 70-kg man is A. 290 mOsm/L
about B. 310 mOsm/L

in
A. 3.5 L C. 330 mOsm/L
B. 10.5 L D. 350 mOsm/L
C. 14 L
D. 28 L 8. Sodium and its accompanying anions
contribute to about ________ of plasma

s.
2. Which substance distributes exclusively in osmolality
intracellular fluid? A. 20 %
A. Heavy water B. 50 %
B. Sucrose C. 70 %
C. Evans Blue dye D. 90 %
D. None of the above

A. Extracellular fluid (ECF)


B. Intracellular fluid (ICF)
iim
3. Which compartment does the term sucrose
space refer to?
9. Which of the following contributes least to
plasma osmolality?
A. Sodium
B. Urea
C. Glucose
C. Interstitial fluid (ISF) D. Proteins
D. Plasma
10. Normal red blood cells were placed in the
4a
4. 100 mg of sucrose is injected into a 70 kg following solutions and observations made after
man. The plasma level of sucrose after mixing is 1 hour. Cells would have lysed when placed in:
0.01 mg/ml. If 5 mg has been metabolized A. 0.3% NaCl
during this period, then what is the ECF B. 0.9% NaCl
volume? C. 1.35% NaCl
A. 9.5 liters
B. 14 liters 11. Which of the following solutions is
C. 17.5 liters hypertonic?
m

D. 10 liters A. 0.9 % NaCl


B. 5% dextrose
5. In a healthy 70 kg adult, plasma volume C. 20% mannitol
(measured using the Evans Blue dye) was found D. Distilled water
to be 3000 ml. Hematocrit was 40%. His blood
Ai

volume would be about: 12. 2 liters of 0.9% NaCl is administered to a


A. 5000 ml 12-year old boy with moderate isotonic
B. 5200 ml dehydration. What will be the change in ICF
C. 5400 ml volume after NaCl administration?
D. 5600 ml A. No change
B. Increases by 2 liters
6. The water content of lean body mass is: C. Increases by 0.5 liter
A. 50 ml/100 g D. Decreases by 0.5 liter
B. 70 ml/100 g
C. 30 ml/100 g 13. ICF volume decreases when dehydration is:
A. isotonic A. Gap junctions
B. hypertonic B. Focal adhesions
C. hypotonic C. Zonula occludens
D. Desmosomes
14. In a patient who has become dehydrated,
body water should be replaced by intravenous 20. Connexins do not allow the passage of:
infusion of [*] A. proteins
A. distilled water B. Na
B. 0.9% NaCl C. Ca

in
C. 5% dextrose solution D. IP3
D. hyperoncotic albumin
E. 10% glucose solution Transport across cell membranes
21. Of the following substances, the lipid bilayer
[*] This question is reproduced from WF Ganong. is most permeable to:

s.
Review of Medical Physiology, Mc Graw Hill, 2005. I A. sodium
would strongly recommend that you spend time B. urea
reviewing multiple-choice questions in the Self-Study C. glucose
Section in this book.
D. water
15. Which of the following exerts the greatest
22. The rate of diffusion is inversely
A. Dextran
B. Hydroxyethyl starch
C. Albumin
D. Fibrinogen
iim
osmotic effect on a mole-mole basis?
proportional to:
A. surface area available for diffusion
B. thickness of the membrane
C. concentration gradient for the substance
D. diffusion coefficient
A primer in cell biology
23. Which one of the following is an example of
16. Which of the following is matched
passive transport?
incorrectly?
4a
A. Calcium efflux by calcium pump
A. Microfilaments: actin, myosin
B. Na-Ca exchanger
B. Intermediate filaments: vimentin, keratin
C. Potassium efflux through potassium leak
C. Microtubules: clathrin
channels
D. Cytoskeleton: spectrin, ankyrin
D. Calcium sequestration in sarcoplasmic
reticulum
17. Mutations in which of the following genes is
known to result in defective ciliary motility?
24. Most of the ATP generated in nerve cells is
m

A. Dynein
utilized to energize the:
B. Flagellin
A. Na-Ca exchanger
C. Kinesin
B. H-ATPase in the cell membrane
D. Supercilin
C. Na-K ATPase
D. synthesis of proteins
18. Cytosolic calcium concentration in unexcited
Ai

cells is about:
25. Which of the following is incorrectly
A. 2.5 mM
matched?
B. 10 mM
A. Na-K ATPase: antiport
C. 150 M
B. H-ATPase: uniport
D. 100 nM
C. SGLT: symport
D. Ca-ATPase: biport
19. Which intercellular junctions allow the
passage of small molecules and ions from one
26. Which of the following statements regarding
cell to another?
regulation of cell cycle is correct?

7
A. They serve to regulate transition of the cell Amount metabolized = 5 mg
from one phase of the cell cycle to the next Amount remaining in ECF = 95 mg
phase. Concentration after mixing = 0.01 mg/ml
B. At restriction points, cyclins always promote Volume of distribution of sucrose
transition from G0 to S phase. = 95 mg / 0.01 mg/ml = 9500 ml = 9.5 L
C. When there are defects in DNA synthesis,
normally, progression occurs from G2 to M 5. Blood volume
phase. = plasma volume [100 / (100Hct)]
D. Loss-of-function mutations in p53 gene do

in
not affect progression at restriction points. 6. Body mass = fat mass + lean body mass.
The water content of lean body mass (fat-free
Answers: General Physiology & Cellular mass) is relatively constant and is about 70
Basis of Medical Physiology ml/100 g. For example, in an individual
1C 2D 3A 4A 5A weighing 70 kg and whose total body water is

s.
6B 7B 8D 9D 10A measured to be 42 L (= 42 kg), lean body mass =
11C 12A 13B 14C 15C 42 / 0.7 = 60 kg, and fat mass is 10 kg. Fat is
16C 17A 18D 19A 20A relatively anhydrous. Body fat % = 10/70 100
21D 22B 23C 24C 25D = 14%. Body fat percentage is greater in women
26A compared to men.

Answer Explanations: iim


1. Intravenously administered sucrose distributes
throughout ECF (plasma + interstitial fluid).
ECF volume in a 70-kg healthy adult is about 14
L (20% of body weight). The volume of
7. Plasma osmolality (mosm/L) =
2 [Na+] + [glucose] / 20 + [BUN] 18/50
(mmol/L) (mg/dL)

BUN: blood urea nitrogen


(mg/dL)

interstitial fluid is about 10.5 L (75% of ECF Plasma osmolality is chiefly due to Na and its
volume) and plasma volume is about 3.5L (25% accompanying anions Cl and HCO3. Normally,
of ECF volume). Heavy water distributes serum osmolality ranges from 280295
4a
throughout body water. Sucrose, inulin and mOsm/Kg H2O.
mannitol distribute exclusively in the ECF.
Evans blue dye stays in the plasma. Blood urea and urea nitrogen:
The formula of urea is NH2CONH2
Typical values in a healthy adult male weighing Molar mass of urea is 60 g; each molecule of
70 kg are as given below: urea has 2 nitrogen atoms.
Compartment Volume Marker The mass of nitrogen in urea is 2 14 = 28 g
m

Total body water 42 L D 2O Thus urea concentration of 60 mg/dL


ICF 28 L - corresponds to a [BUN] of 28 mg/dL.
ECF 14 L Sucrose
Interstitial fluid 10.5 L - 8-10. What is osmosis?
Plasma 3.5 L Evans blue Osmosis is the movement of water across a
semipermeable membrane permeable to water
Ai

4. Indicator-dilution principle: but not to solutes, from a solution with lesser


Volume of distribution of the indicator equals concentration of osmoles to a solution with a
the amount injected (A) divided by the greater concentration of osmoles. This continues
concentration (C) in plasma (of the indicator) until osmotic equilibrium (i.e., the osmolality of
after mixing. either compartment is equal) is attained.

In the above example,


First, sucrose distributes throughout ECF.
Amount of sucrose injected = 100 mg

8
You can view an animation of osmosis at Osmolality is a colligative property that depends
http://tiny.cc/4auab upon the number of solute particles, and not the
size of the particles. To illustrate, the
Full URL http://highered.mcgraw- contribution of 1 Na ion and 1 albumin molecule
hill.com/sites/0072495855/student_view0/chapte toward osmolality of plasma is the same. Since
r2/animation__how_osmosis_works.html the molar concentration of proteins in plasma is
[accessed 10 May 2010] very low (60 g/L) compared to that of Na (140
mM) and Cl (100 mM), one can understand why
What is an osmole? plasma proteins contribute very little to the

in
An osmole (effective osmole) is an osmotically osmolality of plasma compared to Na and its
active particle potentially capable of causing accompanying ions.
osmosis. Examples include sodium ion, chloride
ion, protein anions, and phosphate ion. A solute Colloid osmotic pressure of plasma: While
to which the cell membrane is not as freely proteins are present in plasma at a concentration

s.
permeable as it is to water will function as an of 60-80 g/L, they are not normally present in
effective osmole. For example, in comparison to significant concentrations in the interstitium.
water, the cell membrane is relatively Thus, the osmotic pressure of plasma proteins
impermeable to sodium and chloride ions, (called colloid osmotic pressure or oncotic
mannitol. In this context, it is worth noting that pressure) is much greater than the osmotic
the amount of ions crossing the cell membrane pressure of proteins in the interstitium. This

fluxes.

What is an ineffective osmole?


iim
through channels and transporters in is much
smaller relative to osmotically driven water
oncotic pressure gradient across the capillary
restrains fluid filtration and favors reabsorption
of fluid into the capillary.

10. Hemolysis begins when normocytes are


If the cell membrane is permeable to a solute placed in 0.5% NaCl and is complete in 0.3%
(for example, urea), the substance will move NaCl. In contrast, when RBCs are placed in
across the membrane until its concentration is hypertonic saline, they lose water and diminish
4a
exactly the same on both sides of the membrane. in size.
In such an instance, osmosis (net movement of
water) does not occur. However, note that urea 11-12. Why is 0.9% NaCl called isotonic?
does indeed function as an effective osmole in 0.9% NaCl has the same osmolality as normal
the renal medullary interstitium. human plasma (about 290 mOsm/kg H2O,
please see the calculation below); further, when
Osmotic pressure is the pressure that would be it is infused into a normal human (i.e., one with
required to stop water flux (osmosis) across a an ECF osmolality of 290 mOsm/kg H2O), it
m

semi permeable membrane. does not cause any change in the steady state
volume of red blood cells or other cells; i.e.
Osmotic pressure P = CRT (Vant Hoff because it does not change the steady state
equation), where, osmolality of normal human plasma, it is an
C = concentration of osmoles isotonic solution.
Ai

R = a constant
T = temperature in Kelvin Calculate the osmolality of 0.85% NaCl.
0.85% NaCl contains 0.85 g of NaCl per
In the steady state, the osmolality of all body deciliter of the solution.
fluids is identical; that is, osmotic pressure = 8.5 g/ L of the solution
across the plasma membranes of cells in the Molar mass of NaCl = 58.5 g
steady state is zero and there would be no net 1 mol of NaCl = 58.5 g
water flux (osmosis) across the cell membrane. 8.5 g = 8.5/58.5 mol = 0.145 mol = 145 mmol/L
Each Na is accompanied by 1 chloride ion

9
Therefore, total concentration of osmoles
(osmotically active particles) = 2 145 = 290 In well hydrated individuals, the hypo-
mOsm/L osmolality that occurs when hypotonic dextrose
solutions are infused is sensed by osmoreceptors
13, 14. ECF volume changes and secretion of antidiuretic hormone is
When the terms dehydration and overhydration promptly inhibited to excess excrete water.
are used without further qualification, they are
typically used to refer to ECF volume To summarize the meaning of the term tonicity
contraction and ECF volume expansion of a solution it refers to the effect of an

in
respectively. However, in some instances (as in administered solution on the steady state
Question 14) the term dehydration may be used osmolality of normal human plasma.
to refer to a decrease in total body water as well.
15. Starlings law of filtration
Classification of dehydration*: Starlings equation (Starlings hypothesis,
Type Cause

s.
1896) describes forces that affect fluid flux
Isosmotic Blood loss
Hyperosmotic Vasopressin deficiency
across capillaries.
Hypoosmotic Adrenocortical insufficiency Fluid movement = Kf [(Pc + i) (Pi + c)],
*Overhydration is classified likewise. where,
Kf = capillary filtration coefficient

Isosmotic
Hyperosmotic
Hypoosmotic
volume
Decreases
Decreases
Decreases
iim
Changes in ICF volume in various types of dehydration:
Type ECF ICF volume

No change
Decreases
Increases
Pc = hydrostatic pressure in the capillaries
Pi = hydrostatic pressure in the interstitium
c = capillary colloid osmotic pressure
i = colloid osmotic pressure in the interstitium

Normally, Pc is the principal force favoring


14. The key phrase here is body water which filtration. The osmotic pressure of plasma is
includes ICF as well as ECF. In the steady state, normally about 25 mm Hg higher relative to the
hypertonic dehydration is associated with a osmotic pressure of interstitium because
4a
reduction in ECF as well as ICF volume. So plasma proteins (colloids) are limited to
therapy must be aimed at replenishing total body plasma; proteins in plasma restrain fluid
water, not merely ECF volume. filtration into the interstitium.

Is 5% dextrose isotonic or hypotonic? Albumin is quantitatively the most important


Molar mass of dextrose (D-glucose, C6H12O6) is contributor to the colloid osmotic pressure of
180 g plasma. 1 g of albumin in 100 ml of plasma
m

5% solution contains 50 grams of dextrose per exerts an osmotic effect of 6 mm Hg; the same
liter of the solution concentration of globulins will exert a pressure
50g = 50/180 mol = 0.277 mol = 277 mmol = of only about 1.5 mm Hg.
277 mOsm/L ~ 270280 mOsm/kg H20
16. Microtubules have a 9 + 2 fibrillar structure.
As 5% dextrose has approximately the same They are made up of tubulin, dynein, and
Ai

osmolality as normal human plasma, it is an kinesin. The mitotic spindle is made up of


isosmotic solution. However, when dextrose is microtubules. Colchicine arrests cells in
infused, it is metabolized and the net effect (over metaphase by inhibiting polymerization of
a period of time, especially when large volumes microtubules.
are infused) is that of adding water to plasma.
This excess water can enter cells. Thus, 5% 17. Dynein has ATPase activity. Defects in
dextrose is of value in replenishing ICF volume dynein are implicated in the immotile cilia
in intracellular dehydration. Also, note 5% syndrome (Kartageners syndrome)
dextrose is hypotonic because it dilutes plasma
in the steady state.

10
18. ECF [Ca2+] = 2.5 mM 24. A large fraction of ATP (nearly 70%)
Cytosolic [Ca2+] in unexcited cells = 100 nM synthesized in neurons is used to energize the
ECF [Ca2+] / cytosolic [Ca2+] = 25000 Na-K ATPase and maintain ion gradients across
A transient increase in intracellular calcium is the nerve cell membrane.
used as a signal to activate or inactivate
intracellular processes such as exocytosis, 25. Ca-ATPase is a uniport because it transports
muscle contraction. [Ca2+]i is quickly one species. It is a primary active transport
normalized by the sarcoplasmic-endoplasmic process as it uses ATP as the source of energy.
reticulum calcium pump (SERCA pump) and a

in
Na-Ca antiporter (also called the Na-Ca 26. Please see this animation on Cell Cycle at
exchanger, NCX) in the cell membrane. http://www.cellsalive.com/cell_cycle.htm
Sustained increases in intracellular calcium [accessed 10 May 2010]; another animation on
trigger apoptosis, culminating in cell death. the cell cycle can be viewed at
http://tiny.cc/i8ag0

s.
19. Gap junctions (electrical synapses; The full URL is http://highered.mcgraw-
connexons) are made up of a protein called hill.com/sites/0072495855/student_view0/chapte
connexin; these are large diameter channels that r2/animation__control_of_the_cell_cycle.html
allow the passage of ions and other small [accessed 10 May 2010]
molecules such as amino acids and glucose. ******************************
iim
Thus, they electrically couple adjacent cells.
They are found in visceral smooth muscle and
heart and between some neurons. The ventricle
of the heart contracts as one unit because all
muscle fibers in the ventricle are electrically
coupled by gap junctions. However, connexins
Supplement:
Classification of mechanism of transport
across cell membranes:

For animations on transport across cell


do not allow large molecules such as proteins to membrane visit http://tiny.cc/sgr8e and
pass between cells. http://tiny.cc/vt80i
4a
See http://kosmotorcars.com/portafolio/ and The full URLs, respectively, are
click on NEJM Cx 43 (accessed 10 May 2010) http://www.wiley.com/legacy/college/boyer/047
0003790/animations/membrane_transport/memb
22. Ficks law of diffusion: rane_transport.htm accessed 10 May 2010 and
Diffusion rate (J) = DA dc/dx http://www.northland.cc.mn.us/biology/BIOLO
D: Diffusion coefficient GY1111/animations/passive1.swf accessed 10
A: Area available for diffusion May 2010 (browser needs Shockwave player)
m

dc: concentration gradient


Mechanism Examples
dx: thickness of membrane
Simple diffusion Diffusion of oxygen,
carbondioxide, anesthetic gases,
The diffusion coefficient is affected by factors nitric oxide through lipid layer
such as temperature, and the permeability of the Facilitated diffusion Glucose entry into cells through
membrane to the molecule/ion in question. GLUT 1-5
Ai

Permeability of a membrane to an ion/molecule Primary active Na-K ATPase,


transport H-K ATPase,
is in turn affected by the number of ion channels Ca-ATPase
or transporter molecules available to transport Secondary active Na-glucose cotransporter
the species in question. transport (SGLT), Na-amino acid
cotransport, Na-H exchanger,
Na-Ca exchanger
23. Ion flux through ion channels is a passive
process; i.e. it occurs down a concentration
gradient and requires no input of free energy. Simple diffusion, by definition, is diffusion
occurring through the lipid bilayer.

11
Facilitated diffusion occurs through protein battery) to drive the uphill transport of glucose
molecules in the cell membrane (ion channels or from ECF to ICF. Note that the species that is
transport proteins). actively transported by SGLT is glucose. The
Na-amino acid symporter is similar. The term
See http://tiny.cc/kmi8z secondary refers to the fact that the energy
Full URL http://groups.google.com/group/medical-physiology- source (the sodium ion gradient) which drives
at-som-aimst/browse_thread/thread/b60732d8baf36bee# this process depends upon normal operation of a
primary active transport process the Na-K
Some authors consider ion flux through ion pump that generates a Na ion gradient. If the Na-

in
channels also as an instance of simple K pump fails due to lack of ATP or any other
diffusion this distinction is a matter of taste. reason, then, the Na gradient will be gradually
reduced and all secondary active transport
By definition, a primary active transport processes powered by the Na gradient will also
process is driven by hydrolysis of ATP. be affected.

s.
Notes about the Na-K ATPase:
Nomenclature of transporters based on the
It is a primary active transport process
direction of movement and the number of
The pump is present in the cell membrane
species of transported. Note: this is not a
It is ubiquitous i.e., present in all cells
classification of mechanism of transport.
It pumps 3 Na out of the cell and 2 K ions

the genesis of RMP.


For an animation on the pump, visit
iim
in. Thus it makes the inside of the cell
negative with respect to exterior; i.e. it
contributes to a small extent (about 4 mV) to
Name
Uniport

Symport
(also cotransport)
Description
1 species
transported
2 species
transported in
the same
Example
Ca-ATPase,
GLUT*
Na-glucose
cotransport,
Na-K-2Cl
http://tiny.cc/4iwh0 Full URL is
http://www.brookscole.com/chemistry_d/templates/student_r direction cotransport
esources/shared_resources/animations/ion_pump/ionpump.ht Antiport (also 2 species Na-K ATPase,
ml [accessed 10 May 2010] countertransport) transported in Na-H exchanger,
4a
opposite Cl-HCO3 exchanger.
Another animation is at http://tiny.cc/vxyi0 directions
the full URL is http://highered.mcgraw- *GLUT glucose transporter
hill.com/sites/0072495855/student_view0/chapter2/animation
__how_the_sodium_potassium_pump_works.html
[accessed 10 May 2010] Carrier mediated transport this term refers
It plays an important role in maintaining cell to transport processes in which the transport
volume. If the pump is inhibited as can species physically attaches to a carrier molecule,
happen when ATP is limiting or and is carried by it. Carrier mediated transport
m

pharmacologically (with digoxin) processes can be active or passive. Na-K


intracellular Na increases also increasing the ATPase (an active transport process) and
size of cells. glucose transport via GLUT (a passive transport
Digoxin, a cardiac glycoside inhibits this process) are both examples of carrier-mediated
pump. Inhibition of the pump leads to an transport.
increase in cytosolic Ca and this augments
Ai

the force of contraction of cardiac muscle Other modes of transport: Exocytosis,


cells. endocytosis, transcytosis (vesicular transport).
About 70% of the ATP generated in nerve Exocytosis is triggered by a rise in intracellular
cells is used to energize the Na-K ATPase. calcium. Proteins injected into the circulation
often have been endocytosed into vesicles by
A secondary active transport process utilizes endothelial cells to be exocytosed as vesicles
an ion gradient as a source of energy rather than into the interstitium. This process called
ATP. The Na-Glucose cotransporter (SGLT) transcytosis or vesicular transport requires an
utilizes the energy of the Na gradient (Na input of free energy.

12
2. PHYSIOLOGY OF NERVE Cardiac muscle - 90 mV
AND MUSCLE CELLS SA node - 70 mV
Red blood cells - 10 mV
Resting membrane potential (RMP) refers to
the transmembrane potential (i.e., the potential Driving forces for ion fluxes across membranes
difference across cell membranes) in the steady are the electrical and chemical (concentration)
state (unexcited state). RMP is present across all gradients.
cells. You may find this animation on the
genesis of RMP helpful: Equilibrium potential: the membrane potential

in
http://bcs.whfreeman.com/thelifewire/content/chp44/4401s.swf at which net transmembrane flux of a particular
[Shockwave Flash Object; accessed 10 May ion is zero because the electrical gradient
2010] counterbalances the chemical gradient.

Origin of RMP: Example, Na flux across the membrane would

s.
1. The cell membrane is totally impermeable stop when the membrane potential reaches + 60
to organic anions and proteins present in mV.
ICF.
2. In neurons for example, in the resting state, Equilibrium potential (given by Nernst equation)
the cell membrane is quite permeable to K, of Na:

iim
(about 50 times more permeable to K than
Na). K exits the cell down its concentration
gradient, making the inside of the cell
negative with respect to exterior.
3. Charge separation occurs across the
membrane (a thin capacitor) leaving the
ENa = - 61 log [Na]i / [Na]o
= - 61 log [10/140] = + 61 mV

Similarly, for K:
Ek = - 61 log [K]i / [K]o
inside of the membrane negative with Ki = 140 mM, Ko = 5 mM, EK = - 92 mV
respect to outside.
4. Additionally, the Na-K ATPase mechanism Equilibrium potentials for various ions across nerve cell
4a
membranes:
contributes a small extent to making the ICF (mM) ECF (mM) Equilibrium
inside of the cells negative. potential (mV)
Na 10 140 + 60
If you get a microelectrode into a single nerve K 140 5 - 90
fiber with a reference electrode in ECF and Cl 10 120 - 70
connect them to a galvanometer, you will be Ca 100 nM 2.5 + 130
able to record this potential difference.
Excitability:
m

Excitability is the response to a threshold


stimulus with a propagated action potential.
ICF - + ECF Nerve and muscle are excitable. Other tissues
[Na] = 10 mM - + [Na] = 140 mM are not.
[K] = 140 mM - + [K] = 5 mM
Ai

[Cl] = 10 mM - + [Cl] = 100 mM The physiologic basis of excitability: high


Proteins potassium permeability at rest (and therefore an
RMP in the range of 50 to 90 mV) and
presence of voltage gated Na channels are
Magnitude of RMP in different tissues essential.
Cell / tissue Magnitude of RMP
(inside negative) What is the effect of hyperkalemia on RMP of
Nerve cells - 70 mV cardiac muscle cells?
Skeletal muscle - 90 mV Normally, when plasma [K+] = 5 mM, RMP of
GI smooth muscle Variable; - 40 to - 60 mV cardiac muscle cells = - 90 mV.

13
When plasma [K+] = 10 mM,
RMP = - 61 log [140/10] = - 70 mV If you stimulate the nerve with the cathode (and
That is, hyperkalemia makes RMP less negative. use a subthreshold stimulus), it produces small
SA node has an RMP of -70 mV. In this depolarizations (called catelectrotonic
situation, ventricular cells would be expected to potentials). These small depolarizations are local
compete with SA node to pace the heart. responses; i.e. they are not propagated through
This clearly increases the risk of cardiac the nerve fiber. The stimulus that is just
arrhythmias. adequate to result in a propagated action
potential is called threshold stimulus.

in
Functional organization of neurons:
The membrane potential at which voltage gated
Each neuron has the following zones: Na-channels open all at once to result in a full
fledged action potential is called firing level or
Receptive zone: afferent input converges on the threshold level and it is about -55 mV in nerve

s.
dendrites and soma. fibers. Within a millisecond of opening, Na
channels get deactivated; i.e. close and cannot be
Integrator: all afferent input is integrated by the opened until the membrane potential comes back
soma (cell body) and if the axon hillock is down to firing level. On the other hand, voltage
depolarized to threshold, a full-fledged action gated K channels open allowing rapid efflux of
potential occurs. K. This repolarizes the membrane; i.e., brings it
iim
The axon transmits the nerve impulse (action
potential) down to the nerve terminal where
neurotransmitter is released for synaptic
transmission. The action potential also sweeps
back to RMP.

Differences between local & action potentials:


Local potentials (also called electrotonic
potentials) may be depolarizing (also
back through the cell body wiping it clean of all catelectrotonic potentials) or hyperpolarizing
input and rendering it refractory to all input for a (also called anelectrotonic potentials).
short time. Excitatory postsynaptic potentials and motor end
4a
plate potentials are examples of catelectrotonic
A narrated animation of the action potential can responses. Inhibitory postsynaptic potentials are
be accessed at http://tiny.cc/8qmxi; the full URL anelectrotonic responses. Electrotonic potentials
is http://highered.mcgraw- are graded responses that occur with
hill.com/sites/0072495855/student_view0/chapter14/animation__t subthreshold stimuli (i.e. proportional to
he_nerve_impulse.html [accessed 10 May 2010] stimulus intensity) magnitude is typically a
few mV. They undergo spatial and temporal
Nerve conduction speed is increased several summation. They are not propagated.
m

fold by myelination of axons.


In contrast, action potentials are propagated
Label the various phases of the nerve action responses that occur with threshold stimuli.
potential They are all-or-none; i.e., they occur with a
constant size.
Ai

Excitability during various phases of the action


potential:
Absolute refractoriness: No matter how strong
the stimulus, a nerve is absolutely refractory to
stimulation during the action potential until
repolarization brings the membrane back to
firing level. The basis for absolute refractoriness
is voltage and time inactivation of voltage gated
Na channels

14
then the intensity of the stimulus has no bearing
Relative refractory period: In this period, only on the size of the action potential.
a suprathreshold stimulus would trigger an
action potential. The reason why a stronger Physiologic basis of the all-or-none law:
stimulus is required for excitation is because the Action potential occurs only when the firing
stimulus has to overwhelm the repolarizing level is reached because the voltage-gated
current. sodium channels that allow a massive influx of
Na open only at the firing level.
Supernormal phase: a weaker stimulus would

in
trigger an action potential. Effect of stimulus intensity on electrical
response of nerve and muscle (all-or-
Subnormal phase: during nothing):
afterhyperpolarization, a stronger stimulus Stimulus Intensity Electrical response in
would be required to bring the membrane to nerve / muscle

s.
threshold. Subthreshold No action potential
Threshold Action potential
Stimulus: a transient change in one of the
Suprathreshold Action potential of the
parameters of the environment.
same strength
Stimulus types: electrical, chemical,
mechanical iim
In the figure below, how do stimuli A, B and
C differ from each other?
X-axis: time in milliseconds;
Y-axis: current intensity
4a
A B C

Stimulus parameters:
1. Intensity: subthreshold, threshold,
suprathreshold
2. Duration: typically (in milliseconds) The figure above depicts action potential in a
m

3. Frequency: 1-100 Hz muscle fibre (top panel) and mechanical


4. Rise time: time in which stimulus intensity response (bottom panel) on the same time scale
rises to its maximum value. in skeletal muscle. Note that whereas the
duration of the action potential is only about 5
In the figure above, the stimuli A, B and C have ms, the duration of the mechanical response is
Ai

identical amplitudes but have different rise much longer. Thus, it is possible to summate the
times. mechanical responses to successive stimulation
of muscle. In cardiac muscle, this is not possible
Threshold stimulus: The minimum stimulus (please see the section on Muscle Twitch, next
intensity that will elicit an action potential in an page).
excitable tissue under a given set of conditions.
For Animations on Muscle Physiology: Go to
All or none law: If a stimulus is of sufficient http://tiny.cc/euwwd the full URL is
intensity as to bring the membrane to threshold,

15
http://groups.google.com/group/medical-physiology-at-som- The figure above depicts the effect of an
and take the
aimst/browse_thread/thread/bf1cf8f67ee28a56# increase in the frequency of contraction of
Weblinks from there [accessed 10 May 2010] skeletal muscle on force. With increasing
frequency of stimulation at a given intensity, the
Muscle twitch: A muscle twitch is contraction force of successive contractions increase
followed by relaxation occurring in response to a because of the beneficial effect of previous
single stimulus. contractions; precisely, more calcium is
Twitch duration = contraction time + relaxation available for later contractions than earlier ones.
time. Twitch duration is quite variable and This is called the staircase phenomenon,

in
skeletal muscle is classified as fast or slow, Treppe or Bowditch effect. With a decrease in
depending upon twitch duration. The duration of frequency of stimulation, a gradual decrease in
muscle twitch in fast muscle could be as short as force of contraction is also observed.
10 ms. In contrast, in slow skeletal muscle,
twitch duration is typically greater than 100 ms. Axoplasmic transport: This refers to transport

s.
of molecules in the cytoplasm of the axon. This
In cardiac muscle, when average heart rate is 75 is not to be confused with conduction of the
beats per minute, average twitch duration is 800 nerve impulse, which is much faster.
ms; i.e., cardiac muscle is slower than skeletal
muscle. The twitch duration in the smooth Fast anterograde transport (i.e., from the cell
muscle of sphincters is longer. Smooth muscle is bodies to axon terminals) is brought about by
thus the slowest of three muscle types.iim
Tetanizing stimulus frequency: In skeletal
muscle, with high frequency stimulation, it is
possible to summate the effects of multiple
microtubules (kinesin and several other proteins
are involved).

Axoplasmic transport
Fast anterograde transport
Speed (mm/day)
400
stimuli and increase the force of muscle Slow anterograde transport 0.510
contraction. Tetanus is the most forceful and Retrograde transport 200
sustained muscle contraction (i.e. with no
4a
relaxation in between). To tetanize skeletal
Most of the Questions in this Section have
muscle, it must be stimulated at a minimum
only one correct answer, as would be
frequency called the tetanizing frequency that
suggested by the stem of the question;
depends on the duration of contraction. The
Some questions may have more than one
minimum stimulus frequency required for
correct answer.
tetanizing the muscle is the reciprocal of the
contraction period expressed in seconds.
1. Fast anterograde transport occurs at a rate of
m

about:
For example, if the contraction period is100 ms
A. 40 mm/day
(0.1 s), tetanizing frequency is 1/0.1 = 10 Hz.
B. 400 mm/day
C. 1 mm/day
D. 10 mm/day
Ai

2. The resting cardiac muscle cell is most


permeable to:
A. Na
B. K
C. Ca
D. Cl

3. The membrane potential of cardiac muscle


cells is most affected by even a small change in
plasma concentration of:

16
A. Na C. 200 ms
B. K D. 250 ms
C. Cl
D. Ca 11. When heart rate is about 75 beats per minute,
what is the mean duration of action potential in
4. Hypokalemia would be expected to result in: ventricular muscle cells?
A. increased neuronal excitability A. 5 ms
B. a more negative RMP B. 25 ms
C. no change in RMP C. 200 ms

in
D. a decrease in firing level of neurons D. 250 ms

5. The number of Na channels per square 12. The force of muscle contraction cannot be
micrometer of membrane in myelinated increased by:
mammalian neurons is maximum in the: A. increasing the frequency of activation of

s.
A. Cell body motor units
B. Dendritic zone B. increasing the number of motor units
C. Initial segment activated
D. Node of Ranvier C. increasing the amplitude of action potentials
in motor neurons
6. The site of origin in the nerve of conducted D. recruiting larger motor units
impulses is the:
A. dendritic zone
B. axon hillock
C. node of Ranvier
D. terminal buttons
iim 13. An excitable cell has an RMP of -70 mV and
a firing level of - 50 mV. This cell would be
inexcitable when its membrane potential is:
A. - 30 mV
B. - 55 mV
7. In motor neurons, the portion of the cell with C. - 70 mV
the lowest threshold for the production of a full- D. - 90 mV
4a
fledged action potential is:
A. initial segment 14. For the same conditions (as in the above
B. soma question), the cell would be most excitable when
C. dendritic zone its membrane potential is:
D. node of Ranvier A. - 30 mV
B. - 55 mV
8. Action potential conduction velocity is C. - 70 mV
influenced by: D. - 90 mV
m

A. axon diameter
B. temperature 15. Although the equilibrium potential of Na is
C. myelination + 60 mV, the membrane potential does not reach
this value during the overshoot because:
9. In the CNS, the membranes that wrap around A. the concentration gradient of Na is reversed
Ai

myelinated neurons are those of: B. Na channels undergo rapid inactivation


A. Schwann cells C. K efflux commences immediately after the
B. oligodendroglia upstroke
C. endothelial cells D. the membrane is impermeable to Na
D. astrocytes
16. Which one of the following statements about
10. The duration of action potential in a skeletal electrotonic potentials is incorrect?
muscle fiber is typically: A. They are graded responses
A. 5 ms B. They are local (non-propagated) responses
B. 25 ms

17
C. They may be depolarizing or
hyperpolarizing 24. Ryanodine receptor is located in the:
D. They are produced by a threshold stimulus. A. sarcolemma
B. T-tubule
17. Thin filaments do not contain: C. terminal cisterns of sarcoplasmic reticulum
A. actin D. cytosol
B. myosin
C. troponin 25. Which of the following blocks the ryanodine
D. tropomyosin receptor?

in
A. Dantrolene
18. The ATPase activity of which of the B. Curare
following proteins is altered to regulate skeletal C. Cocaine
muscle contraction? D. Hexamethonium
A. Actin

s.
B. Myosin 26. Which of the following slow the relaxation
C. Troponin process in skeletal muscle?
D. Tropomyosin A. Slow myosin ATPase
B. Inhibition of Ca-Mg ATPase
19. The activity of which contractile protein is C. ATP depletion
altered to regulate smooth muscle contraction?
A. Actin
B. Myosin
C. Calmodulin
D. Tropomyosin
iim 27. Rigor mortis is due to:
A. damage to actin & myosin
B. rapid sequestration of Ca in ER
C. increased myosin ATPase
D. ATP depletion
20. Smooth muscle lacks:
A. actin 28. Staircase phenomenon (Treppe) is due to:
B. myosin A. increased availability of calcium
4a
C. troponin B. summation
D. tropomyosin C. tetanus
D. increased excitability
21. An example for nonsyncytial smooth muscle
is: 29. Which of the following statements regarding
A. iris type I muscle fibers is incorrect?
B. sphincter of Oddi A. They are rich in myoglobin
C. vas deferens B. Their oxidative capacity is high
m

D. uterus C. Their myosin ATPase activity is high


D. Their glycolytic capacity is moderate
22. The twitch duration in fast type skeletal
muscle fibers is about: 30. Which of the following is incorrect about
A. 10 ms type II muscle fibers?
Ai

B. 50 ms A. They are called slow fibers


C. 100 ms B. Their myosin ATPase activity is high
D. 250 ms C. They contain little myoglobin
D. They are rich in glycolytic enzymes
23. The major source of calcium for contraction
of skeletal muscle is: 31. Which of the following statements is
A. ECF incorrect?
B. cytosol A. Contraction against a constant load with
C. mitochondria approximation of the ends of the muscle is
D. sarcoplasmic reticulum (SR) called isotonic contraction.

18
B. Contraction can occur without an 38. Which of the following muscle types is the
appreciable decrease in the length of the fastest?
muscle. A. Skeletal muscle
C. Isometric contractions do work whereas B. Smooth muscle
isotonic contractions do not. C. Cardiac muscle
D. Muscles can lengthen while doing work.
39. Which of the following characteristics is
32. The smallest amount of muscle that can exhibited only by skeletal muscle?
contract in response to excitation of a single A. Gradation of force production

in
motor neuron is: B. Refractoriness
A. 1 muscle fiber C. Beneficial effect
B. a muscle fasciculus D. Staircase phenomenon
C. the entire muscle E. None of the above
D. all muscle fibers supplied by that neuron

s.
40. Non-linearity of length-tension relationship
33. The size of the motor unit is smallest in: is most evident in:
A. type II muscle fibers A. skeletal muscle
B. single-unit smooth muscle B. smooth muscle
C. orbicularis oculi C. cardiac muscle
D. soleus
iim
34. Gradation of force in skeletal muscle is not
achieved by:
A. increasing intracellular calcium
B. recruitment of motor units
41. In adults, intravesical pressure rises suddenly
when intravesical volume reaches about:
A. 100 ml
B. 200 ml
C. 300 ml
C. size principle D. 400 ml
D. asynchronous firing of motor units
E. varying release of calcium from SR 42. The latch-bridge mechanism in smooth
4a
muscle is responsible for:
35. Regarding the ionic basis of action potential A. fast muscle twitch
in cardiac muscle cells, which one of the B. sustained muscle contraction
following is incorrect? C. excitation-contraction coupling
A. Phase 0: Na influx D. unstable membrane potential
B. Phase 1: K influx
C. Phase 2: Ca influx 43. The minimum stimulus strength that
D. Phase 3: K efflux produces a compound action potential in nerve
m

or muscle is:
36. When heart rate is about 75 beats/min, the A. rheobase
duration of absolute refractory period of cardiac B. chronaxie
muscle is: C. twice rheobase
A. 50 ms D. twice chronaxie
Ai

B. 100 ms
C. 200 ms 44. Which one of the following accelerates
D. 300 ms contraction as well as relaxation of heart
muscle?
37. When heart rate is about 75 beats/min, the A. Norepinephrine
duration of ventricular systole is about: B. Calcium
A. 0.1 s C. Digoxin
B. 0.2 s D. Potassium
C. 0.3 s

19
45. Which of the following statements about B. does not occur in smooth muscle
cardiac muscle is incorrect? C. partly explains the release phenomenon that
A. Summation cannot occur occurs in neurological diseases
B. It is a fast muscle
C. Slow myosin ATPase activity Answers: Physiology of Nerve & Muscle Cells
D. It contains myoglobin 1B 2B 3B 4B 5D
6B 7A 8all 9B 10A
46. Which nerve fiber type is most susceptible 11D 12C 13A 14B 15BC
to conduction block by local anesthetics? 16D 17B 18B 19B 20C

in
A. Type A 21A 22A 23D 24C 25A
B. Type B 26all 27D 28A 29C 30A
C. Type C 31C 32D 33C 34E 35B
36C 37C 38A 39E 40B
47. Nociceptors (nerve endings signaling pain) 41D 42B 43A 44A 45B

s.
signal through: 46C 47CE 48A 49 50AC
A. A fibers
49: 1B, 2C, 3A, 4D
B. A fibers
C. A fibers
Answer Explanations:
D. A fibers
1. Fast anterograde transport from the cell
E. C fibers

A. type A
B. type B
iim
48. The nerve fiber type most susceptible to
conduction block by pressure is:
bodies to axons is brought about by kinesin. It
occurs at a speed of 400 mm/day.

4. You can predict this by using the Nernst


equation. Hypokalemia makes cells less
excitable by reducing RMP (i.e. making it more
C. type C
negative).
49. Neuromuscular transmission is impaired in
5. Reference: Ch 2, p 59 In: WFG 22nd ed.
4a
conditions listed below (in List A), however, the
pathogenesis of muscle weakness is different in
12. The reason is action potentials are all or
each. Match items in List A with those in List B.
none and occur with a constant magnitude.
List A
A. Cobra venom intoxication
21. Gap junctions are absent from nonsyncytial
B. Myasthenia gravis
smooth muscle (example, smooth muscle of
C. Botulism
iris). This makes possible fine graded
D. Lambert-Eaton syndrome
m

contractions of the iris and regulation of pupil


diameter.
List B
1. Antibodies to nicotinic Ach receptors in the
23. Skeletal muscle depends on the SR as the
motor end plate
major source of calcium. Cardiac muscle and
2. Toxin inhibits Ach release from presynaptic
smooth muscle depend on influx of calcium
Ai

nerve terminals in skeletal muscle


from ECF and Ca-induced calcium release for
neuromuscular junction
contraction.
3. Competitive blockade of nicotinic AchR in
skeletal muscle motor end plate
27. Note that ATP is required for muscle
4. Antibodies to voltage-gated calcium
contraction as well as relaxation.
channels in presynaptic terminals
28. At or above threshold intensity, an increase
50. Denervation hypersensitivity: in the frequency of stimulation (successive
A. is due to upregulation of neurotransmitter
stimulation rather) produces a gradual increase
receptors in the denervated structure

20
in the amplitude of successive contractions. This According to the size principle, small motor
is called staircase phenomenon or Treppe or units are recruited first and large motor units that
Bowditch effect. It occurs in skeletal as well as consist of large rapidly conducting axons are
cardiac muscle. In cardiac muscle, it is attributed recruited for more forceful contractions.
to increased availability of intracellular calcium
(beneficial effect of previous contractions).
Slow motor unit Fast motor unit
29. Classification of skeletal muscle fibers
Type I Type II Small, slow conducting Large, rapidly conducting
axons axons

in
Other names Slow, Fast,
oxidative, glycolytic, Innervate slow muscle Innervate fast fatigable
red white fibers (type I) muscle fibers (type II)
Myosin ATPase rate Slow Fast
Ca-pumping capacity of SR Moderate High 34. Gradation of force in skeletal muscle is
achieved by:

s.
Oxidative capacity High Low
(myoglobin, content of
recruitment of motor units
mitochondria, capillary the frequency of stimulation of motor units
density) asynchronous firing of motor units
variations in preload.
Fast muscle fibers (type II) have high myosin
iim
ATPase activity and their twitch duration is
short, whereas slow fibers (type I) have slow
myosin ATPase activity, and the twitch duration
is correspondingly longer.
Gradation of force is not achieved by varying
calcium release from the sarcoplasmic reticulum
because each action potential that enters the T
tubule releases a constant amount of calcium
from the terminal cisterns of the SR.
31. Types of muscle contraction
Isotonic contraction Isometric contraction 36. Cardiac muscle fibers are refractory during
Tension developed is Tension developed; Muscle the upstroke of the action potential and until at
constant; muscle shortens does not shorten least repolarization is half complete. This is
4a
External work is done External work is not done
Example: Walking, Trying to lift a heavy
called effective (or absolute) refractory period,
bending, running weight, sustained handgrip and the heart cannot be excited during this time
no matter how strong the stimulus is. In the
relative refractory period, a supranormal
W = Force displacement; since the muscle stimulus can evoke an action potential. Because
ends are fixed in an isometric contraction, no of the long duration of absolute refractory
external work is done in an isometric period, diastole commences before the heart is
m

contraction. reexcitable. Thus, summation of the type seen in


skeletal muscle cannot occur in cardiac muscle
32. A motor unit is the smallest amount of and why cardiac muscle cannot be tetanized.
muscle that can contract in response to This is a safety feature since the heart has to fill
excitation of a single motor neuron. with blood between successive contractions.
Ai

33. Motor unit: an alpha motor neuron and all 37. Events in the cardiac cycle
muscle fibers innervated by it. Electrical (depolarization and repolarization)
Innervation ratio (IR): number of muscle are followed by mechanical events (systole
fibers in a motor unit. and diastole)
In small motor units: IR 3-10 Cardiac cycle time: RR interval (variable)
In large motor units: IR >> 100 Action potential duration: variable
Motor point: point where nerve enters muscle If average HR is 75 bpm, then average
(anatomical) cardiac cycle time = 0.8 s = 800 ms
Average action potential duration ~ 250 ms

21
Duration of absolute refractory period ~ 200 intravesical pressure. This means that the
ms bladder is not shortening in this range because
Relative refractory period ~ 50 ms if the bladder shortened, the intravesical pressure
Note: Refractory period length depends would have increased.
upon the duration of the action potential. For
example, when HR is about 150 bpm, the Second, in a hollow viscus, wall tension T = Pr.
duration of the action potential is definitely i.e., P = T/r. When intravesical volume increases
shorter. from 50 to 350 ml, the bladder is stretched and
its radius increases. Stretch (the stimulus) elicits

in
38. Note that the terms slow and fast are applied some tension (the response) in the wall of the
to muscle based on twitch duration. Twitch bladder. This is called passive tension because it
duration is the duration of a single contraction does not cause the radius to decrease but merely
followed by relaxation. In slow type skeletal enables it to withstand the distending pressure.
muscle fibers, it could be as long as 100 ms. The On the other hand, when intravesical volume is

s.
twitch duration in cardiac muscle is normally increased to 400 ml, the bladder wall has been
800 ms (at a HR of 75 beats/min). The twitch adequately stretched and a strong reflex
duration could be as short as 10 ms as it is in contraction of the bladder occurs leading to
fast skeletal muscle. The minimum twitch shortening (active tension) of the detrusor and
duration in the heart is about 300 ms (at a heart emptying of the bladder
rate of 200/min). Smooth muscle is perhaps the

(tonic contraction).

40. Some important definitions:


iim
slowest. It is capable of sustained contraction

Preload: the load on a muscle before it


Total tension = active + passive tension.

As far as this question is concerned, this


discussion is intended to emphasize the fact that
the relation between length (in this example,
contracts. In the ventricle, the preload (end- intravesical volume) and tension is quite
diastolic fiber length) varies with the end- unpredictable in smooth muscle.
diastolic volume.
4a
42. The twitch duration in sphincters and smooth
Afterload: the load which contracting muscle muscle is particularly longer. Latch bridges are
has to overcome before it shortens. actomyosin complexes that detach slowly; this is
made possible by a slow myosin ATPase. In
Within physiologic limits, an increase in end- turn, this enables maintenance of tension for
diastolic fiber length increases tension longer periods without consumption of more
developed (force of ventricular contraction). ATP.
This is the Frank-Starling law of the heart.
m

Similarly, in skeletal muscle, there is a linear 43. The strength-duration curve is a plot of the
relationship between preload and force of strength of the electrical stimulus required to
contraction when other factors such as elicit compound action potentials in nerve or
recruitment are held constant. muscle against the duration of the stimulus.
Rheobase is the minimum stimulus strength that
Ai

However, in smooth muscle, length-tension will produce a response. Chronaxie has been
relationships are nonlinear over the physiologic defined as the minimum stimulus duration at
range. Consider for example the smooth muscle which twice the rheobasic current will produce a
of the bladder. Refer to the cystometrogram in a response.
normal human (Fig 38-25, p. 727, Ch 38, WFG,
2005). Note that an increase in intravesical 44. Norepinephrine has a positive chronotropic
volume from 0 to 50 ml is accompanied by an (increases rate of contraction), inotropic
increase in intravesical pressure. However, an (increases force of contraction), and lusitropic
increase in intravesical volume from 50 up to effect (i.e., it accelerates relaxation) on the heart.
380 ml is not accompanied by an increase in

22
45. Cardiac muscle is slow (i.e., its twitch
duration is longer compared to fast skeletal
muscle). The average twitch duration in the
heart is 800 ms when HR is 75 bpm.

47. Fast pain is transmitted through myelinated


A-delta fibers, and slow pain is the result of
transmission through unmyelinated C fibers.

in
49. You can view an animation of cholinergic
transmission onto nicotinic acetylcholine
receptors at http://tiny.cc/k45rs The full URL is
http://highered.mcgraw-
hill.com/sites/0072495855/student_view0/chapter14/animat
ion__chemical_synapse__quiz_1_.html [accessed 10

s.
May 2010]

iim
4a
m
Ai

23
3. FUNCTIONS OF THE
7. Presynaptic inhibition occurs at:
NERVOUS SYSTEM A. axoaxonal
B. axosomatic
Most of the questions in this section have only C. axodendritic synapses
one correct answer, as should be apparent from
the stem of the question; however, some 8. Renshaw cell inhibition of alpha motoneurone
questions may have more than one correct is an example of:
answer in that case, please check all correct A. negative feedback inhibition

in
answers. B. feedforward inhibition
C. presynaptic inhibition
1. Which of the following types of cells work as D. postsynaptic inhibition
scavenger cells in the CNS? E. indirect inhibition
A. Microglia F. direct inhibition
B. Oligodendroglia

s.
C. Ependymal cells 9. Which of the following is/are examples of
D. Astrocytes ligand-gated ion channels?
A. Nicotinic Ach receptor
2. Which is the commonest type of synapse in B. GABA-activated Cl channels
the CNS? C. IP3 receptor
A. Axosomatic synapse
B. Axodendritic synapse
C. Axoaxonal synapse
iim
3. Excitatory postsynaptic potentials (EPSP) are
D. Glycine receptor

10. Which of the following statements about


synaptic potentials is incorrect?
A. They are propagated down the postsynaptic
not produced by: neuron
A. opening of Na channels B. They undergo spatiotemporal summation
B. closure of K channels C. They are analogous to generator potentials
4a
C. opening of calcium channels and end-plate potentials
D. opening of chloride channels D. IPSP hyperpolarize the postsynaptic neuron
E. They are proportional to the amount of
4. Inhibitory postsynaptic potentials (IPSP) may transmitter released by the presynaptic
be produced by: neuron.
A. closure of Na channels
B. closure of Ca channels 11. Is conduction of nerve impulse in motor
C. opening of K channels axons unidirectional or bidirectional? Explain.
m

D. opening Cl channels
12. Most excitatory neurotransmission in the
5. The minimum time for transmission across brain is mediated by:
one synapse is: A. glutamate
A. 0.5 ms B. glycine
Ai

B. 1 ms C. GABA
C. 1.5 ms D. GnRH
D. 2 ms
13. The inhibitory amino acid neurotransmitters
6. Inhibitory postsynaptic potentials are an in the CNS are:
example of: A. glutamate and glycine
A. postsynaptic inhibition B. glutamate and aspartate
B. presynaptic inhibition C. GABA and glycine
C. direct inhibition D. aspartate and glycine
D. indirect inhibition

24
14. The GABA-A receptor is a: B. Weber-Fechner law
A. Na channel C. Mullers law of specific nerve energies
B. Cl ion channel D. Bell-Magendie Law
C. Ca channel
D. Cation channel 22. The relation between the magnitude of
sensation and stimulus intensity is given by the:
15. IPSPs due to chloride influx are produced A. law of specific nerve energies
by: B. labeled line principle
A. acetylcholine C. Weber-Fechner law

in
B. GABA D. law of projection
C. glutamate
D. substance P 23. A single sensory axon and all of its
peripheral branches constitute a:
16. Which of the following neurotransmitters A. receptive field

s.
has both excitatory and inhibitory effects? B. sensory unit
A. Glycine C. dermatome
B. GABA D. sensory nerve
C. Aspartate
D. Glutamate 24. Intensity of a sensory stimulus (whether
threshold or subthreshold) is encoded by all of

A. Glutamate
B. Glycine
C. Acetylcholine
D. Substance P
iim
17. Which of the following is an excitotoxin? the following except:
A. size of generator potentials
B. frequency of action potentials in sensory
neurons
C. recruitment of sensory units
D. size of action potentials
18. Inhibitory interneurons in the spinal cord
release: 25. All of the following are slowly adapting
4a
A. glycine receptors except:
B. GABA A. muscle spindles
C. substance P B. carotid sinus
D. neuropeptide Y C. pain receptors
D. Pacinian corpuscle
19. Neurons mediating direct inhibition in the
spinal cord usually secrete: 26. A typical example of a monosynaptic reflex
A. glycine is:
m

B. GABA A. stretch reflex


C. substance P B. superficial abdominal reflex
D. glutamate C. withdrawal reflex
D. light reflex
20. Tetanospasmin inhibits the release of:
Ai

A. GABA 27. The receptor for the stretch reflex is the:


B. glutamate A. muscle spindle
C. glycine B. Golgi tendon organ
D. acetylcholine C. Pacinian corpuscle

21. No matter where a particular sensory 28. In the stretch reflex, the afferents are:
pathway is stimulated along its course to the A. I a fibers
cortex, the conscious sensation produced is B. I b fibers
referred to the location of the receptor. This is: C. II fibers
A. the law of projection D. A delta fibers

25
29. The term small motor nerve system refers 37. All neural influences affecting muscle
to: contraction ultimately funnel through:
A. Ia afferents from muscle spindle A. -motor neurons
B. -motor neurons B. -motor neurons
C. -motor neurons C. corticospinal tract
D. basal ganglia
30. In the stretch reflex, relaxation of antagonist
muscles is due to: 38. Which of the following is the best example

in
A. reciprocal innervation of a polysynaptic reflex?
B. presynaptic inhibition A. Stretch reflex
C. Ia fiber stimulation B. Axon reflex
D. antidromic conduction C. Inverse stretch reflex
D. Withdrawal reflex

s.
31. The muscle spindle is made to contract by
stimulation of: 39. In humans, spinal shock is characterized by:
A. -motor neurons A. hypertonia
B. -motor neurons B. hyperreflexia
C. both of the above C. spastic paralysis
D. loss of autonomic function

stimulation of:
A. -motor neurons
B. -motor neurons
C. both of the above
iim
32. A muscle can be made to contract by
40. Which of the following is not a feature of
lower motor neuron lesions?
A. Muscle atrophy
B. Fasciculations
C. Fibrillations
33. The Jendrassiks maneuver facilitates stretch D. Denervation hypersensitivity
reflexes by: E. Spastic paralysis
4a
A. exciting alpha motor neurons
B. increasing gamma efferent discharge 41. Locomotion generators in spinal cord are
C. releasing glycine turned on by pattern generators in the:
A. suppressor strip
34. The Hoffmann reflex (H reflex) is typically B. midbrain
recorded from: C. medulla
A. soleus D. pons
B. orbicularis oculi
m

C. diaphragm 42. The crossed extensor response is seen in:


D. thenar muscles A. normal humans
B. the phase of spinal shock
35. The receptor for the inverse stretch reflex is: C. chronic paraplegia
A. muscle spindle
Ai

B. Golgi tendon organ 43. The mass reflex:


C. C fiber A. is a sign of a central inhibitory state
D. delta fibers B. occurs in the decerebrate animal
C. is due to irradiation of afferent impulses
36. Impulses from Golgi tendon organ pass to D. is a manifestation of release of spinal
inhibitory interneurons in the spinal cord via: reflexes from inhibitory control by higher
A. I a centers
B. I b
C. II a 44. Which of the following fibers has the
D. C fibers greatest threshold?

26
A. Touch C. two-point discrimination
B. Pain D. pain perception
C. Pressure
D. Cold 52. Which one of the following neural pathways
is predominantly an uncrossed pathway?
45. The gate theory of pain was proposed by: A. Somatosensory pathway
A. Charles Sherrington B. Gustatory pathway
B. Wall and Melzack C. Visual pathway
C. Weber-Fechner D. Auditory pathway

in
46. Anterolateral cordotomy does not interfere 53. Axons of ganglion cells in the retina
with perception of: terminate in the:
A. pain A. lateral geniculate nucleus
B. pressure B. pretectal nucleus

s.
C. temperature C. suprachiasmatic nucleus
D. fine touch D. superior colliculus

47. Which of the following procedures has not 54. The light reflex is integrated in the:
been tried out in the past to relieve intractable A. midbrain
pain? B. frontal eye field
A. Decortication
B. Cingulate gyrectomy
C. Anterolateral cordotomy
D. Prefrontal lobotomy
iim C. medulla
D. primary visual area

55. Which one of the following is not a


component of the near response?
48. Joint position sense is transmitted by: A. Pupillary constriction
A. anterior spinothalamic tract B. Convergence of the visual axes
B. lateral spinothalamic tract C. Increase in convexity of the lens
4a
C. dorsal column-medial lemniscal system D. Ciliary muscle relaxation

49. Which of the following is/are termed 56. Which of the following statements about
synthetic senses? Argyll-Robertson pupil is incorrect?
A. Vibratory sensibility A. Light reflex is absent
B. Fine touch B. Accommodation reflex is present
C. Joint position sense C. The lesion is in the Edinger-Westphal
D. Stereognosis nucleus
m

E. Two-point discrimination D. It occurs in neurosyphilis

50. Impaired stereognosis in the absence of a 57. Central visual fields are mapped with:
detectable defect in touch and pressure sensation A. tangent screen (campimeter)
points to a lesion in: B. perimetry
Ai

A. the dorsal column


B. the medial lemniscus 58. Impedance matching is a function of:
C. the parietal lobe posterior to the postcentral A. scala media
gyrus B. endolymph
D. any of the above C. ear ossicles and tympanic membrane
D. cochlear nucleus
51. Ablation of somatosensory area (SI) does
not significantly impair: 59. What is the amplification provided by the
A. joint position sense lever action of the auditory ossicles and the large
B. touch localization

27
size of the tympanic membrane compared to the B. saccule
oval window? C. posterior semicircular canal
A. 5 times D. utricle
B. 10 times
C. 22 times 67. Taste projection area is located in the:
D. 45 times A. foot of the postcentral gyrus
B. precentral gyrus
60. Endolymph is produced by: C. cingulate gyrus
A. stria vascularis

in
B. inner hair cells 68. Which of the following reflexes is always
C. sustentacular cells absent in the blind?
D. outer hair cells A. Vestibulo-ocular reflex
B. Nystagmus
61. Which of the following structures is most C. Visual accommodation

s.
likely involved in the spatial localization of D. Light reflex
sound?
A. Spiral ganglion 69. Saccades are programmed in the:
B. Hair cells A. medial longitudinal fasciculus
C. Superior olivary complex B. frontal cortex
D. Cochlear nerve C. hypothalamus

A. Reissners membrane
B. Tectorial membrane
iim
62. Which of the following functions as a
frequency analyzer in hearing?
D. superior colliculus

70. In which layer of the cortex do specific


thalamic afferents end?
A. Layer VI
C. Basilar membrane B. Layer IV
D. Middle ear ossicles C. Layer I
D. Layer II
4a
63. Inner hair cells in the organ of Corti are
depolarized by: 71. In which layer of the cerebral cortex do non-
A. K influx specific thalamocortical projections end?
B. Na influx A. Layer I-IV
C. Ca influx B. Layer II
D. closure of K channels C. Layer IV
D. Layer VI
64. Hyperpolarization of receptor cells activates:
m

A. phototransduction 72. Alpha block is produced by:


B. auditory signaling A. mental arithmetic
C. gustation B. stimulation of ascending reticular activating
D. olfactory signaling system
C. opening one eye only
Ai

65. Which of the following changes occur in rod


cells when rhodopsin is activated by light? 73. A 47 Hz rhythm normally occurs in:
A. Increase in cGMP A. hypothalamus
B. Deactivation of phosphodiesterase B. hippocampus
C. Depolarization of rod cells C. parieto-occipital area
D. Decreased release of neurotransmitter D. frontal area

66. Linear acceleration in the vertical plane is 74. A low-frequency, high amplitude,
signaled by hair cells in: synchronized cortical rhythm occurs during:
A. anterior semicircular canal A. stage 1 & 2 slow-wave sleep

28
B. stage 3 & 4 slow-wave sleep B. Decerebrate rigidity
C. REM sleep C. Extensor rigidity
D. narcolepsy D. Cog-wheel rigidity

75. Delta waves (0.5-4 Hz) are seen in: 82. Which of the following is absent in
A. stage 1 sleep decorticate animals?
B. stage 4 sleep A. Gross movements
C. an awake adult with eyes open B. Temperature regulation
D. an awake adult with eyes closed C. Hopping and placing reaction

in
D. Vestibulo-ocular reflex
76. Which of the following is not a
characteristic of REM sleep? 83. Which of the following is not a feature of
A. Slow waves corticospinal tract disease?
B. Hypotonia A. Cogwheel rigidity

s.
C. Dreaming B. Spasticity
D. PGO spikes C. Plantar extensor response
D. Exaggerated deep tendon reflexes
77. Gamma oscillations in EEG, which occur
when an individual is aroused and focuses 84. All neural influences affecting muscle
attention on something, occur at a frequency of: contraction ultimately funnel through:
A. 00.4 Hz
B. 47 Hz
C. 012 Hz
D. 3080 Hz
iim A. -motor neurons
B. -motor neurons
C. corticospinal tract
D. basal ganglia

Control of Posture and Movement: 85. Lesions of the ventral corticospinal tract
78. Guarding is an example of: result in:
A. stretch reflex A. difficulty with balance
4a
B. flexor withdrawal reflex B. deficits in skilled voluntary movements
C. clasp knife effect
D. - linkage 86. Righting reflexes are integrated for the most
part in the:
79. The procedure in which the hindbrain and A. medulla
the spinal cord are separated from the rest of the B. midbrain
brain by transection at the superior border of C. spinal cord
pons is called: D. cerebral cortex
m

A. decortication
B. decerebration 87. The cerebellum receives sensory input from
C. deafferentation the:
D. rhizotomy A. labyrinth
B. proprioceptors
Ai

80. Which of the following is pathognomonic of C. eyes


decerebration? D. all of the above
A. Exaggerated standing
B. Babinski sign 88. Climbing fiber input to the cerebellum
C. Absence of saccades comes from:
D. Relief of spasticity by cerebellectomy A. superior olivary complex
B. red nucleus
81. Which type of rigidity is clinically more C. inferior olivary complex
common? D. locus ceruleus
A. Decorticate rigidity

29
89. Feedforward inhibition has been well B. Sweating
described in neural circuits in: C. Curling up in a ball
A. spinal cord D. Insensible water loss
B. cerebellum
C. stretch reflexes 97. The process of consolidation; i.e., the
D. nociceptive pathway conversion of short-term memories to long-term
memories occurs in the:
90. The dolls eye reflex is integrated in the A. hippocampus
A. cerebral cortex B. amygdala

in
B. medial longitudinal fasciculus C. cerebral cortex
C. thalamus D. hypothalamus
D. medulla
98. Which of the following is/are an example(s)
91. Hypothalamus does not play a prominent of associative learning?

s.
role in the regulation of: A. Habituation
A. food and water intake B. Sensitization
B. temperature C. Classical conditioning
C. respiration D. Avoidance responses
D. osmolality
99. Intercortical transfer of memory occurs via:

in:
A. anorexia
B. obesity
C. hypersexuality
iim
92. Lesions of ventromedial hypothalamus result A. corpus callosum
B. anterior commissure
C. posterior commissure
D. all of the above

D. amnesia 100. A long lasting facilitation of transmission


in neural pathways following a brief period of
93. The circadian rhythm generator in the high frequency stimulation is termed
4a
hypothalamus receives inputs chiefly from: A. long-term potentiation (LTP)
A. retina B. post-tetanic potentiation (PTP)
B. superior colliculus C. sensitization
C. thalamus D. habituation
D. medial geniculate nucleus
101. Which receptor is directly involved in long-
94. Vanilloid receptors are involved in term potentiation?
transduction of: A. Cholinergic receptors
m

A. pain and temperature B. P2 receptor


B. benign tactile stimuli C. NMDA receptor
C. itch and tickle D. GABA-B receptor
D. vibration and joint position sense
102. Emotional responses often outlast the
Ai

95. When ambient temperature is about 20C, duration of the stimulus because:
body heat is lost chiefly by: A. limbic cortex is made up of three layers
A. radiation and conduction B. neocortex cannot modify emotional behavior
B. vaporization of sweat C. limbic circuits have prolonged
C. respiration afterdischarge
D. urination and defecation.
103. Disinhibition is a striking feature of disease
96. Which of the following is not a heat loss of:
mechanism? A. frontal lobe
A. Panting B. temporal lobe

30
C. occipital lobe 6. Classification of synaptic inhibition:
D. parietal lobe Direct Postsynaptic Glycine
Inhibition inhibition producing IPSP
on postsynaptic
104. The categorical hemisphere is chiefly neurons
concerned with:
A. language Indirect Presynaptic GABA producing
B. recognition of faces inhibition inhibition inhibition at
C. stereognosis axoaxonal
synapses
D. spatiotemporal relations

in
Classification of synaptic inhibition based on
105. The representational hemisphere is
structure of neural circuits:
concerned with:
Negative feedback inhibition: Renshaw cell
A. spatiotemporal relations
inhibition is a classic example of negative
B. stereognosis
feedback inhibition. Renshaw cells are

s.
C. recognition of faces
inhibitory interneurons in the spinal cord.
D. recognition of musical themes

Answers: Functions of the CNS


1A
6AC
11
16A
21A
26A
2B
7A
12A
17A
22C
27A
3D
8ADF
13C
18A
23B
28AC
iim
4all
9all
14B
19A
24D
29C
5A
10A
15B
20C
25D
30A
31B 32C 33B 34A 35B
36B 37A 38D 39D 40E
41B 42C 43CD 44B 45B
4a
46D 47A 48C 49ADE 50C
51D 52B 53all 54A 55D Feedforward inhibition occurs in cerebellar
56C 57A 58C 59C 60A circuits
61A 62C 63A 64A 65D
66B 67A 68C 69BD 70B 11. In the intact human, the action potential is
71A 72ABC 73B 74B 75B initiated at the axon hillock and propagates
76A 77D 78B 79B 80A through the axon membrane to the nerve
m

81A 82C 83A 84A 85A terminal. However, if we stimulated axons at the
86B 87D 88C 89B 90B median nerve with a threshold stimulus applied
91C 92B 93A 94A 95A through electrodes at the elbow, action potential
96C 97A 98CD 99D 100A in excited axons would travel orthograde to the
101C 102C 103A 104A 105all nerve terminal as well as retrograde into the cell
Ai

body located in the spinal cord. In a motor axon,


Answer explanations: it is anterograde transmission to the axon
2. 98% synapses in the CNS are axodendritic. terminal that can release neurotransmitter.
Retrograde conduction of the impulse to the cell
3. Opening of chloride ion channels in body depolarizes it sweeping it clean, and
postsynaptic membrane would produce rendering it refractory to stimulatory input for a
hyperpolarization of postsynaptic neurons. definite period of time.

16. Glycine, an inhibitory neurotransmitter in


the spinal cord, also has excitatory actions on

31
NMDA receptors. In the absence of glycine, Rapidly-adapting Pacinian corpuscle
glutamate cannot exert its excitatory effects on touch receptor Meissners corpuscle
the NMDA receptor. See p. 110, Ch 4, WFG, Slowly-adapting touch Merkels disks
2005. receptor Ruffini endings

17. Classification of glutamate receptors: Touch: Pacinian corpuscle


Metabotropic Vibration: Pacinian corpuscle
Ionotropic Pressure is sustained touch. Vibration is
NMDA receptor (Na & Ca channel) repetitive touch stimuli. Pressure is presumably

in
AMPA receptor coded by slowly adapting touch receptors.

Uncurbed excitatory neurotransmission In a teleological sense, if the carotid sinus


adapted permanently to an increase in arterial
Increase in glutamatergic transmission pressure, it wouldnt play a role in the long term

s.
control of arterial pressure. It is important to
Reason: Glutamate reuptake defective? note that adaptation can occur at the level of
receptors (peripheral resetting) as well in the
Increase in intracellular calcium in postsynaptic neural circuits in the CNS processing
neurons information from the receptor (this is called

Neuron damage and death iim


18. Golgi bottle neurons and Renshaw cells are
two classic interneurons in spinal cord. Both
release glycine. Both cause direct inhibition of
central resetting).

26. Even in the stretch reflex, an interneuron is


interposed between the Ia afferent and the alpha
motoneurone innervating the antagonist muscle;
however, this is perhaps the best answer for the
postsynaptic neurons. question.

20. Tetanospasmin produces spastic paralysis by 31, 32. Muscle spindle is a stretch receptor.
4a
preventing the release of glycine from inhibitory Contraction of the intrafusal fibers stretches the
interneurons in the spinal cord. central portion of the intrafusal fiber (muscle
spindle). The afferents from the muscle spindle
Botulinum toxin causes flaccid paralysis by pass via Ia and II fibers. The stretch reflex
inhibiting the release of Ach from the operates to regulate muscle length.
neuromuscular junction.
A- motor neurons innervate the intrafusal
Strychnine blocks glycine receptors. fibers whereas the extrafusal fibers (the regular
m

contractile units of skeletal muscle) are made to


22. Weber-Fechner Law (Power law): contract by stimulation of alpha motor neurons.
Magnitude of the sensation felt (R) = KSA where
S is stimulus intensity. A- motor neurons constitute the final common
path for all neural influences causing muscle
Ai

Mullers law of specific nerve energies (the contraction.


labeled line principle):
Each sensory pathway starting from the receptor Monosynaptic stretch reflex (example: knee
up to the cortex is labeled for transducing a jerk)
particular sensory modality. Stimulus Stretch of muscle
Receptor Muscle spindle
Bell Magendie law: In spinal nerves, dorsal Afferents Ia and II fibers
Integration Spinal cord
roots are sensory and ventral roots are motor.
Response Contraction of muscle
Latency 20 ms
25. Touch receptors: Significance It assesses the integrity of the reflex

32
arc and higher influences upon it paraplegics (i.e. following
Diminished or lost Lower motor neuron type disease, recovery from spinal
in phase of spinal shock, shock)
deafferentation,
Exaggerated in With Jendrassiks maneuver and The crossed extensor response, which occurs
conditions increasing gamma motor
neuron discharge, upper motor due to irradiation of afferent stimuli, occurs only
neuron type disease, chronic spinal in spinal animals that have recovered from
animals / humans spinal shock. It is a sign of release of spinal
H reflex The pathway for this reflex is the lower motor neurons from the inhibitory control
same as any monosynaptic reflex of higher centers.

in
(MSR); it is a sensitive means of
assessing integrity of MSR when
reflexes are not clinically elicitable, 39. Spinal shock is presumably due to sudden
as can occur in acute onset loss of tonic bombardment of -motor neurons
weakness. However, it is recorded by supraspinal neurons.
only from soleus (by EMG). The H

s.
reflex is absent in Guillain-Barre
syndrome. 40. Spasticity is a release phenomenon. It occurs
as a result of release of brain stem motor
35. Golgi tendon organ is a stretch receptor neurons from inhibitory control by higher
located in series with collagen fibers in the centers (i.e. the cerebral cortex and the basal
muscle tendon; it is innervated by Ib fibers ganglia)
iim 43. The mass reflex is a sign of a central
excitatory state; it is presumably a manifestation
of release of spinal lower motor neurons from
inhibitory control by higher centers. Denervation
supersensitivity may contribute to the
Inverse stretch reflex (autogenic inhibition)
occurrence of the mass reflex.
Excessive stretch
45. The gate theory of pain posits that there is a
4a
mechanism to facilitate or inhibit transmission
Activation of Ia afferents
of nociceptive impulses at the level of the spinal
cord. The putative gate is thought to be located
Forceful contraction of muscle
at the substantia gelatinosa of the spinal cord.
Higher centers exert influences on this gate. For
Stretch activation of Golgi tendon organ in
example, serotonin released from raphe spinal
tendon (activation of Ib afferents)
neurons inhibits onward transmission of
m

nociceptive inputs (endogenous analgesia


Muscle relaxation
system).
38. Characteristics of withdrawal reflexes:
47. Ascending pathways and the sensory
Stimulus: Noxious stimulus
modalities they code:
Dorsal column Fine touch, touch localization,
Ai

Integration: Spinal cord


lemniscal system two-point discrimination,
Control: Normally suppressed by
vibration, joint position sense
higher centers
Anterior spinothalamic Pressure, crude touch
Response: Withdrawal of affected
tract
body part
Lateral spinothalamic Pain, temperature
Exaggerated in Chronic spinal animals as
tract
a result of release from
inhibitory control from
higher centers. Thus the 49. Synthetic senses are synthesized in the
crossed extensor sensory association area in the brain (posterior
component of the response parietal cortex) from component senses.
is seen only in chronic

33
Stereognosis is a good example of a synthetic cochlea can be accessed at http://tiny.cc/lsbbc
sense. The full URL is http://highered.mcgraw-
hill.com/sites/0072495855/student_view0/chapter19/animation__e
ffect_of_sound_waves_on_cochlear_structures__quiz_1_.html
50. Since touch and pressure are both intact, one [Accessed 10 May 2010]
could presume that all sensory pathways right up
to the somatosensory cortex are intact. Impaired 59. Amplification by lever action of ear ossicles
stereognosis must therefore be due to a defect = 1.3
posterior to the somatosensory area (i.e. in the Ratio of surface area of tympanic membrane and
posterior parietal cortex). oval window = 17; total amplification = 17 1.3

in
= 22
51. Ablation of somatosensory area does not
result in impairment of pain perception since the 60. Endolymph is rich in K like ICF;
mere perception of pain does not require the Perilymph is rich in Na; like ECF;
cortex. Cortical processing is concerned Endolymph and perilymph do not mix.

s.
primarily with finer aspects of information
processing including discrimination of subtle 63. Hair cells are depolarized by K influx
differences in stimulus intensity, localization of because the endolymph is rich in K.
stimulus.
66.
53. Visual pathway: Retina >> Ganglion cells

cortex (V1), Area 17

Pathway for accommodation:


iim
>> Lateral geniculate nucleus >> Primary visual

Retina >> Ganglion cells >> Lateral geniculate


Stimulus
Linear acceleration in the
vertical plane
Linear acceleration in the
horizontal plane
Rotational acceleration
Sensory receptor
Saccule

Utricle

Semicircular canals in
the plane of the rotation
nucleus >> Primary visual cortex (V1), Area 17
>> Frontal eye field (Area 8) >> Edinger 67. Note that there is no separate taste area in the
Westphal nucleus (preganglionic brain as there is for other special senses.
4a
parasympathetic neurons) >> Ciliary ganglion Impulses from taste buds finally reach the foot
(parasympathetic postganglionic neurons) >> of the postcentral gyrus.
Iris, ciliary muscle
68. Think how the accommodation reflex is
Pathway for entrainment of circadian elicited. You ask a person to look at an object
rhythms: Ganglion cells >> Suprachiasmatic placed nearby after he looks at an object placed
nucleus (SCN) >> other relays in the farther away. Obviously, this is integrated in the
hypothalamus >> brain stem reticular formation cerebral cortex and a person who is blind (on
m

>> autonomic efferents both eyes) will not have this reflex.

56. Light reflex absent; accommodation reflex 70, 71. Types of thalamocortical projections:
present Argyll Robertson pupil; lesion in the Specific: function to transmit information to
pretectal nucleus of the midbrain where light specific areas in the cortex; results in sensory
Ai

reflex is integrated. perception; they end in layer IV of the cortex


Nonspecific: function to arouse the individual;
Light reflex present; accommodation reflex end in layers I-IV of the cortex.
absent Wernickes pupil. In this situation, the
lesion could be present anywhere beyond the 70 75. Notes on EEG:
exit of the fibers from the optic nerve to the In an adult human at rest with mind wandering
pretectal nucleus. and eyes closed, the most prominent component
of the EEG is a fairly regular pattern of waves at
58. A narrated animation of function of the a frequency of 8-12 Hz, and amplitude of about
tympanic membrane, middle ear ossicles and the 50 microvolts when recorded from the scalp.

34
This pattern is the alpha rhythm. It is most Function: Generation of dream imagery?
prominent in the parieto-occipital area.
Preparatory Overview for Questions 78-88
The alpha rhythm is replaced by a fast irregular An overview of control of posture and
low voltage beta rhythm (1830 Hz). This movement
phenomenon is called alpha block. A breakup What is meant by posture
of the alpha rhythm is produced by any kind of What is meant by movement
sensory stimulation such as mental arithmetic. What are the posture regulating pathways?
What are the movement regulating

in
Large amplitude, regular 47 Hz waves called pathways?
the theta rhythm occurs in children and is
generated in the hippocampus in experimental Animal preparations used to study the
animals. regulation of posture:

s.
Stages of sleep: Stages I IV of NREM (slow Spinal preparation: spinal cord transection
wave sleep) followed by rapid eye movement at midthoracic level
(REM) sleep Decerebrate preparation: transection of the
neuraxis at the superior border of pons
Sleep spindles (occur in stage II sleep) Midbrain preparation: transection of the
Frequency: 1014 Hz; amplitude: 50 microvolts.

EEG waves
Delta
Theta
Alpha
Beta
Frequency
0.54 Hz
47 Hz
813 Hz
1430 Hz
iim
neuraxis on top of the midbrain
Decortication: removal of cerebral cortex
alone leaving subcortical structures intact

What does each of the following mean?


Upper motor neuron (UMN)
Gamma 3080 Hz Lower motor neuron (LMN)
Pyramidal pathway
Delta waves occur in deep sleep (stages III and Extrapyramidal pathway
4a
IV). Lateral system pathways
Medial system pathways
REM sleep is called paradoxical sleep because
it is marked by rapid, low voltage, irregular EEG
activity. The threshold for sensory arousal from
REM sleep is sometimes greater than NREM
sleep presumably because the brain is actively
m

processing information during REM sleep


(dreams).

74, 75. EEG during sleep:


Stage Characteristics
Stage 1 Alpha rhythm
Ai

Stage 2 Sleep spindles and K complexes (beginning of


slowing; reduction in alpha content)
Stages 3 Characterized by slow waves (theta and delta)
&4
REM High frequency, low amplitude rhythm similar
sleep to the wakeful state
The mechanism of decerebrate rigidity:
Pontogeniculooccipital (PGO) spikes
Origin: Pons
Termination: Occipital cortex
Neurotransmitter: Acetylcholine

35
reflexes
Medulla Labyrinthine righting reflexes,
tonic neck reflexes
Midbrain Righting reflexes
Cerebral Hopping and placing reactions,
cortex optokinetic reflexes

Classification of movements:
Reflexive All reflexes

in
movements
Rhythmic Swallowing
movements
Voluntary Writing, talking, skilled
movements movements

s.
Classification of upper motor neurons:
Posture regulating upper motor neurons Descending motor pathways:
Movement regulating upper motor neurons Lateral system pathways
Lateral corticospinal tract
Posture regulating upper motor neurons: Rubrospinal tract
Origin: Brain stem
Termination: iim
On alpha and gamma motor
neurons in spinal cord
Controlled by: Inhibitory inputs from cerebral
cortex (suppressor strip 4s)
in the anterior edge of the
Medial system pathways:
Ventral corticospinal tract
Reticulospinal tract
Vestibulospinal tract
Tectospinal tract

precentral gyrus and basal Lateral system pathways are movement


ganglia. regulating pathways. If we could manage to cut
only lateral corticospinal tract axons (for
4a
Movement mediating / regulating upper example, by sectioning pyramids in monkeys),
motor neurons: posture would not be affected but the animal
Origin: Corticospinal tract (layer V, would lose its ability to perform fine skilled
pyramidal cell axons) from movements. The affected muscles would
primary motor cortex, sensory become hypotonic.
cortex, and premotor area.
Termination: On alpha-motor neurons / Medial system pathways are posture regulating
m

interneurons innervating alpha pathways.


motor neurons in spinal cord
Lesions: limited to the corticospinal and Other descending monoaminergic pathways
corticobulbar tracts produce Raphespinal pathways
weakness rather than Ceruleospinal pathway
paralysis and the affected
Ai

musculature is generally Please do read this post at http://tiny.cc/5u9pr


hypotonic. Isolated The full URL is http://groups.google.com/group/medical-
physiology-at-som-
involvement of corticospinal aimst/browse_thread/thread/0cd009733e4080b8# on
tract is not clinically common. consequences of use of misleading terminology
regarding descending motor pathways.
Level of integration of principal postural
reflexes:
Level Postural reflexes
Spinal cord Stretch reflexes, withdrawal
Cerebellum:

36
Functional divisions of the cerebellum
Division Function 94. Vanilloid receptors are present in nociceptor
Spinocerebellum Maintenance of nerve terminals as well as fibers transducing
(midline & paravermal zone) posture
Vestibulocerebellum Maintenance of
heat and cold sensations. See p.123, Ch 5, WFG,
(flocculonodular lobe) posture and 2005.
equilibrium
Neocerebellum Coordination of 97. Notes on memory:
(lateral cerebellar hemispheres) voluntary movement Memory is retention and storage of learnt
information.

in
Output from the Purkinje cells to the deep
cerebellar nuclei is always inhibitory. Implicit memory is reflexive
Output from the deep cerebellar nuclei to the (nondeclarative). It is nondeclarative in that
thalamus is excitatory. one is not aware of its execution. Examples
Cerebellum facilitates stretch reflexes and include skills one has perfected.
usually hypotonia occurs in cerebellar

s.
lesions. Explicit memory (which is declarative)
Selective ablation of the flocculonodular requires conscious recall of events or facts.
lobe of the cerebellum has been shown to It requires processing in the hippocampus.
abolish the symptoms of motion sickness.
The symptoms of motion sickness occur due Working memory is a form of short term
iim
to excessive stimulation of the vestibular
system via its connections to the brain stem
and the flocculonodular lobe of the spinal
cord. See p.184, Ch 9, WFG, 2005.
memory used to plan actions; example,
dialling a phone number one has just seen on
the phone book. It is stored in prefrontal
cortex.

88. The only source of climbing fiber input to Consolidation refers to the formation of
the cerebellum is the inferior olivary complex. new long term memories from short-term
There is evidence that this pathway is activated memories, and it occurs in the hippocampus.
4a
when a new motor task is learnt. The hippocampus is rich in NMDA
receptors. NMDA receptors are a type of
90. This is the vestibulo-ocular reflex, used as a glutamate receptors.
test of brain stem integrity, in patients who are
comatose. The absence of the dolls eye reflex in Short-term memory is prone to erasure;
comatose patients is a grave prognostic sign. long-term memory (true memory) is
resistant to erasure.
92. Hypothalamus and food intake:
m

Satiety center works by inhibiting feeding center Short term memory (STM) Hippocampus
Feeding center located in Lateral hypothalamus Working memory Prefrontal lobes
Satiety center located in Ventromedial hypothalamus Long term memory (LTM) Neocortex

Lesions in ventromedial Obesity


nucleus lead to
Mechanisms implicated in memory:
Post-tetanic potentiation
Ai

Lesions in lateral Anorexia


hypothalamus lead to Long-term potentiation
Changes in synaptic strength
93. Circadian rhythms are 24-h biological Increases in synaptic contacts
rhythms entrained by the light-dark cycle. They
are paced in the suprachiasmatic nucleus (SCN) Post-tetanic potentiation: Enhanced
in the hypothalamus. The SCN receives neural postsynaptic potentials after a brief tetanizing
input from retina. In blind individuals, circadian train of stimuli in the presynaptic neuron.
rhythms are free running and have a
periodicity of about 25 hours.

37
Long-term potentiation: Long-lasting limbic system of reward centers and
facilitation of transmission in neural pathways punishment centers
following a brief period of high-frequency We may apply this concept to reform
stimulation. This process is important for society BF Skinner
consolidation, i.e. the formation of long term
memories. Habituation: The response to a benign stimulus
repeated over and over gradually decreases.
Conditions which influence consolidation This is due to decreased release of excitatory
(formation of new LTM from STM) neurotransmitter from the presynaptic neuron.

in
Repetition of stimulus (facilitates) The opposite response where the response to a
Sleep facilitates consolidation noxious stimulus repeated over and over
Convulsions, ECT (inhibit) gradually increases is termed sensitization.
Anesthetics, tranquilizers (inhibit)
Hypoxia (inhibits) Both habituation and sensitization are examples
Inhibitors of protein synthesis (inhibit)

s.
of nonassociative learning because the organism
learns about a single stimulus.
98. Types of learning:
Associative learning Nonassociative 99. Limbic system:
learning The seat of emotions
Associating a conditioned Learning about a single
It is three layered
stimulus with a neutral
stimulus
Examples include
classical (Pavlovian)
conditioning, and operant /
instrumental conditioning
iim stimulus

Examples include
habituation and
sensitization
It is phylogenetically older compared to
neocortex
It has reciprocal connections with cortex;
thus it is possible to control emotions.
It has connections with reticular activating
system; thus, emotions can impact upon
Classical conditioning
visceral function.
(Pavlovian conditioning)
There is considerable reverberation of
Conditioned stimulus (food)
4a
impulses in limbic circuits (Papez circuit);
Conditioned response (salivation)
this is possibly why emotions last longer and
Conditioned reflex
are memorized.
Neutral stimulus (bell ringing)
Pairing the neutral stimulus with the
conditioned stimulus several times
The neutral stimulus (bell ringing)
eventually elicits the same response as the
m

conditioned stimulus (food).


Conditioned reflexes (or behavior) are
classic examples of associative learning.

Instrumental conditioning
(Synonymous with operant conditioning)
Ai

This phenomenon was described by


B.F.Skinner (19041990)
Here the animal learns by operating on the
environment.
The main learning point here is that animal
behavior is determined by the likely
consequences of that behavior. From
experiments conducted with the Skinner box
arose concepts such as the presence in the

38
4. ENDOCRINOLOGY & Signal transduction cascades and second
messengers:
REPRODUCTION Signal transduction Second messenger(s)
cascades
Adenylate cyclase cascade cAMP
Types of intercellular communication: Phosphatidyl inositol Inositol trisphosphate (IP3),
Type Mediators pathway DAG, and Ca
Neurocrine Neurotransmitters acting across synapses Guanylyl cyclase cGMP
Endocrine Hormones acting on target cells located at Hormones acting upon Hormone-receptor complex
distant sites intracellular receptors
Paracrine Hormones acting locally without entering

in
the general circulation
Gap junctions Cells are electrically coupled by gap Drugs / toxins used in G-protein research:
junctions which allow passage of ions and Drug/toxin Mechanism of action
small molecules Cholera toxin ADP-ribosylation of subunit of Gs,
Juxtacrine Cells linked to adjacent cells through cell activation of adenylate cyclase
adhesion molecules and cell adhesion Pertussis toxin Inhibition of subunit of Gi

s.
molecule receptors in extracellular matrix Forskolin Directly activates adenylate cyclase

Link to animations on cellular mechanism of


Some notes on G-proteins:
action of chemical messengers [accessed 10
1. They have 3 subunits (, , );
May 2010]
2. Each subunit is distinct (heterotrimeric)
http://tiny.cc/1malr
The full URL is iim
http://highered.mcgrawhill.com/sites/0072507470/student_view0/c
hapter17/animation__second_messenger__camp.html

http://tiny.cc/6vea6
The full URL is
3. They are guanylyl nucleotide (GTP / GDP)
binding proteins
4. The subunit has inherent GTPase activity
5. The and subunits are coupled to effectors
molecules like adenylyl cyclase, ion
channels.
http://highered.mcgraw- 6. G proteins couple hormone binding to the
hill.com/sites/0072507470/student_view0/chapter17/animation__
membrane-bound_receptors_that_activate_g_proteins.html receptor to an intracellular signaling
pathway. In essence, they are signal
4a
http://tiny.cc/r96s0 coupling proteins.
The full URL is 7. Hormone receptors that are coupled to G-
http://highered.mcgraw- proteins are called G-protein coupled
hill.com/sites/0072507470/student_view0/chapter17/animation__i receptors (GPCR)
ntracellular_receptor_model.html
8. GPCR have seven transmembrane domains;
Components of signal transduction pathways thus they are often called seven-helix
and their functions: receptors or serpentine receptors
m

Components Functions 9. There are several types of G proteins; Gs is


Hormone / neurotransmitter Signal stimulatory G protein and it activates
Receptor Recognizes the signal adenylyl cyclase; Gi is inhibitory G protein
Signal coupling proteins Coupling ligand binding to and it inhibits adenylyl cyclase; activation of
intracellular signaling Gq leads to activation of the membrane
Second messengers Intracellular molecules that
bound enzyme phospholipase C.
Ai

amplify extracellular
hormonal (first messenger) 10. Other families of proteins such as ras
signals several fold resemble G-proteins in structure and
Effector molecules Enzymes, channels, function.
transporters, that mediate 11. Mutations in G proteins are implicated in the
the physiologic effects of
the hormone development of cancer.

39
MECHANISMS OF INTERCELLULAR B. protein kinases A (PKA) and C
COMMUNICATION C. cAMP and cAMP dependent PKA
1. How many transmembrane domains do G- D. cGMP and calcium
protein coupled receptors have?
A. 2 9. Most of the calcium in the endoplasmic
B. 3 reticulum (ER) is sequestered by:
C. 7 A. calmodulin
D. 12 B. caldesmin
C. calbindin

in
2. How many subunits do G proteins have? D. calsequestrin
A. One
B. Two 10. Cytosolic calcium concentration in unexcited
C. Three cells is about:
D. Four A. 2.5 mM

s.
B. 10 mM
3. Which G protein activates adenylyl cyclase? C. 150 M
A. Gs D. 100 nM
B. Gq
C. Gi 11. Which one of the following stimulates the
release of calcium from the endoplasmic
D. Gl

GTPase activity?
A. Alpha
B. Beta
iim
4. Which subunit of G protein has intrinsic
reticulum?
A. IP3
B. PTH
C. 1, 25 (OH)2 D3
D. DAG
C. Gamma
D. Delta 12. Protein kinase C is activated by:
A. IP3
4a
5. Which of the following hormones does not B. diacylglycerol (DAG)
act via a G-protein coupled receptor? C. cAMP
A. TRH D. guanylyl cyclase
B. Angiotensin II
C. ADH 13. All of the following chemical messengers act
D. Thyroxine via receptor tyrosine kinases except:
A. insulin
B. ANP
m

6. Which toxin inhibits the function of Gi?


A. Cholera toxin C. epidermal growth factor
B. Pertussis toxin D. platelet derived growth factor
C. Forskolin
D. Saxitoxin 14. Guanylyl cyclases are activated by:
A. CO, NO and ANP
Ai

7. Which receptor does not span the cell B. NO, ANP and ET1
membrane seven times? C. AII, NO and ANP
A. Rhodopsin D. ANP, ET1 and AII
B. Beta-adrenergic receptor
C. Insulin receptor 15. Which of the following hormones mediate(s)
D. M2 receptor its effects by activating nuclear / cytosolic
receptors?
8. The second messengers in the phosphatidyl A. Thyroxine
inositol cascade are: B. Retinoic acid
C. Steroid hormones
A. IP3, DAG and calcium
40
Answers: Mechanisms of Intercellular B. NO and CO
Communication C. glycine and glutamate
1C 2C 3A 4A 5D D. FSH and LH
6B 7C 8A 9D 10D
11A 12B 13B 14A 15all 7. Growth hormone secretion is increased by all
of the following except:
******************************** A. cortisol
B. hypoglycemia
ENDOCRINOLOGY & C. exercise

in
REPRODUCTION D. protein meal
1. Which of the following hormones is not a 8. Which of the following hormones has
glycoprotein? intrinsic lactogenic activity?
A. TRH A. TSH
B. FSH

s.
B. MSH
C. LH C. GH
D. hCG D. dopamine
2. Which of the following is not synthesized 9. A human growth hormone variant (hGH-V) is
from prepro-opiomelanocortin (pre-POMC)? expressed primarily in:
A. MSH
B. ACTH
C. Endorphin
D. Dynorphin
iim
3. Which of the following hormones does not
A. liver
B. spleen
C. kidney
D. placenta

10. Check all correct statements.


act through G-protein coupled receptors? A. The beta cell releases equimolar amounts of
A. Dopamine insulin and C-peptide
B. Epinephrine
4a
B. NSILA is due to circulating C-peptide
C. Angiotensin II C. C-peptide has a shorter half-life compared to
D. ACTH insulin
E. Retinoic acid D. C-peptide level in plasma provides an index
F. Thyroxine of B-cell function in patients receiving
exogenous insulin.
4. The group of chemical messengers whose
actions are known to be mediated by receptor 11. Which of the following increases insulin /
m

tyrosine kinases includes: glucagon molar ratio the most?


A. angiotensin II, ANP and ET 1 A. A large carbohydrate meal
B. EDRF, ANP and AVP B. Intravenous glucose
C. FSH, LH and GHRH C. Overnight fast
D. insulin, EGF, IGF 1, PDGF D. A small protein meal
Ai

E. Low carbohydrate diet


5. Section of the pituitary stalk does not inhibit
the secretion of _____from the anterior pituitary. 12. Arrange the following steps in the order in
A. growth hormone which they occur in the thyroid gland.
B. FSH 1. Coupling reaction
C. LH 2. Iodination
D. prolactin 3. Endocytosis of thyroglobulin
4. Oxidation of iodine
6. The term neurohormone is applied to: 5. Iodine uptake
A. oxytocin and vasopressin 6. Cleavage of thyroglobulin

41
7. Secretion of T4 20. Which of the following hormones has
8. Iodide trapping maximum glucocorticoid activity?
A. Cortisol
13. Bone resorption is induced by: B. Corticosterone
A. IGF-1 C. Deoxycorticosterone
B. insulin D. Cortisone
C. estrogens
D. glucocorticoids 21. In humans, the most abundant adrenocortical
hormone is:

in
14. Check all correct statements regarding A. cortisol
oxytocin. B. corticosterone
A. It is essential for milk ejection C. deoxycorticosterone
B. It increases uterine contractility D. DHEA sulfate
C. It facilitates ejaculation of semen

s.
D. It facilitates sperm transport in the uterus 22. The major adrenal androgen is:
A. etiocholanolone
15. Calcitonin inhibits the activity of: B. testosterone
A. osteoblasts C. dihydrotestosterone
B. osteoclasts D. dehydroepiandrosterone (DHEA)
C. parafollicular cells E. androstenedione
D. thyroid

A. insulin
B. glucagon
iim
16. Somatostatin inhibits the secretion of:
23. In which zones is corticosterone converted to
aldosterone?
A. Zona glomerulosa
B. Zona fasciculata
C. growth hormone C. Zona reticularis
D. gastrin
24. The hormone that is synthesized only in the
4a
17. Hans Selye defined stressors specifically zona glomerulosa is:
as stimuli that increase the secretion of ______ A. DHEA
in healthy humans or laboratory animals. B. aldosterone
A. CRH C. corticosterone
B. ACTH D. deoxycorticosterone
C. TSH
D. cortisol 25. Injection of pharmacologic doses of ACTH
in experimental animals produces all of the
m

18. Which of the following organelles is a major following except an:


site of steroidogenesis? A. increase in cortisol
A. Peroxisomes B. increase in lymphocyte count
B. Ribosomes C. increase in skin pigmentation
C. Smooth endoplasmic reticulum D. increase in aldosterone secretion
Ai

D. Rough endoplasmic reticulum


26. Glucocorticoids decrease the number of
19. Which of the following hormones has the circulating:
highest mineralocorticoid activity? A. eosinophils
A. Cortisol B. lymphocytes
B. Corticosterone C. platelets
C. Deoxycorticosterone D. red blood cells
D. Cortisone
27. Which of the following does not occur as a
consequence of adrenalectomy?

42
A. Decreased ability to excrete a water load B. FSH
B. Glucose intolerance C. growth hormone
C. Increased vulnerability to hypotension D. inhibin
D. Reduced alertness
E. Dextrose fever 35. The fetal adrenal predominantly secretes:
A. progesterone
28. Insulin sensitivity is not reduced in: B. androstenedione
A. glucagon-secreting tumors C. DHEA
B. tumors of adrenal medulla D. DHEA sulfate

in
C. tumors of adrenal cortex
D. hypopituitarism 36. The principal estrogen secreted by the
fetoplacental unit is:
29. Which of the following peptides has been A. estriol
shown to have significant incretin effects? B. estradiol

s.
A. Vasoactive intestinal polypeptide C. estrone
B. Somatostatin
C. Glucagon like polypeptide 37. 21 hydroxylase deficiency results in:
D. Pancreatic polypeptide A. congenital adrenal hyperplasia
B. testicular feminization syndrome
30. Which of the following is the first of a new C. male pseudohermaphroditism

B. Adiponectin
C. Resistin
iim
class of incretin mimetics approved for use in
the treatment of type II diabetes?
A. Exenatide
38. The virilizing form of adrenogenital
syndrome usually occurs due to a deficiency of:
A. 21 hydroxylase
B. 17 hydroxylase
D. Leptin C. 11 hydroxysteroid dehydrogenase
D. 3 hydroxysteroid dehydrogenase
31. Which of the following hormone peaks
4a
during the mid-luteal phase? 39. The hypertensive form of congenital adrenal
A. Inhibin hyperplasia is due to a deficiency of:
B. Activin A. 11 hydroxylase
C. Follistatin B. 21 hydroxylase
D. Relaxin C. 3-hydroxysteroid dehydrogenase
D. DHEA sulfokinase
32. Which of the following causes luteolysis?
A. PGF2 alpha 40. Which of the following almost guarantees
m

B. Progesterone maleness?
C. Estradiol A. Presence of Y chromosome
D. Relaxin B. Functional embryonic testes
C. Absence of Barr body in neutrophils
33. hCG can be detected in the blood of a D. Well developed male external genitalia
Ai

pregnant woman as early as ___ days post


conception. Answers: Endocrinology & Reproduction
A. 2 days 1A 2D 3EF 4D 5D
B. 6 days 6A 7A 8C 9D 10AD
C. 10 days 11A 13D 14all 15B
D. 14 days 16all 17A 18C 19C 20A
21D 22D 23A 24B 25B
34. Human chorionic gonadotropin is 26AB 27B 28D 29C 30A
structurally and functionally similar to: 31A 32A 33B 34A 35D
A. LH

43
36A 37A 38A 39A 40D Uterine contractions at the time of sexual
intercourse promote sperm transport in the
Answer explanations: uterus
Contraction of vas deferens and propulsion
1. TRH is a tripeptide of sperm toward urethra (during ejaculation)

2. Pre-proopiomelanocortin (POMC) derivatives 17. Hans Selye defined stress as any stimulus
are: that increases the release of ACTH in
Beta-endorphin experimental animals or normal humans.

in
MSH (alpha, beta and gamma)
ACTH 21. DHEA levels peak in the third decade of life
Beta-lipotropin (LPH) and decline thereafter. Thus, it has been looked
CLIP upon as a marker of aging. There is no evidence
at the moment that DHEA supplementation

s.
3. Retinoic acid, thyroxine, steroid hormones act reverses changes associated with aging.
mainly through receptors located in the cytosol However, the most abundant adrenocortical
or nucleus of target cells. However, recent hormone in plasma is DHEAS. Thus, it is
evidence suggests that at least some of the appropriate to use plasma DHEAS level as an
physiologic effects of steroid hormones like index of functioning adrenocortical mass.
estrogens, aldosterone are mediated via
iim
receptors located in the cell membrane. These
actions have come to be therefore called
nongenomic actions of steroid hormones.

5. Normally, dopamine from the hypothalamus


26. Glucocorticoids decrease the number of
eosinophils, basophils and lymphocytes in the
circulation.

Glucocorticoids given in pharmacologic doses


inhibits the release of prolactin. suppress cell mediated immunity by:
inhibiting IL-2 gene transcription
6. Neurohormones are hormones synthesized inhibiting T cell proliferation
4a
and secreted by neurons into the general inducing apoptosis in lymphocytes
circulation
27. Mechanism of dextrose fever in
10. Non-suppressible insulin like activity adrenocortical insufficiency:
(NSILA) is insulin activity that is not suppressed Suppose large amounts of 5% dextrose (an
by anti-insulin antibodies. It is due to insulin- isoosmotic solution) is administered,
like growth factors IGF-I (somatomedin C) and
IGF-II. Dextrose is metabolized
m


12. Steps in thyroid hormone synthesis: The water added dilutes plasma; however water
1. Iodine uptake (Iodide trapping) excretion is defective
2. Oxidation of iodine
3. Iodination of tyrosine residues Plasma osmolality decreases
Ai

4. Coupling of iodotyrosines
5. Endocytosis of thyroglobulin Water enters cells
6. Cleavage of thyroglobulin
7. Secretion of T4 Water influx into temperature-sensitive neurons
disrupts function of the hypothalamic thermostat
14. Functions of oxytocin:
Milk ejection Fever
Uterine contractions at the time of labor

44
28. Diabetogenic hormones (hormones that
reduce insulin sensitivity and thus increase Development of the male external genitalia
plasma glucose levels): requires the presence of significant amounts of
Epinephrine dihydrotestosterone (the biologically most active
Norepinephrine androgen) and normal androgen receptors.
Glucagon
Growth hormone Barr bodies are visible as drumstick
Cortisol chromosomes in only about 1-15% of
Thyroxine neutrophils. Barr bodies are absent in individuals

in
with Turners syndrome. However, patients with
29. Incretins are gastrointestinal hormones that Turners syndrome are not male.
increase insulin secretion in response to an oral
glucose load. Gender is assigned at birth on the basis of
Examples: external genitalia. The presence of well-
Glucose dependent insulinotropic

s.
developed male external genitalia at birth
polypeptide (GIP) is the most important signifies that genetic sex is male, testes are
incretin functional, and dihydrotestosterone was present
Glucagon-like polypeptide (GLP) has also in adequate amounts. The purpose of the
been shown to have incretin effects. question is to clarify the difference between the
terms genetic sex, Gonadal sex and phenotypic

diabetes.
iim
30. Exenatide is an incretin mimetic. It is a GLP
analog and has been approved for use in type 2

31. Inhibin from the ovary inhibits the release of


sex.

Note the exact meaning of these terms:

Genetic sex
44 XY (male) 44XX (female)

FSH from the pituitary. Testes (male) Ovaries (female)


Gonadal sex
34. HCG is a glycoprotein with 2 subunits & Phenotypic sex Male external Female external
4a
(at birth) genitalia genitalia
The subunit is identical with that in TSH, LH
& FSH. The subunit confers specificity. When
[hCG] is high, stimulation of TSH receptors Supplement on steroidogenesis
could result in mild hyperthyroidism. HCG
maintains the corpus luteum of pregnancy; it is What happens in the placenta?
luteotropic in contrast to LH which is Placentas priority is progesterone and the
luteinizing; indeed both HCG and LH act upon maintenance of pregnancy.
m

the same receptor.


Cholesterol
40. The Y chromosome contains the testis- Side chain cleavage enzyme
determining gene (or SRY); this gene encodes a Pregnenolone Fetus DHEA
transcription factor that is essential for the
development of embryonic testes. Presence of Y Progesterone Fetus Cortisol
Ai

chromosome merely dictates the formation of


embryonic testes.

Testosterone, one of the hormones produced by


embryonic testes, induces the formation of male
internal genitalia but not external genitalia.
Mullerian inhibiting substance from the
embryonic testes causes regression of Mullerian
ducts.

45
What happens in the fetal zone of adrenal
cortex? This is recruited to produce DHEAS How is cortisol produced?
and 16-OH DHEAS
Cholesterol 17-hydroxy-pregnenolone
Side chain cleavage enzyme
Pregnenolone
17-hydroxylase 17-hydroxy-progesterone
17-hydroxy pregnenolone 21-hydroxylase
17, 20 - lyase

in
Dehydroepiandrosterone (DHEA) 11-Deoxycortisol
Adrenal sulfokinase 11-hydroxylase
Dehydropepiandrosterone sulfate (DHEAS)
Cortisol
What is the need for DHEAS and

s.
16-OH DHEAS? Why does virilization occur in CAH?
Fetal DHEAS Placenta Estradiol 21-hydroxylase deficiency
Excess progesterone &
Excess 17-hydroxy-progesterone
16-OH DHEAS Placenta Estriol
Androstenedione (large amounts)
iim
The placentas priority is progesterone and
pregnancy. The fetal zone of the adrenal, which
constitutes about 80% of the gland, degenerates
at the time of birth. The major function of the
fetal zone is to produce DHEAS. The placenta
Masculinizing effects in large concentrations

Congenital virilizing adrenal hyperplasia:


Block in synthesis of glucocorticoids
synthesizes estradiol from DHEAS and Increase in ACTH
maintains pregnancy. Compensatory adrenal hyperplasia
Increased synthesis of adrenal androgens
4a
What happens in zona glomerulosa? Manifestations in males:
Cholesterol Accentuation of male characteristics
Precocious pseudopuberty
Pregnenolone Manifestations in females:
Adrenogenital syndrome in newborns
Progesterone Precocious pseudopuberty
Virilization in postpubertal females
Deoxycorticosterone
m

Clinical presentation depends on severity of


Corticosterone deficiency:
1. Ambiguous genitalia at birth (female
Aldosterone pseudohermaphroditism)
2. Adrenogenital syndrome
Ai

What happens in zona fasciculata and zona 3. Precocious pseudopuberty in males


reticularis? (Fetal as well as adult) 4. Virilization in postpubertal females

Pregnenolone

Progesterone
21-hydroxylase
11-Deoxycorticosterone
11-hydroxylase
Corticosterone

46
in
s.
steroidogenesis in the adrenal cortex:
Enzyme
17-
hydroxylase
iim
Consequences of deficiencies of enzymes involved in

Consequences of deficiency
Rare, no sex hormones, female genitalia
develop regardless of genetic sex,
characterized by an increase in plasma
corticosterone, hypertension &
hypokalemia
21- 95% congenital adrenal hyperplasia is due
hydroxylase to deficiency of this enzyme, salt losing
4a
form of CAH
11- Cortisol deficiency; virilization due to
hydroxylase excessive synthesis of adrenal androgens;
hypertension due to mineralocorticoid
activity of high levels of 11-deoxycortisol
m
Ai

47
5. GASTROINTESTINAL C. Peyers patches
D. gut associated lymphoid tissue
PHYSIOLOGY
8. Stomach is indispensable because it produces:
1. Excitatory motor neurons in the myenteric
A. hydrochloric acid
plexus release:
B. pepsin
A. Ach
C. intrinsic factor
B. VIP
D. rennin
C. NO
D. NE

in
9. The postprandial alkaline tide is most likely to
be abolished by:
2. Postganglionic parasympathetic neurons
A. vagotomy
innervating the gastrointestinal smooth muscle
B. omeprazole
are located in:
C. gastrin
A. myenteric plexus

s.
D. calcium
B. submucosal plexus
C. paravertebral ganglia
10. The pattern of electrical and motor activity
D. prevertebral ganglia
in the gastrointestinal tract during periods of
fasting is called:
3. The term brain of the gut is used to refer to
A. basic electrical rhythm
the:
A. autonomic ganglia
B. enteric nervous system
C. migratory motor complex
D. interstitial Cells of Cajal
iim B. migrating motor complex (MMC)
C. peristalsis
D. segmentation

11. The major humoral mediator of meal-


stimulated enzyme secretion is:
4. Which hormone(s) is/are produced in the
A. secretin
stomach?
B. CCK
A. Ghrelin
4a
C. GIP
B. Motilin
D. gastrin
C. Secretin
D. VIP
12. The major humoral mediator of pancreatic
duct cell secretion of bicarbonate rich juice is:
5. Which one of the following is not a
A. secretin
prominent stimulator of gastric acid secretion?
B. CCK
A. Prostaglandin E2
C. somatostatin
m

B. Gastrin
D. pancreozymin
C. Acetylcholine
D. Histamine
13. The most potent stimulus for secretin release
from the duodenum is:
6. Vagotomy does not prevent gastric acid
A. acid load
secretion occurring in response to:
Ai

B. peptide
A. injection of histamine
C. fatty acid with > 8 carbons
B. injection of insulin
D. carbohydrate
C. sight and smell of food
D. emotions
14. Which of the following statements regarding
somatostatin is incorrect?
7. Enterocytes are replenished by mitotically
A. It inhibits release of growth hormone
active undifferentiated cells located in:
B. It inhibits release of insulin
A. Brunners glands
C. It decreases blood flow to the intestine
B. crypts of Lieberkuhn
D. It stimulates gastric acid secretion

48
C. segmentation
15. The major humoral mediator of gall bladder D. mass action contraction
contraction in response to a fat meal is:
A. CCK 23. The most frequent type of movement in the
B. gastrin small intestine in the digestive state is:
C. secretin A. peristalsis
D. somatostatin B. antiperistalsis
C. slow wave
16. Which is the most important cholagogue? D. segmentation

in
A. Secretin
B. CCK 24. The type of intestinal contraction that
C. Gastrin hastens transit in the GIT in the digestive state
D. GIP is:
A. peristalsis

s.
17. The most important choleretic(s) is/ are: B. segmentation
A. bile salts C. tonic contraction
B. CCK D. migrating motor complex
C. secretin
D. gastrin 25. The amount of protein lost in the stool is
about:

because they:
iim
18. Bile salts are essential for absorption of fat

A. solubilize dietary lipids in micelles


B. increase surface tension
C. are hydrophobic
A. 0.5 g/day
B. 30 g/day
C. 5 g/day
D. 2-3 g/day

D. contain cholesterol Answers: Gastrointestinal Physiology


1A 2A 3B 4A 5A
19. Micelles in bile are formed by: 6A 7D 8C 9B 10B
4a
A. bile salts and phospholipids 11B 12A 13A 14D 15A
B. bile acids and bile salts 16B 17A 18A 19A 20B
C. cholesterol and bile salts 21B 22BD 23D 24A 25A
D. cholesterol and phospholipids
Answer explanations:
20. The total circulating bile salt pool is 4. Ghrelin stimulates food intake. It is also a
approximately: stimulator of the growth hormone secretagogue
A. 35 mg
m

receptor in the anterior pituitary, a receptor that


B. 3.5 g mediates growth hormone release.
C. 150 mg
D. 30 g 6. Phases of gastric acid secretion:
Cephalic phase: Vagally-mediated responses
21. Most of the water in the gastrointestinal tract induced by activity in the CNS. It is abolished
Ai

is absorbed in the: by atropine / vagotomy.


A. duodenum Gastric phase: Local reflex responses induced
B. jejunum by presence of food in the stomach.
C. ileum Intestinal phase: Reflex and hormonal feedback
D. colon effects on gastric secretion initiated from the
mucosa of the intestine. The effect of this is to
22. The type(s) of contraction that normally mainly inhibit gastric acid secretion.
occur(s) only in the colon is/are:
A. peristalsis
B. antiperistalsis

49
10. The migrating motor complex sweeps
secretions from the mid-stomach through to the
terminal ileum. The MMC presumably serves to
prepare the gut for the next meal; i.e. it is the
housekeeper of the small intestine. It is
augmented by motilin.

17. Choleretics stimulate bile secretion. Bile


salts are themselves amongst the most potent

in
choleretics. Cholagogues (e.g. cholecystokinin)
increase bile flow. CCK enhances bile flow into
the intestine by stimulating contraction of the
gall bladder.

s.
20. The total bile salt pool of 3.5 g recycles
repeatedly via the enterohepatic circulation six
to eight times per day.

22-24. Gastrointestinal motility


In any discussion on GI motility you can build

Propulsive movements vs. mixing


movements
iim
your ideas along the following three lines:
Digestive vs. interdigestive motility

Anatomical: stomach, duodenum, proximal


colon, sigmoid colon etc.

Digestive motility Interdigestive motility


4a
Peristalsis MMC
Segmentation
Haustration

Propulsive movement Mixing movement (fosters


digestion & absorption)
Peristalsis Segmentation
MMC Haustral shuttling
m

Variations in frequency of BER:


Region Frequency of BER
Stomach 34/min
Duodenum 12/min
Ileum 8/min
Ai

Proximal colon 2/min


Sigmoid colon 6/min

Rate of peristalsis: The rate of peristalsis is


quite variable (225 cm/min). Each peristaltic
wave is propagated usually for a short distance
(45 cm). On the other hand, the rate of MMC is
about 5 cm/min. The MMC operates from mid
stomach until the terminal ileum. It is not
propagated any further.

50
6. CIRCULATING BODY FLUIDS Working definitions:
Normocytes MCV within normal limits
Macrocytes MCV > 95 fL
Red blood cell indices Microcytes MCV < 80 fL
What are the RBC indices? Hypochromia MCHC < 25 g/dL
How are they derived?
Mention normal values for each. 1. Macrocytes have an MCV > than:
How are they used in the differential A. 80 fL
diagnosis of anemias? B. 95 fL
Calculate RBC indices given that RBC C. 90 fL

in
count = 5 million/mm3, blood Hb = 15 g/dL, D. 85 fL
and PCV = 45%
2. Each red blood cell contains approximately:
RBC Derivation Normal A. 20 pg
Index values
B. 25 pg

s.
MCH Hb in g/dL 10 2932 pg C. 30 pg
RBC in millions/mm3
D. 35 pg of hemoglobin
MCV PCV 10 8095 fL
RBC in millions/mm3
3. The most reliable RBC index is:
MCHC Hb in g/dL 100 3235 g/dL A. MCV
PCV
iim
MCH, mean corpuscular hemoglobin; MCV, mean
corpuscular volume; MCHC, mean corpuscular hemoglobin
concentration; fL, femtoliter; pg, picogram.

I use this simple trick below to recall the right


B. MCH
C. MCHC
D. mean cell diameter

4. The amount of hemoglobin present in 100 ml


of red blood cells is called:
formula for each index, in case I forget them.
Normally, A. MCH
MCV = 90 = 450 / 5 = (PCV 10) B. MCHC
= 45 10 / 5 RBC C. Hb index
4a
MCH = 30 = 150 / 5 = Hb 10 D. Price Jones index
= 15 10 / 5 RBC count
MCHC = 33 = 1500 / 45 = Hb 100
= 15 100 /45 PCV
5. MCHC does not usually exceed:
A. 30 g/dL
MCHC = MCH/MCV; B. 33 g/dL
15 g of Hb is present in 100 ml of blood C. 37 g/dL
15 g of Hb is present in 45 ml of PRBC D. 25 g/dL
m

30 g of Hb is present in 90 ml of PRBC
33 g of Hb is present in 100 ml of PRBC 6. Primary hemostasis refers to cessation of
bleeding due to:
MCHC is the amount of Hb present in 100 ml of A. formation of a definitive clot
packed red blood cells. B. clot retraction
C. formation of a temporary platelet plug.
Ai

RBC indices are useful in the differential


diagnosis of anemias. Anemias may be classified 7. Platelet aggregation is stimulated by:
as macrocytic, microcytic or normocytic based A. thromboxane A2
on MCV, and normochromic or hypochromic B. fibrinogen
based on MCHC. Hyperchromia is uncommon C. ADP
but it may occur in hereditary spherocytosis. D. thrombin
E. epinephrine

8. Platelet aggregation is inhibited by:

51
A. ADP C. Factor V
B. 5-HT D. Antithrombin III
C. PG I2
D. TX A2 16. Factor V Leiden:
A. is a mutated form of factor IX
9. Which of the following clotting factors is not B. is inactivated by protein C
vitamin K dependent? C. is present in a large subset of patients with
A. Factor II venous thromboembolism
B. Factor V D. is available for clinical use

in
C. Factor VII
D. Factor IX 17. Which condition(s) is / are characterized by
E. Factor X an increase in both bleeding time and clotting
time?
10. The extrinsic pathway is triggered by the A. Afibrinogenemia

s.
release of: B. Hypoprothrombinemia
A. factor VII C. Hemophilia A
B. tissue factor D. Von Willebrands disease
C. tissue factor pathway inhibitor
D. contact factor 18. Red cell antigens A and B are chemically:
A. phospholipids

A. plasminogen
B. TPA
C. urokinase
D. plasmin
iim
11. The enzyme that ultimately lyses fibrin is: B. glycosphingolipids
C. glycopeptides
D. polypeptides

19. Red blood cell antigens A and B are also


present in:
12. The extrinsic pathway is inhibited by: A. saliva
A. tissue factor B. semen
4a
B. thromboplastin C. amniotic fluid
C. tissue factor pathway inhibitor (TFPI) D. pancreas
D. contact factor
20. A person with the Bombay phenotype does
13. The test that screens the extrinsic pathway is: not have:
A. prothrombin time (PT) A. glycophorins
B. activated partial thromboplastin time (aPTT) B. HLA on leukocytes
C. thrombin time C. H substance
m

D. urea solubility test D. cadherins


E. clot lysis time
21. CO is formed as an end product of:
14. Prolongation of prothrombin time does not A. heme metabolism
occur when there is a deficiency of only: B. arginine metabolism
Ai

A. factor VIII C. oxidation of acetoacetate


B. factor IX
C. factor VII 22. Heme is converted to bilirubin mainly in the:
D. vitamin K A. kidneys
B. liver
15. Mutations in which of the following have C. spleen
been implicated in the pathogenesis of D. bone marrow
hypercoagulable states?
A. Protein C 23. The protein that binds extracorpuscular
B. Protein S hemoglobin is:

52
A. hemin D. Fibrinogen
B. haptoglobin E. -globulin
C. hemopexin
D. haptopexin 31. Erythropoietin is produced by:
A. interstitial cells surrounding peritubular
24. Which of the following is not synthesized in capillaries in the kidneys
the liver? B. perivenous hepatocytes
A. IgG C. Kupffer cells of liver
B. 2 macroglobulin

in
C. Albumin 32. Hereditary spherocytosis occurs due to
D. Angiotensinogen mutations in genes coding for:
A. spectrin and ankyrin
25. Which of the following plasma proteins are B. Na-K ATPase
protease inhibitors? C. glucose 6 phosphate dehydrogenase

s.
A. 1 antitrypsin D. pyruvate kinase
B. Transferrin
C. C-reactive protein 33. The average half-life of neutrophils in the
D. Antithrombin III circulation is:
A. 6 hours
26. Which of the following is a negative acute B. 5 days
phase reactant?
A. Albumin
B. C-reactive protein
C. 2 macroglobulin
D. Transferrin
iim C. 2 weeks
D. 1 month

Answers: Circulating Body Fluids


1B 2C 3C 4B 5C
6C 7all 8C 9B 10B
27. ESR is increased in: 11D 12C 13A 14AB 15all
A. anemia 16C 17ABD 18B 19all 20C
4a
B. hypofibrinogenemia 21A 22C 23B 24A 25AD
C. spherocytosis 26A 27A 28C 29AC 30D
D. all of the above 31AB 32A 33A

28. The protein content of lymph draining from Answer Explanations:


the _______ is highest. 3. MCHC is the most reliable index because it
A. choroid plexus does not depend on the RBC count. This is
B. skeletal muscle
m

because the estimation of RBC count, done by


C. liver manual hemocytometry, is more error prone than
D. gastrointestinal tract the estimation of Hb or PCV.
29. Osmotic fragility of red blood cells is 8. ADP, TX A2, thrombin, serotonin, and
decreased in: epinephrine stimulate platelet aggregation.
Ai

A. sickle cell anemia Prostacyclin inhibits platelet aggregation.


B. hereditary spherocytosis Aspirin therapy lowers TXA2/PGI2.
C. microcytic hypochromic anemia
D. macrocytic anemia 9. Vitamin K dependent clotting factors are
factors II, VII, IX and X and Protein C and
30. When a serum sample is electrophoresed, Protein S. Protein C and protein S inhibit
which one of the following bands is absent? clotting
A. Albumin
B. 1 globulin
C. 2 globulin

53
10. The extrinsic pathway is called so because it hypofibrinogenemia and VWD. Thrombin is a
is triggered by a factor extrinsic to plasma potent stimulator of platelet aggregation. At
(tissue thromboplastin). Simply, it is triggered least, theoretically, prothrombin deficiency
by tissue injury. The extrinsic pathway is also would be expected to delay platelet aggregation.
the fastest limb of the clotting cascade.
Normally, PT = 12 14 seconds. 19. If red cell antigens are also present in
exocrine secretions, such individuals are called
11. Fibrinolytic system: secretors. In some, they are not secreted. The
Plasminogen to plasmin significance of this is unknown.

in
Enzyme: Tissue plasminogen activator (TPA)
Fibrin to fibrin degradation products 20. H substance is absent in persons with the
Enzyme: Plasmin (fibrinolysin) Bombay phenotype. Normally, if an individual
has H substance, his blood group would be O.
12. TFPI is tissue factor pathway inhibitor. An individual lacking H substance will have anti

s.
H antibodies in plasma and can receive
13. Prothrombin time screens the extrinsic limb transfusions only from a person with the
and the final common pathway of the clotting Bombay phenotype.
cascade. APTT screens the intrinsic limb of the
clotting cascade. 22. Heme oxygenase activity is highest in the
spleen. Second, the spleen is much more
Test
Bleeding time

Clotting time

Prothrombin time
Assessing
Platelets and
vessel wall
Intrinsic &
iim
Extrinsic pathway
Extrinsic pathway
Factors

VII, X,
sensitive to red cell injury. See Wintrobes Clinical
Hematology, Lee GR et al, 10th ed, Lippincott Williams
and Wilkins, volume 1, p 280.

23. Haptoglobin binds extracorpuscular


hemoglobin. Hemopexin binds heme.
& common Prothrombin,
pathway Fibrinogen Haptoglobin binds extracorpuscular hemoglobin
Activated partial Intrinsic pathway VIII, IX, X, XI, and prevents it from being filtered and excreted
thromboplastin & common XII by the kidney. Serum haptoglobin levels are
4a
time pathway reduced in hemolytic anemias.

15. Loss-of-function mutations in protein C, 26. Albumin is a negative acute phase reactant;
protein S, antithrombin III and mutations in i.e., its synthesis is reduced during the acute
factor V resulting in resistance to inactivation to phase response.
protein C have all been implicated in
hypercoagulable states. 27. Presumably, the increase in ESR in anemia
m

has little to do with rouleaux formation; ESR


16. Factor V Leiden, a mutated form of Factor may be high because both PCV and viscosity are
V, resists inactivation by protein C and is low which in turn reduce the resistance to
present in a large subset of patients with venous sedimentation of cells. By the same reasoning,
thromboembolism. ESR is reduced in polycythemia since the
viscosity (and therefore the resistance to
Ai

17. Fibrinogen is essential for platelet sedimentation) is high. However, if blood is


aggregation. Also fibrinogen is a clotting factor. more viscous because of an increase in plasma
Von Willebrands factor mediates the levels of globulins and fibrinogen, ESR is higher
attachment of platelets to subendothelial since these proteins facilitate rouleaux
collagen. Also vWD binds factor VIII and formation.
prolongs its half-life in the circulation. When
there is a deficiency of vWF, more factor VIII 29, 30. Osmotic fragility increases when the
spills over into the urine. Thus, bleeding time RBC cytoskeleton is abnormal, as in hereditary
and clotting time are both increased in spherocytosis.

54
Normal range: 0.5% - 0.3% NaCl
Increased in: hereditary spherocytosis
Decreased in: microcytic anemia & sickle cell
anemia.
Sickle cells and microcytes show greater
resistance to osmotic lysis by virtue of their
smaller size and normal cytoskeleton.

30. There is little fibrinogen in serum.

in
31. See p. 459, Ch 24, WFG, 2005.

32. Hereditary spherocytosis occurs due to


mutations in cytoskeletal proteins spectrin,

s.
ankyrin, Band 3. Defects in ankyrin are reported
to be commoner.
***************************************

iim
4a
m
Ai

55
7. CARDIOVASCULAR C. late depolarization of infarct
D. early repolarization of infarct
PHYSIOLOGY
6. Hypocalcemia is associated with QT
List of acronyms that are used in this section: prolongation because:
HR heart rate A. it is invariably associated with bundle
SBP systolic blood pressure branch block
DBP diastolic blood pressure B. it increases ventricular activation time
PP pulse pressure C. it lengthens the duration of ventricular

in
MAP mean arterial pressure repolarization
RPP rate-pressure product D. it accelerates opening of potassium channels
BPM beats per minute
RR RR interval 7. AH interval is normally:
Q (with a dot on top of it) cardiac output A. 3050 ms
TPR total peripheral resistance

s.
B. 60125 ms
TBF tissue blood flow C. 100200 ms
D. 80120 ms
Electrical properties of the heart
1. The heart continues to beat even after all 8. The ability of the AV node to generate its own
nerves to it are sectioned. This property is impulses when the sinus node is sick is due to:
called:
A. excitability
B. conductivity
C. automaticity
D. contractility
iim A. a constant phase 4 membrane potential
B. slow calcium entry during phase zero
C. spontaneous diastolic depolarization
D. the absence of prepotentials

9. The AV node does not conduct more than:


2. Prepotentials are normally absent from: A. 180
A. P cells in the SA node B. 230
4a
B. AV nodal cells C. 280
C. Purkinje fibers D. 330 impulses / minute
D. working myocardial cells
Cardiac cycle; systolic time intervals
3. What is the effect of vagal stimulation on the 10. Which of the following need to be recorded
membrane potential of the SA node? in order to determine systolic time intervals?
A. It increases an inward calcium current A. ECG, ECHO, PCG
B. It increases the slope of the prepotential B. ECG, PCG and carotid artery pulse (CAP)
m

C. It activates a hyperpolarizing potassium C. ECHO, CAP and PCG


current D. ECG, CAP and apexcardiogram
D. It increases intracellular cAMP
11. Which of the following is not essential to
4. Intrinsic heart rate can be determined by: determine QS2, left ventricular ejection time
Ai

A. vagotomy (LVET) and pre-ejection period (PEP)?


B. administration of atropine A. Pulse transducer
C. beta adrenergic receptor blockade B. Electrocardiograph
D. IV administration of atropine and atenolol C. Phonocardiogram
D. Swan-Ganz catheter
5. The ST segment is elevated in acute
myocardial infarction because of: 12. The dicrotic notch is absent from:
A. flow of current into the infarct during A. radial arterial pulse tracing
diastole B. pulmonary arterial pulse tracing
B. TP segment depression C. aortic pulse tracing

56
D. none of the above The Microcirculation; Dynamics of blood
flow
13. The maximum pressure rise in the ventricle 19. Windkessel vessels are represented by the:
occurs during: A. aorta
A. ejection B. internal jugular v.
B. isovolumetric contraction C. arterioles
C. protodiastole D. muscular arteries
D. diastasis
20. The stopcocks of the circulation are:

in
14. During the cardiac cycle, aortic valve closes A. arterioles
at the end of: B. capillaries
A. isovolumetric systole C. valves
B. rapid ejection D. venules
C. diastasis

s.
D. protodiastole 21. Across which site in the circulation is the
pressure drop maximum?
15. The cardiac output of a 50 year old man at A. Arterioles
rest is 6 L / min; mean HR is 75 BPM. Left B. Venules
ventricular end-diastolic volume (LVEDV) is C. Capillaries
120 ml. What is the mean ejection fraction?
A. 50 %
B. 66 %
C. 75 %
D. 35 %
iim D. Aortic valve

22. Which of the following structures are not


innervated?
A. Arterioles
B. Postcapillary venules
16. During exercise, a man consumes 2 liters of C. AV anastomoses
oxygen per minute. His arterial O2 content is 200 D. Precapillary sphincters
ml/L and the oxygen concentration of mixed
4a
venous blood is 120 ml/L. His cardiac output is: 23. At any time, the greatest fraction of blood is
A. 16 L/min present in the:
B. 32 L/min A. heart
C. 40 L/min B. arteries
D. 25 L/min C. veins
D. capillaries
17. Venous return is reduced by all of the
following except:
m

24. What fraction of total blood volume is


A. Valsalva maneuver present in the capillaries at any given time?
B. positive end-expiratory pressure A. 5%
C. intravenous bolus of frusemide B. 20%
D. deep inspiration C. 15%
D. 1%
Ai

18. If blood [Hb A] = 15 g/dL and Hb is fully


saturated with oxygen, average stroke volume = 25. The term capacitance vessels is applied to:
70 ml, average HR = 72/min, calculate whole- A. pulmonary capillaries
body oxygen delivery; i.e., the amount of B. thoroughfare channels
oxygen delivered to the tissues per minute. C. shunts
A. 1 L/min D. veins and venules
B. 0.5 L/min
C. 2 L/min 26. Hydraulic conductivity of capillaries is
D. 2.5 L/min highest in:
A. glomeruli

57
B. intestinal villi radius is accompanied by an increase in wall
C. skin tension.
D. brain
33. Bernoullis principle could be applied to
27. Rank the hydraulic conductivity of explain why:
capillaries in various parts of the body. A. intravesical pressure does not change with
A. GIT > glomerulus in kidney > lung > brain increase in intravesical volume
B. Glomerulus in kidney > brain > skin B. mean pressure in the femoral artery is
C. GI mucosa > heart > brain greater than that in the brachial artery

in
D. GI mucosa > circumventricular organs > C. coronary blood flow is compromised in
lung aortic stenosis
D. tissue blood flow ceases below critical
Biophysics of the circulation: closing pressure
28. Turbulence is almost always present when

s.
Reynolds number is more than: Regulation of cardiovascular function:
A. 2000 34. Quantitatively, the most important means of
B. 2500 increasing flow to an actively metabolizing
C. 3000 tissue is:
A. increasing cardiac output
29. The average arm-to-tongue circulation time B. increasing peripheral resistance
is:
A. 5 seconds
B. 10 seconds
C. 15 seconds
D. 20 seconds
iim C. increasing blood pressure
D. decreasing local vascular resistance

35. Which one of the following is not a


vasodilator metabolite?
A. Adenosine
30. Thin walled capillaries do not burst when B. Potassium ions
intracapillary pressure is increased because: C. Endothelin-1
4a
A. they lack smooth muscle cells D. Hydrogen ions
B. the blood flow rate is less E. ADP
C. they have a small radius
D. capillary hematocrit is less than whole-body 36. What is the chemical identity of
hematocrit. endothelium-derived relaxing factor (EDRF)?
A. Nitrous oxide
31. That capillaries can withstand high internal B. Nitric oxide
pressures without bursting is explained using: C. Potassium
m

A. Bernoullis principle D. Carbon monoxide


B. Laplaces law
C. Poiseuille Hagen law 37. Which one of the following does not have
D. Fahraeus-Lindquist effect vasodilator actions?
A. NO
Ai

32. It follows from Laplaces law that: B. CO


A. dilated hearts must generate greater tension C. Potassium
in order to generate a given pressure. D. Angiotensin III
B. thin-walled capillaries do not burst because
of a smaller radius. 38. Blood flow to exercising skeletal muscle is
C. deficiency of surfactant would result in lung increased by all of the following except:
collapse. A. K+
D. intravesical pressure does not increase when B. norepinephrine
bladder is filled because an increase in C. in PCO2 in muscle
D. adenosine

58
A. central blood volume increases
39. The most potent vasoconstrictor is: B. heart rate decreases
A. endothelin 1 C. central venous pressure decreases
B. angiotensin II D. discharge from arterial baroreceptors
C. norepinephrine decreases
D. vasopressin E. stroke volume decreases by about 40%

40. Check whether the following statements 45. Mareys law states that:
about the arterial baroreflex mechanism are true A. when BP increases, HR decreases

in
or false. B. when BP decreases, HR increases
1. Baroreceptors are free nerve endings C. when BP decreases, HR decreases
responsive to stretch. D. when BP increases, HR increases
2. Application of suction force with a neck
chamber loads the arterial baroreceptors. 46. An increase in whole body oxygen demand

s.
3. The effect of head-up tilt would be to unload is met chiefly by:
arterial baroreceptors in the carotid sinus. A. increasing cardiac output
4. Herrings nerve terminates in the nucleus B. increasing oxygen content of arterial blood
tractus solitarius. C. increasing oxygen extraction from arterial
5. Activity in the arterial baroreceptors reflexly blood
excites vagal outflow to the heart and D. increasing blood pressure

vessels.
iim
inhibits sympathetic outflow to resistance

6. The reflex effect of arterial baroreceptor


loading is a decrease in HR and MAP.
7. The reflex response to baroreceptor
47. During severe exercise, a well-trained athlete
may be able to achieve a cardiac output of:
A. 15 liters
B. 25 liters
unloading is a decrease in sympathetic C. 35 liters
outflow to the heart and blood vessels. D. 45 liters
4a
41. Intravenous injection of norepinephrine in 48. Vasomotor ischemia triggers an increase in
the intact animal leads to: sympathetic outflow increasing BP and thereby
A. an increase in BP & HR facilitating restoration of cerebral blood flow.
B. a decrease in BP & HR This is called:
C. an increase in HR & decrease in BP A. Bainbridge reflex
D. an increase in BP & decrease in HR B. the CNS ischemic pressor response
C. Heads paradoxical reflex
42. The first reflex response to an increase in D. Mareys reflex
m

arterial pressure produced by intravenous


injection of phenylephrine is: 49. Heart rate is slowed by:
A. increase in RR interval A. deep inspiration
B. increase in heart rate B. Bainbridge reflex
C. decrease in cardiac output C. increased intracranial tension
Ai

D. decrease in total peripheral resistance D. carotid massage

43. Which of the following maneuvers evokes 50. Heart rate is slowed by all of the following
an increase in vagal discharge to the heart? except:
A. IV infusion of phenylephrine A. deep expiration
B. Carotid massage B. fear
C. Pressure on the eyeball C. anger
D. Irrigation of the ear canals D. IV infusion of phenylephrine

44. On rising from the supine position:

59
51. Which of the following varies from beat-to- D. Bainbridge reflex
beat?
A. RR interval 58. Activity in the noradrenergic nerves to the
B. Venous return heart causes / has:
C. Stroke volume A. coronary vasoconstriction
D. Cardiac output B. coronary vasodilation
E. Blood pressure C. no effect on blood flow

52. During the strain phase of the Valsalva 59. If the noradrenergic nerves to the heart are

in
maneuver (forced expiration with the glottis stimulated after giving a -blocker, then what
open and maintaining an expiratory pressure of would be the effect on coronary blood flow?
40 mm Hg for 15 seconds): A. Coronary vasodilation
A. venous return decreases B. Coronary vasoconstriction
B. cardiac output decreases C. No change

s.
C. blood pressure decreases D. Unpredictable
D. heart rate increases
E. sympathetic outflow to blood vessels 60. Capillaries empty when mechanically
F. TPR gradually increases stimulated. This is called the:
A. axon reflex
53. Atrial natriuretic peptide is metabolized by: B. white reaction
A. dicarboxypeptidase
B. kininase II
C. neutral endopeptidase
D. kininase I
iim C. flare response
D. red reaction

61. The wheal in the triple response is due to:


A. contraction of precapillary sphincters
54. Cardiac output increases during: B. increased capillary permeability
A. Valsalva maneuver C. axon reflex
B. positive pressure ventilation D. decreased absorption of fluid
4a
C. negative g
D. head-up tilt 62. Which of the following physiologic
responses has a neural basis?
55. Which of the following does not occur A. Red reaction
following prolonged space missions? B. White reaction
A. Cardiac hypertrophy C. Flare
B. Postural hypotension D. Reactive hyperemia
C. Weight loss
m

D. Motion sickness 63. Of the following vascular beds,


autoregulation of tissue blood flow is least
Regional circulation: prominent in the:
56. The arteriovenous O2 concentration A. skin
difference is highest across the: B. heart
Ai

A. brain C. brain
B. heart D. kidneys
C. kidneys
D. liver 64. When determining BP with a
sphygmomanometer, a spuriously high value (of
57. The mechanism that regulates cerebral blood either SBP or DBP) may be recorded when:
flow during cerebral compression is the: A. there is an auscultatory gap
A. CNS ischemic response B. the cuff is smaller than preferable
B. Cushings reflex C. the person is obese
C. Bezold-Jarisch reflex D. the arm is not placed at the level of the heart

60
resting heart rate in healthy humans is around 70
Answers: beats per minute indicates that the effects of
1C 2CD 3C 4D 5all cardiac vagal tone at rest are greater than the
6C 7B 8C 9B 10B effect of sympathetic outflow to the heart.
11D 12D 13B 14D 15B Patients with transplanted hearts have higher
16D 17D 18A 19A 20A resting heart rates closer to IHR because of
21A 22D 23C 24A 25D cardiac denervation. Also, because of
26A 27C 28C 29C 30C denervation, they are less able to increase their
31B 32all 33C 34D 35C heart rate in response to exercise.

in
36B 37D 38B 39A 40
41D 42A 43all 44CDE 45AB 5. The infarct is deprived of blood supply and
46A 47C 48B 49D 50C consequently its RMP becomes less negative.
51all 52all 53C 54C 55A What do you expect the effect of ATP depletion
to be on RMP? RMP will become less and less
56B 57AB 58B 59B 60B

s.
negative. The other reason why RMP becomes
61B 62C 63A 64all
less negative is that potassium is lost from cells
40: statement 7 is false; others are true
as a result of injury. Thus, the infarcted zone
loses its surface positivity and consequently it is
Answer Explanations:
negative with respect to surrounding normally
1. The automaticity is attributed to the presence
polarized tissue (surface positive, inside

extrinsic innervation. See below.


iim
of pacemaker cells which demonstrate
spontaneous depolarization in the absence of

2. Note that the terms prepotential, pacemaker


potential and spontaneous diastolic
negative). Therefore, during diastole,
extracellularly, current flows into the infarct.
This depresses the baseline, i.e., the TP segment
is depressed. However, the arrangement in ECG
recorders is such that TP segment depression is
recorded as ST segment elevation.
depolarization are often used interchangeably.
They all mean the same. Normally prepotentials
Secondly, the infarct depolarizes late with
are present only in the SA node and AV node
4a
respect to surrounding normal tissue probably
which contain pacemaker cells (P cells).
due to decrease in conduction velocity in the
infarcted tissue; the effect of this late
However, in abnormal situations (e.g.
depolarization is to cause ST segment elevation.
hypoxemia) other regions of the heart (e.g. a
ventricular focus) exhibit prepotentials. This
Finally, ischemic myocardium repolarizes faster
state is one of increased automaticity, and
due to accelerated opening of K channels. The
premature ventricular or atrial depolarizations
effect of early repolarization is also ST segment
m

are manifestations of increased automaticity.


elevation. Thus, myocardial infarction is
3. Stimulation of the right vagus nerve, which
characterized by ST segment elevation in leads
predominantly innervates the SA node,
facing the infarct. See p 563-4, Ch 28, WFG,
decreases the firing rate of the SA node. The
2005.
effect of acetylcholine on pacemaker cells in the
SA node is to activate a hyperpolarizing
Ai

7. The AH interval, from the A wave to the start


potassium current.
of the H spike is normally 60-125 ms.
His Bundle Electrogram:
4. Intrinsic heart rate (IHR) is the rate at which Interval Physiologic Time (ms)
the heart will beat when completely denervated. correlate
This is determined, in humans, by intravenous PA interval Atrial activation 30
administration of a standard dose of atropine and time
AH interval AV nodal delay 60-125
atenolol. In healthy humans, IHR is about 100-
110/min. The magnitude of IHR reflects the HV interval His Bundle - 35-50
automaticity of the SA node. The fact that ventricle

61
The AH interval represents AV nodal Therefore, average SV = 80 ml.
conduction time. See p. 554, Ch 28, WFG, 2005. Average LVEDV = 120 ml.
Therefore, ejection fraction = SV/LVEDV =
8. The AV node, like the SA node, exhibits 0.66 = 66%.
prepotentials but its rate of discharge is much
less compared to the SA node. 16. The Ficks principle states that the amount of
a substance consumed by an organ per unit time
9. See p. 557, Ch 28, WFG, 2005. (A) =
= A-V conc. difference blood flow

in
10, 11, 12. Systolic time intervals (STI) Thus, blood flow (Q) = A / A-V conc. diff
QS2 is the time duration from the beginning of In this example, O2 consumption = 2000 ml/min.
the Q wave to the first high frequency AV O2 difference = 80 ml/L. Cardiac output =
component of the aortic component A2 of the 2000/80 = 25 L/min.
second heart sound. Note that this is an

s.
electromechanical interval; QS2 is called 18. Cardiac output = 5 L/min; Blood Hb = 15
electromechanical systole. It is remarkably g/dL. Oxygen carrying capacity of blood is
constant. approximately 20 ml/dL i.e. 200 ml/L.

Left ventricular ejection time (LVET) is the Whole body oxygen delivery = cardiac output
period from the upstroke of the carotid artery oxygen content of arterial blood.
iim
pulse to the dicrotic notch (an oscillation on the
falling wave of the carotid artery pulse). The
dicrotic notch in the carotid artery pulse tracing
represents the closure of the aortic valve. Note
that ejection is a part of systole.
Therefore about 1L of oxygen will be delivered
to the tissues per minute.

23. At any time, 54% of blood volume is


contained in the veins and venules & vena cava.
Pre-ejection period (PEP) is calculated as QS2
LVET. It denotes the time taken for electrical 26. See Table 30-2, p. 579, Ch 30, WFG, 2005.
4a
excitation of left ventricle, excitation-contraction
coupling and isovolumetric ventricular 30. For a cylindrical structure, transmural
contraction. pressure (P) = T/r (Laplaces law) where T is
wall tension and r is radius
If the duration of ventricular excitation and
excitation-contraction coupling can be assumed Though capillaries are thin walled, they have a
to be constant, then, PEP can be taken to reflect smaller radius and consequently need to develop
the duration of isovolumetric ventricular less tension in order to withstand a given
m

contraction (IVVC). distending (transmural) pressure.

PEP, which reflects the duration of The statement in the question should not mislead
isovolumetric contraction, is prolonged in heart you into believing that capillaries can withstand
failure. This is also associated with a decline in a pressure of 100 mm Hg they will definitely
Ai

left ventricular ejection time. Thus, PEP/LVET rupture at that pressure. Rather if we compared a
ratio is a sensitive index of left ventricular larger vessel and a capillary distended by
systolic performance. STI cannot be used for physiologic pressures (say 40 mmHg), one still
diagnosis of specific heart conditions. Of late, wonders how the thin walled capillary is able to
STI have been superseded by echocardiography. withstand it. That is because it also has a smaller
radius.
13. See Fig 29-2 on p.566, Ch 29, WFG, 2005
33. Bernoullis principle: The greater the
15. Cardiac output = SV HR velocity of flow in a vessel, the lower the lateral
HR = 75/min; cardiac output = 6 L/min. pressure distending its walls.

62
independent variable) and that the converse is
Note coronary arteries originate virtually at a not true.
right angle from the aorta, above the aortic
valve. Thus, in aortic stenosis, the lateral Mareys law is only a restatement of negative
pressure at the level of the origin of coronary feedback regulation of BP by the arterial
arteries is reduced, and this in turn reduces baroreflex; i.e., when BP increases, baroreflex
coronary blood flow. However, this is only one mediated lowering of HR occurs. When BP
of many mechanisms that contribute to the decreases, HR reflexly increases. This is a
reduction in coronary blood flow in aortic mechanism to rapidly regulate and maintain BP

in
stenosis. within a normal range.

34. Tissue blood flow = BP / local vascular There are several exceptions to this:
resistance. Quantitatively, changes in local
vascular resistance achieved mainly through 1. During exercise, BP and HR both increase.

s.
local autoregulatory mechanisms contribute the The goal here is to allow cardiac output to
most to regulating tissue blood flow. increase and deliver more oxygen to actively
metabolizing tissue, and it is logical to raise
39. See p. 599, Ch 31, WFG, 2005 BP rather than maintain it at resting levels.

41. Norepinephrine has a greater affinity for 2. Vasomotor ischemia occurring in the face of
iim
alpha adrenergic receptors than beta adrenergic
receptors. So when it is administered
intravenously to a healthy individual with
normal BP, it elicits a rise in total peripheral
resistance because of alpha adrenergic receptor
hypotension triggers an increase in
sympathetic outflow (CNS ischemic
response) leading to an increase in BP but
this is not accompanied by HR lowering.

mediated vasoconstriction. The rise in BP elicits 3. Sometimes, when MAP is less than 50 mm
a baroreflex mediated lowering of HR that Hg, activation of arterial chemoreceptors
overwhelms the direct cardioacceleratory effect leads to a vagally mediated bradycardia.
4a
of norepinephrine.
46. At rest, blood flow through the systemic
43. Phenylephrine is an alpha adrenergic circulation (i.e., cardiac output) = 5 L/min
agonist, so it raises TPR and BP and elicits a Assuming that functional Hb A concentration is
baroreflex mediated lowering of HR. Carotid 15 g/dL and PaO2 is 100 mm Hg and that Hb is
massage mechanically activates the vagus nerve fully saturated with oxygen,
contained within the carotid sheath increasing Oxygen content of arterial blood = 200 ml/L
cardiac vagal outflow. Whole body oxygen delivery = 1 L/min
m

Whole body oxygen consumption = 250 ml/min


44. Central venous pressure decreases upon Whole body oxygen extraction = 0.25
rising due to a decrease in venous return. A This is sometimes called oxygen utilization
decrease in arterial pressure, which occurs on coefficient (OUC).
rising from the supine position, also decreases Theoretically, oxygen extraction can increase up
Ai

the stretch of arterial baroreceptors (i.e., arterial to 1. Thus, an increase in oxygen extraction is
baroreceptors are said to be unloaded) and the one mechanism of fulfilling an increase in
firing rate in the carotid sinus nerves decreases. oxygen demand.
This reflexly inhibits vagal outflow to the heart However, in trained athletes, cardiac output can
and increases sympathetic outflow from the be increased 7 times from its resting value.
brain resulting in an increase in heart rate and a
rise in total peripheral resistance. 47. See Table 29-3, p. 572, Ch 29, WFG, 2005.

45. Mareys law states that HR (the dependent 48. Vasomotor ischemia stimulates sympathetic
variable) is inversely related to BP (the outflow. This leads to an increase in BP that in

63
turn serves to restore cerebral blood flow. This is Response The BP overshoot triggers a baroreflex
the CNS ischemic pressor response. mediated lowering of HR
Significance BP maintained

Cushings reflex vs. CNS ischemic response


Reflex Cushings reflex CNS ischemic
pressor response 54. During negative g, i.e. acceleration in the
Stimulus Raised intracranial Vasomotor long axis of the body from foot to head, there is
tension causing ischemia due to a headward shift of body fluids and central
vasomotor any cause blood volume, cardiac output and blood pressure
ischemia as a (example,
increase. When intrathoracic pressure is raised

in
result of hypotension)
compression of as occurs during the Valsalva maneuver, venous
brain cells return reduces and cardiac output decreases.
Response Increased Increased Similarly, during positive pressure ventilation,
sympathetic sympathetic the increase in ITP reduces venous return and
outflow outflow
decreases cardiac output. Passive head-up tilt is

s.
Effect Increase in BP. If Increase in BP &
the individual had HR toward normal associated with a decreased cardiac output since
normal BP at venous return is reduced.
baseline, his BP
would become 55. Cardiac atrophy rather than hypertrophy
higher
HR change If BP increases Increases (since
occurs following prolonged space missions since
above normal (say
from 120 mm Hg
to 140 mm Hg),
then a baroreflex
mediated
bradycardia would
occur
iim
when MAP is too
low the arterial
baroreflex
mechanism does
not operate)
the load on the heart is reduced. This is because
skeletal muscles are not regularly used in
microgravity conditions. Postural hypotension
occurs upon return to earth (where the effects of
earths gravity are again fully manifest) because
of autonomic as well as physical deconditioning.
Significance A mechanism to A mechanism to Motion sickness in space occurs due to
eventually eventually conflicting neural inputs from visual cues and
maintain cerebral maintain cerebral
vestibular system and diminished input from
4a
blood flow blood flow
muscle proprioceptors.
49. Note that raised intracranial tension (ICT) is
56. If the arteriovenous concentration difference
not necessarily associated with bradycardia. An
individual with raised ICT may be hypotensive of a substance (example, oxygen) across a
vascular bed is high, it means that the vascular
due to blood loss. In this instance, hypotension
is usually associated with tachycardia. bed extracts a large fraction of this substance.
Oxygen extraction ratio across the heart is 0.5,
m

Rather, if raised ICT leads to hypertension, this high compared to other tissues. Cardiac venous
usually elicits a baroreflex mediated lowering of oxygen tension is low and little additional
oxygen can be extracted from the blood in the
heart rate.
coronaries, so increases in myocardial oxygen
consumption require increases in coronary blood
52. Valsalva maneuver:
flow. If that cannot happen, myocardial ischemia
Ai

Maneuver Forced expiration against a closed


glottis would result with predictable consequences.
Stimulus Decrease in BP due to a decrease in
venous return and consequently stroke At rest, oxygen extraction ratio in the heart
volume (SV)
= (AV O2 difference / arterial O2 content) 100
Response Immediate increase in HR and a more
gradual increase in TPR = (114 / 200) 100 = 0.55
Goal To maintain BP This is based on data in Table 32-1, p. 612,
Immediately Venous return suddenly increases, SV WFG, 2005.
after the increases, TPR is already high. Thus BP
maneuver = SV HR TPR overshoots

64
58, 59. Activity in the noradrenergic nerves to
the heart increases myocardial oxygen demand When there is an auscultatory gap and you use
since it would increase the force as well as rate the palpatory method, you will get a correct
of cardiac contraction. However, this normally estimate of systolic pressure (say 210 mm Hg).
results in coronary vasodilation caused by Sounds will cease at some point (say 160 mm
products of metabolism. The pressor effect of Hg). However, if you do not deflate any further
norepinephrine on alpha-adrenergic receptors in (because sounds have disappeared), you wont
the coronary arteries is not manifest therefore. realize that there is an auscultatory gap and will
However, the direct effect of norepinephrine on end up overestimating diastolic pressure.

in
segments of isolated coronary arteries is Something like 210 / 160 mm Hg will be
vasoconstriction. On the other hand, the increase recorded when the actual pressure is 210 / 90
in myocardial oxygen demand (and mm Hg.
consumption) during exercise is attenuated in _________________________________
individuals taking beta-blockers because the Supplementary Material:

s.
norepinephrine mediated rise in heart rate and
contractility are also diminished, and
Effect of load on muscle contraction:
consequently their ability to exercise is
Within physiologic limits, the energy of cardiac
diminished. The learning point here is that the
contraction is directly proportional to preload.
coronary circulation is capable of excellent
This is the Frank-Starling law of the heart.
autoregulation.

60-63. The triple response:


Response
Wheal production
Mechanism
iim
Increased capillary and
venular permeability
Preload: The load on a muscle before it
contracts. In the ventricle, the preload (end-
diastolic fiber length) varies directly with the
end-diastolic volume.
Red reaction Venodilation Afterload: The load which contracting muscle
Flare Axon reflex has to overcome before it shortens. The velocity
of shortening varies inversely with afterload. For
4a
The white reaction is the response to a benign the left ventricle, the afterload is the total
stimulus. It occurs due to contraction of systemic vascular resistance.
precapillary sphincters.
Cardiac output: The volume of blood ejected
The triple response is evoked by a noxious by the left or the right ventricle per minute.
stimulus.
Cardiac output = stroke volume heart rate
m

With the exception of the flare which is


mediated by an axon reflex, the triple response, Cardiac index is the cardiac output per square
indeed, is an example of autoregulation. meter of body surface area.
However, we might say that autoregulation of Regulation of stroke volume:
tissue blood flow is least prominent in the skin; Stroke volume is influenced by preload,
Ai

this is because, cutaneous blood flow is mainly afterload and myocardial contractility.
regulated by neural signals from the
hypothalamus; this is logical since skin serves as An increase in stroke volume for a given preload
the interface across which heat exchange is and afterload is due to an increase in myocardial
regulated to maintain body temperature. contractility. This is sometimes called
homometric regulation (meaning same length).
64. All four options are correct. When there is In contrast, heterometric regulation refers to
an auscultatory gap and you do not use the changes in stroke volume brought about by
palpatory method, you will underestimate changes in end diastolic volume.
systolic pressure.

65
The best index of myocardial contractility is the Ejection of blood produces an increment in
rate of pressure rise (dP/dt) in the ventricle arterial blood pressure; this is called pulse
during isovolumetric contraction. However, pressure.
clinically, the ejection fraction is used as a
surrogate of myocardial contractility. Pulse pressure depends upon two factors:
Volume of blood ejected (stroke volume) and
Myocardial contractility is markedly enhanced arterial compliance. If arteries are thick and rigid
by sympathetic stimulation, digoxin and (and thus less compliant), a given stroke volume

in
depressed by myocardial ischemia, acidosis. produces a greater rise in pulse pressure.

Inotropic effects: the term refers to the effect of It is often stated that pulse pressure = SBP
a stimulus on myocardial contractility. DBP. However, conceptually, systolic pressure
= diastolic pressure + pulse pressure

s.
What is BP determined by?
BP = cardiac output total peripheral resistance RPP is a useful index because it reflects
myocardial O2 demand. Greater the HR, greater
Blood pressure indexes: the myocardial oxygen demand; to generate a
Systolic pressure (SBP): the highest pressure in greater systolic pressure, greater wall tension
needs to develop (according to Laplaces law)
iim
the arteries during the cardiac cycle.

Diastolic pressure (DBP): the lowest pressure


in the arteries during the cardiac cycle.

Pulse pressure: SBP DBP


and this also increases oxygen consumption.

The arterial baroreflex mechanism for


regulating blood pressure:
Inputs Mean arterial pressure, pulse
pressure
Receptor Called arterial baroreceptors
Mean arterial pressure = diastolic pressure + Location of Adventitia of the carotid sinus and
1/3 (pulse pressure). However, this applies only receptors aortic arch
4a
when HR is in the 60-90 BPM range. MAP is Receptor type Free nerve endings
actually determined by integrating the arterial Adequate stimulus Stretch (both tonic as well as phasic
pressure curve. response)
Afferent Via IX (from the carotid sinuses)
and X nerves (from the aortic arch)
MAP is also equal to (SP + 2DP)/3 Termination of NTS in medulla
first order
Rate-pressure product (RPP): It is calculated neurons
Receptor MAP between 70 and 110 mm Hg
m

as the product of SBP and HR and generally


operating range
divided by 100 to get a smaller number. If SBP Receptor Increase in static (mean arterial
is 120 mm Hg and HR is 80 BPM, then RPP is characteristics pressure) as well as phasic stretch
96 mm Hg BPM 10-2 (pulse pressure) increase firing rate;
decrease in stretch decreases firing
Physiologic correlates of the blood pressure rate. There are presumably two
Ai

subtypes of receptors of which is


indexes: adapting and the other nonadapting.
Diastolic pressure reflects the total resistance Response to an An increase in BP leads to a
offered to peripheral run off of blood increase in BP decrease in cardiac output and TPR
(specifically, the resistance offered by the through cardiac vagal excitation and
inhibition of sympathetic outflow
arterioles).
Response to a Sympathetic outflow from the
decrease in BP medulla is disinhibited; and vagal
When blood is ejected into the arteries, arterial outflow to the heart is disinhibited.
blood pressure increases from about 80 mm Hg Response time: It takes only about 1 second for a
to about 120 mm Hg. change in BP to result in a
compensatory change in HR; it takes

66
a little longer (about 10 seconds) for
changes in TPR to occur following a
primary change in BP

Isotonic versus isometric exercise:


Isotonic (dynamic) Isometric (static) exercise;
exercise; example, running example, sustained
isometric handgrip
BP increases mainly due to BP increases primarily due
an increase in cardiac to an increase in TPR

in
output
TPR may fall in severe Cardiac output may reduce
exercise because of or not change significantly
accumulation of vasodilator from baseline as a result of
metabolites the baroreflex mechanism.

s.
***************************************

iim
4a
m
Ai

67
8. PULMONARY balanced by the natural tendency of chest wall to
recoil outward.
PHYSIOLOGY
Lung capacities in a healthy adult male
Respiratory minute volume and alveolar weighing 70 kg:
ventilation: Capacity Volume (ml)
Functional residual RV + ERV 2300
capacity (FRC)
Respiratory minute volume = VT respiratory Inspiratory capacity (IC) VT + IRV 3700
rate Vital capacity VT + IRV + 4800

in
ERV
Alveolar ventilation = TLC RV + ERV + 6000
(VT anatomical dead space) respiratory rate VT + IRV

Anatomical dead space is roughly equal to body Note that the term expiratory capacity is not
used. Unless otherwise stated, vital capacity is

s.
weight in pounds.
the maximum amount of air that can be expelled
If VT = 500 ml and respiratory rate = 12 per after a forced inspiration. If measured during
minute, then respiratory minute volume = 6 inspiration, it is specifically called inspiratory
liters vital capacity.

iim
If the anatomical dead space = 150 ml, then
alveolar ventilation = (500 150) 12 = 4.2
liters/minute. Note that only alveolar ventilation
could contribute to gas exchange.
Testing mechanics of respiration:
What are the factors affecting vital capacity?
1. Strength of muscles of inspiration
2. Lung compliance
3. Strength of muscles of expiration
Lung volumes and capacities: 4. Airways resistance
Each lung volume is an independent fraction of
total lung capacity (TLC). Lung capacities are Evaluating airways resistance:
4a
combinations of lung volumes. Values given Examine the relationship between air flow,
below are for a healthy young adult male airways resistance and airway pressure before
weighing 80 kg with a body surface area of you consider evaluating airways resistance.
about 1.8 m2. Values in females are about 10%
lower. Note that lung volumes and capacities are Air flow = transairway pressure / airway
greatly influenced by height, weight, body resistance;
surface area.
Transairway pressure = intrathoracic pressure
m

Tidal volume VT 500 ml pressure at the mouth (which is zero mm Hg


Inspiratory reserve volume (IRV) 3200 ml with reference to atmospheric pressure)
Expiratory reserve volume (ERV) 1100 ml
Residual volume (RV) 1200 ml
FEV1, the volume of air expelled during the first
second of a forced vital capacity maneuver,
Tidal volume is about 5 7 ml/kg body weight;
Ai

varies directly with expiratory pressure and


inversely with airways resistance. It is normally
By definition, the reserve volumes are recruited at least 80% of FVC; FEV1/FVC is > 0.8.
with effort.
Note that if vital capacity is reduced, FEV1 will
FRC is the resting lung volume or the volume of be reduced, still FEV1/FVC may be normal.
air present in the lungs at the end of a tidal
expiration. It is the lung volume at which the For example, in restrictive lung disease, FEV1 is
inward elastic recoil of lung parenchyma is reduced because vital capacity is reduced;

68
however, FEV1/FVC is normal because airways have a subnormal FEV1/FVC and yet his vital
resistance is normal. capacity could be normal.

Thus, FEV1/FVC is reduced when airways Mechanism of reduction Disease


resistance is increased. of vital capacity
Weakness of muscles of Myasthenia gravis,
respiration poliomyelitis
Peak expiratory flow rate (PEFR) is the highest Lung compliance reduced Restrictive lung disease
flow rate during forced expiration (vital capacity Increased airways Major airway obstruction;
maneuver). Note that when flows are reduced, resistance bronchial asthma

in
flow rates are reduced. Thus, for example, the
peak flow rate during tidal expiration is much In summary,
lower than that during forced expiration. FEV1 FVC FEV1/FVC PEFR
Normally PEFR is about 12 liters/second or 720 OLD* < 80%
RLD* Normal or >
liters/minute (values are 10-15% lower in age 80%

s.
matched females). *OLD & RLD are obstructive & restrictive lung disease
respectively.
PEFR is dependent upon vital capacity. If vital
capacity is reduced, PEFR is also reduced. Thus, Air trapping occurs whenever there is
it does not make sense to use PEFR as an index expiratory flow limitation. Since flow is driven
of airways resistance in a patient with restrictive by pressure and opposed by resistance, air
lung disease. iim
So, PEFR must be used as an index of airways
resistance only in patients known to have
obstructive airways disease (low FEV1/FVC)
trapping is usually a consequence of an increase
in airways resistance. Air trapping occurs in
chronic obstructive pulmonary disease. As a
result of this, total lung capacity increases,
however the vital capacity is subnormal; in other
because FEV1/FVC is normalized for flow words, patients with obstructive lung disease
whereas PEFR is not. breathe at a higher FRC compared to normal
subjects. The fraction of FRC that increases is
4a
Forced expiratory time: this is a useful bedside the residual volume.
index of airways resistance. Auscultate the
trachea during forced expiration, if expiratory Questions:
sounds are heard for longer than 4 seconds, 1. The volume of air that moves in or out during
airways resistance is increased. each tidal breath is called:

Maximum mid-expiratory flow rate 2. The amount of air present in the lungs at the
(MMEFR): The first 25% of expired air comes end of a tidal breath is:
m

mainly from the major airways; MMEFR is the A. residual volume


highest rate at which the middle 50% of VC is B. expiratory reserve volume
expelled; it is also called forced expiratory flow C. functional residual capacity
rate (FEF 25 75). This reflects the resistance of
the small airways that are narrowed in bronchial 3. The largest lung volume is:
Ai

asthma. A. residual volume


B. inspiratory reserve volume
Using vital capacity as an index of pulmonary C. expiratory residual volume
function: D. tidal volume
Vital capacity is always reduced in lung disease;
however, total lung capacity may be increased or 4. By spirometry, one cannot determine:
decreased depending on the cause. However, A. vital capacity
vital capacity is not a sensitive index of B. functional residual capacity
pulmonary function. For example, a person can C. residual volume
D. total lung capacity

69
12. The largest volume of gas that can be moved
5. Functional residual capacity can be into and out of the lungs in 1 minute by
determined using: voluntary effort is called:
A. spirometer A. respiratory minute volume
B. computerized spirometry B. minute ventilation
C. helium dilution technique C. maximal voluntary ventilation
D. whole body plethysmography D. vital capacity

6. A person breathes into and from a spirometer 13. Maximum voluntary ventilation is about:

in
(volume 12 liters) containing 10% helium gas A. 60-80 L/min
mixture. After equilibration, helium B. 125-170 L/min
concentration of expired gas was found to be C. 200-220 L/min
6.7%. His vital capacity is 4.2 liters. What is his D. 90-100 L/min
residual volume?

s.
A. 1000 ml 14. Which is the major muscle of inspiration?
B. 1200 ml
C. 1500 ml 15. Which muscle does not contract during
D. 1800 ml forced expiration?
A. Internal intercostals
7. In a healthy individual with a total lung B. External intercostals

about:
A. 400 ml
B. 1000 ml
iim
volume of 6 liters, the amount of oxygen present
in the lungs at the end of a normal expiration is
C. Transversus abdominus
D. Rectus abdominis

16. The normal compliance of the human lungs


and chest wall is:
C. 100 ml A. 0.1 L/cm H2O
D. 250 ml B. 0.2 L/cm H2O
C. 0.3 L/cm H2O
4a
8. The maximum amount of gas that can be D. 0.4 L/cm H2O
exhaled after a full inspiration is called:
A. expiratory reserve volume 17. Resting lung volume would be lesser than
B. vital capacity FRC if it were not for the presence of:
C. total lung capacity A. surface tension forces
D. functional residual capacity B. surfactant
C. elastic recoil forces
9. Vital capacity correlates inversely with body
m

surface area. True / false? 18. In which pathologic condition is lung


compliance increased?
10. Vital capacity is greater in the upright
posture compared to supine posture. True / 19. Normally, during quiet breathing,
false? intrapleural pressure is negative during
Ai

inspiration as well as expiration. True/False?


11. Residual volume is normally about ____%
of total lung capacity. 20. The major constituent of surfactant is:
A. 10 A. neutral lipid
B. 20 B. dipalmitoyl phosphatidyl choline
C. 30 C. phosphatidylglycerol
D. 40 D. surfactant Protein A & D

21. Intrapleural pressure is positive during:


A. deep inspiration

70
B. tidal expiration 29. The most sensitive index of small airways
C. forced expiration resistance in a patient with bronchial asthma is:
D. tidal inspiration A. vital capacity
B. FEV1
22. Which of the following produce C. FEV1/FVC
bronchodilation? D. PEFR
A. VIP E. MMEFR
B. Epinephrine
C. Leukotriene C4 30. In chronic obstructive lung disease:

in
D. PAF A. FRC increases
B. TLC increases
23. Bronchodilation is mediated by: C. VC decreases
A. 1 receptors D. RV increases
B. 2 receptors

s.
C. M2 receptors 31. Which of the following statements is
D. 2 adrenoceptors correct?
A. Normally, ventilation is greater at the apex
24. Bronchial tone is highest at about: than at the lung bases
A. 6 AM B. In the upright position, lower lung zones are
B. 10 AM more compliant compared to upper lung
C. 2 PM
D. 6 PM
iim
25. Which of the following produces
bronchoconstriction?
zones
C. In the upright position, blood flow is greater
at the lung bases than at the apices
D. Positive intrapleural pressures during forced
expiration prevent airways from closing.
A. VIP
B. PAF 32. Anatomic dead space is determined using:
C. LTB4 A. whole body plethysmography
4a
D. Epinephrine B. spirometry
E. Nitric oxide C. single-breath nitrogen curve
D. single-breath carbon monoxide method
26. Which of the following is least likely to
produce bronchoconstriction? 33. The pattern of ventilation in lungs is
A. NO assessed by:
B. Sulfur dioxide A. Kety method
C. Cool air B. inhalation of radioactive xenon
m

D. Exercise C. angiography
D. single breath technique
27. The FEV1/FVC ratio is normally greater
than: 34. Under basal conditions, the amount of
A. 0.8 oxygen consumed per minute is about:
Ai

B. 0.7 A. 100 ml
C. 0.9 B. 250 ml
D. 0.5 C. 350 ml
D. 500 ml
28. Peak expiratory flow rate (PEFR), an index
of airways resistance, is reduced in: 35. Under basal conditions, the respiratory
A. restrictive lung disease quotient (i.e., the volume of CO2 produced /
B. bronchial asthma volume of oxygen consumed) is about:
A. 1
B. 0.8

71
C. 0.7 C. lung bases
D. 1.2
43. In the upright position, the V/Q ratio at the
36. Under basal conditions, respiratory exchange lung apex is about:
ratio is normally: A. 1
A. 0.7 B. 0.7
B. 0.8 C. 0.5
C. 0.9 D. 3
D. 1.0

in
Gas transport between lungs and the
37. Under basal conditions, in a person on a tissues:
mixed diet, respiratory quotient is about: 44. Arterial blood contains more oxygen than
A. 0.7 carbon dioxide. True / false?
B. 0.8

s.
C. 0.9 45. Venous blood is normally 40% saturated
D. 1.0 with oxygen. True / false?

38. Alveolar oxygen tension (PAO2) is 46. The sigmoid shape of the oxygen-
influenced by: hemoglobin dissociation curve is due to:
A. barometric pressure

D. respiratory exchange ratio


iim
B. fraction of oxygen in inspired air (FiO2)
C. PACO2

39. At rest, the normal diffusing capacity of the


A. allosteric effects
B. steric effects of oxygen binding on the
configuration of Hb
C. the effects of 2,3BPG on oxygen affinity of
Hb

lungs for carbon monoxide is about: 47. The major buffer of carbondioxide in blood
A. 25 ml/min/mm Hg is:
B. 50 ml/min/mm Hg A. bicarbonate
4a
C. 75 ml/min/mm Hg B. albumin
D. 100 ml/min/mm Hg C. hemoglobin

40. The mean systemic arterial pressure and the 48. If blood [Hb] is 15 g/dL, how much oxygen
mean pulmonary artery pressure are respectively would be transported in blood?
90 and 15 mm Hg. Can you tell the ratio of A. Approximately 20 ml/dL
systemic and pulmonary vascular resistances? B. Approximately 15 ml/dL
A. 1
m

C. Data inadequate
B. 10
C. 6 49. If Hb were completely absent, how much
D. Data inadequate plasma would be required to transport oxygen
required for basal metabolism?
41. Ventilation-perfusion ratio (whole lung) is A. 10 liters
Ai

normally: B. 27 liters
A. 1 C. 52 liters
B. 0.8 D. 84 liters
C. 0.6
D. 1.2 50. Which of the following shifts the
oxyhemoglobin dissociation curve to the right?
42. In the upright position, V/Q ratio is highest A. CO2
in the: B. 2, 3-BPG
A. lung apices C. Increase in tissue temperature
B. middle zone D. Decrease in tissue pH

72
Regulation of respiration:
51. Suppose you administer 100% oxygen at 4 58. The major stimulus for spontaneous
atmospheres, then how much oxygen would be respiration is:
transported in 100 ml of plasma? A. CO2
A. 0.3 ml B. O2
B. 3 ml C. lactate
C. 6 ml D. HCO3
D. 9 ml
59. Central chemoreceptors are chiefly

in
52. The decrease in oxygen affinity of Hb when stimulated by:
the pH of blood falls is: A. an increase in CSF [H+]
A. Bohr effect B. a fall in PaO2
B. Haldane effect C. a decrease in PaCO2
C. Hawthorne effect D. all of the above

s.
D. Hamburger effect
60. The neurons pacing spontaneous breathing
53. Which of the following shifts the are located in:
oxyhemoglobin dissociation curve to the left? A. pneumotaxic center
A. CO B. apneustic center
B. Increase in [2,3 BPG] in RBC C. pre-Bottzinger complex

D. Decrease in tissue pH
iim
C. Increase in tissue temperature

54. The commonest cause of arterial hypoxemia


is:
D. dorsal motor nucleus of vagus

61. Systemic arterial chemoreceptors


(commonly called peripheral chemoreceptors)
are mainly stimulated by:
A. Type II respiratory failure A. a decline in PaO2
B. lactic acidosis B. an increase in [H+] of arterial plasma
C. V/Q mismatch C. a decrease in blood flow through them
4a
D. Anemia D. an increase in PaCO2

55. PaO2 is / may be normal in hypoxia due to: 62. The plasma/CSF ratio of proteins is:
A. hypoventilation A. 3
B. diffusion limitation B. 10
C. severe circulatory shock C. 20
D. anemia D. 300
E. cyanide poisoning
m

63. Forced hyperventilation to exhaustion may


56. Administration of O2 rich gas mixtures result in:
improves tissue oxygenation most in: A. a decrease in serum ionized calcium
A. hypoxic hypoxia B. an increase in plasma protein level
B. anemic hypoxia C. an increase in blood pH
Ai

C. hypokinetic hypoxia
D. histotoxic hypoxia 64. Hypocapnia is usually due to:
A. hypoxemia
57. CO2 is chiefly transported in blood: B. alkalosis
A. as bicarbonate C. decreased CO2 production
B. bound to hemoglobin D. alveolar hyperventilation
C. as dissolved CO2
D. in combination with plasma proteins 65. Normally, PaCO2 chiefly depends on:
A. total lung capacity
B. PaO2

73
C. alveolar ventilation C. 28%
D. rate of CO2 production D. 35%

66. Caution needs to be exercised in 73. Regarding physiologic changes during


administering O2 to patients with: moderate isotonic exercise in a healthy young
A. ARDS adult, which of the following are true and which
B. metabolic acidosis are false?
C. type I respiratory failure 1. PaO2 declines
D. type II respiratory failure 2. PaCO2 increases

in
3. pH of arterial blood decreases (eventually
67. Which of the following data suggests type II during intense exercise)
respiratory failure? (pH, PaCO2 and plasma 4. A-V O2 difference increases
[HCO3] given below in that order). 5. SvO2 increases
A. 7.4, 60 mm Hg, 30 mmol/L 6. Plasma [K+] increases

s.
B. 7.2, 80 mm Hg, 30 mmol/L 7. The impulse rate from carotid bodies
C. 7.33, 46 mm Hg, 20 mmol/L increases.
8. During maximal exercise, O2 consumption
68. J receptors are located in: may reach up to 4 L /min, about 16 times
A. pulmonary interstitium basal oxygen consumption.
B. alveoli 9. VO2 max (maximum oxygen consumption
C. bronchial mucosa
D. roof of the fourth ventricle
iim
69. Stimulation of J receptors results in:
A. apnea
during intense exercise) is increased by
training.
10. Heart rate may remain elevated for as long
as 1 hour after severe exercise.

B. tachypnea ***************************************
C. hyperpnea Common misconception:
D. apnea followed by tachypnea
4a
Are chemoreceptors stimulated in anemia or
70. An increase in the duration of expiration not?
produced by lung inflation is due to: For the sake of clarity, let us ask a question like
A. Hering-Breuer deflation reflex this:
B. pulmonary chemoreflex
C. Bezold-Jarisch reflex Chemoreceptors are not stimulated in:
D. Hering-Breuer inflation reflex A. anemia
m

B. anemic hypoxia
71. Which of the following statements regarding
Cheyne-Stokes respiration are correct? Note the difference between anemia, which
A. It is characterized by periods of hyperpnea simply means that the concentration of
punctuated by apnea. hemoglobin is lower than normal, and anemic
B. It is attributed to increased sensitivity of the hypoxia - a consequence of anemia associated
Ai

respiratory center to CO2. with anerobic metabolism and acidosis.


C. It may occur if lung-to-brain circulation time
is prolonged. The effect of acidosis would be to stimulate the
D. It occurs in severe congestive heart failure peripheral chemoreceptors (i.e., those in the
carotid bodies) in anemic hypoxia. In an anemic
72. During mouth-to-mouth resuscitation, what individual who is at rest, and whose arterial pH
is the oxygen concentration of the resuscitating is normal, peripheral chemoreceptors may not be
gas mixture? as stimulated.
A. 16%
B. 21%

74
The additional increase in ventilation (during 41B 42A 43D 44F 45F
exercise) produced by the acidosis is dependent 46B 47C 48C 49D 50all
upon the carotid bodies and does not occur if 51D 52A 53A 54C 55DE
they are removed. See p 682, Ch 37, WFG, 56A 57A 58A 59A 60C
2005. 61ABC 62D 63AC 64D 65C
66D 67B 68A 69D 70D
Now take this question: 71all 72A
Systemic arterial chemoreceptors are stimulated 1 Tidal volume; 14 Diaphragm; 18 Emphysema
in (check all correct answers):

in
A. anemic hypoxia 73. Physiologic changes during exercise:
B. hypoxic hypoxia 1F 2F 3T 4T 5F
C. hypokinetic hypoxia 6T 7T 8T 9T 10T
D. histotoxic hypoxia
Answer Explanations:

s.
All are correct. The question is whether
5. FRC (the volume of air remaining in the lungs
peripheral chemoreceptors are stimulated or not
at the end of a tidal breath) cannot be determined
in various types of hypoxia. It is not how they
by spirometry.
are stimulated. There are several factors besides
a low PaO2 that could stimulate peripheral
FRC is determined by:
chemoreceptors. These are:
iim
A decrease in blood flow through the
chemoreceptors as occurs in circulatory shock
(stagnant hypoxia) can stimulate them.
1. helium dilution technique
2. whole body plethysmography

6. Principle: Indicator-dilution principle


Indicator: Helium. This is used because it has a
very low molar mass (low density), that it does
Inhibition of oxygen utilization in
chemoreceptor cells (by injection of cyanide) not dissolve in blood when it is inhaled at 1
also activates chemoreceptors. atmosphere. Initially, the helium gas is present
only in the spirometer. The person breathes from
4a
Whenever hypoxia is accompanied by acidosis as well as into the spirometer. The circuit is
(for example in an anemic individual doing closed. After a few breaths, the helium
exercise), acidosis activates the receptors and concentration in the spirometer will reduce
increases minute ventilation. Injection of K+ has because it would have also got diluted in lung
been shown to activate them; this may partly volume.
account for exercise induced changes in
ventilation. Initially,
m

C1: Helium concentration in spirometer = 10%


Thus, in anemic hypoxia, arterial V1: Spirometer volume = 12 liters
chemoreceptors are not stimulated by hypoxia, C2: Helium concentration in spirometer (=
but rather by the ensuing acidosis. lungs) = 6.7%
V2: Volume of distribution is now total lung
capacity + spirometer volume.
Ai

Answers to Pulmonary Physiology:


Very little helium actually dissolves in blood. So
2C 3B 4BCD 5CD
it can be neglected.
6D 7A 8B 9F 10T
As per law of conservation of mass,
11B 12C 13B 15B C1V1 = C2 (V1+V2)
16B 17B 19T 20B Substituting, we get,
21C 22AB 23D 24A 25BC 10 12 = 6.7 (12 + TLC)
26A 27A 28AB 29E 30all TLC = [10 12 / 6.7] 12 = 18 12 = 6L
31BC 32C 33B 34B 35B VC = 4.2 L
36B 37B 38all 39A 40C So RV = 6 4.2 = 1.8 L = 1800 ml.

75
35-37. At rest, 250 ml of oxygen is consumed
7. FRC = 2300 ml per minute and 200 ml of CO2 is produced.
Because alveolar oxygen is diluted with CO2,
the fraction of oxygen in alveolar gas is only Respiratory exchange ratio (RER) is the ratio of
about 16%. the volume of CO2 to O2 exchanged across the
Thus, the amount of oxygen present in lungs at lungs. It is normally 200 / 250 = 0.8
the end of a tidal expiration = (16 / 100) 2300
= 370 ml Respiratory exchange ratio reflects the average
respiratory quotient (RQ) when gas exchange

in
10. Intrathoracic blood volume increases by across lungs is normal.
about 400 ml in the supine position compared to
standing; the increase in pulmonary blood RQ is different for different energy substrates.
volume is greater in patients with left ventricular Carbohydrate 1
systolic failure and consequently they may be Protein 0.8

s.
breathless when lying supine. Fat 0.7
Average RQ 0.8
16. Lung compliance is the change in lung
volume for a given change in pressure. The average RQ depends upon the metabolic
Compliance = V / P state. Suppose RQ is 1, then you may infer that
The normal compliance of human lungs and
iim
chest wall is about 0.2 L/cm H2O. Compliance is
reduced in restrictive lung disease.

22. VIP is released by nonadrenergic


noncholinergic neurons. VIP relaxes bronchial
only carbohydrate is being used.

38. The alveolar gas equation:


Alveolar oxygen tension PAO2
= [(PB-PH20) FiO2] [(PAC02)/R]

smooth muscle. VIP deficiency has been PB is barometric pressure (760 mm Hg at mean
implicated in asthma. sea level);
PH20 is the pressure of water vapor at body
4a
Leukotrienes are potent bronchoconstrictors. temperature (it is 47 mm Hg);
FiO2 is the percentage of oxygen in inspired air;
Zileuton, a lipoxygenase inhibitor inhibits the and PACO2 is the CO2 tension in alveolar gas
synthesis of leukotrienes. It is used in the (normally about 40 mm Hg). It is equal to
management of asthma. PaCO2 because CO2 is highly soluble and
readily equilibrates across the alveolocapillary
Zafirlukast is a leukotriene receptor blocker. membrane. So you can substitute PaCO2 for
m

PACO2.
25. Inhaled nitric oxide has been shown to
produce bronchodilation as well as a decrease in R is respiratory exchange ratio (as defined
pulmonary artery pressure. Substance P, platelet above, see answer explanation for Questions 35-
activating factor (PAF) and leukotrienes are 37); it is assumed to be 0.8 when the alveolar
bronchoconstrictors. gas equation is used for solving common clinical
Ai

problems.
32. What is a shunt; what is dead space?
Shunt: Perfusion of unventilated alveoli results Substituting, we get,
in shunting of deoxygenated blood across the PAO2 = [(760 47) 0.21] [40/0.8]
lungs to the heart, i.e. a right-to-left shunt. = 150 50 = 100 mm Hg
Dead space: Ventilation of unperfused alveoli is
effectively an extension of anatomic dead space 39. When alveolar ventilation is held constant,
since it cannot contribute to gas exchange. the amount of CO2 removed is limited by
pulmonary blood flow. However, CO2

76
accumulation is invariably due to a decrease in For example, at rest, alveolar ventilation = 12
alveolar ventilation. (500 150) = 4.2 L/min;
Assuming VT = 500 ml, VD = 150 ml;
When alveolar oxygen tension is constant, At rest, pulmonary blood flow = 5 L/min;
normally (i.e., at rest), oxygen exchange is Thus, whole lung V/Q ratio = 0.84;
limited by perfusion because enough time is In the lung apices, V/Q approaches infinity;
available for equilibration across the In the lung bases, V/Q is lower than 1.
alveolocapillary membrane. However, during
exercise, when blood flow rates are higher, the It must however be noted that alveolar

in
time available for equilibration is lesser and ventilation is much higher at the lung bases than
oxygen exchange is increasingly limited by the apices because the lower lung zones are less
diffusion (diffusion-limited). Thus, impairment distended at the start of inspiration and therefore
in diffusion capacity of the lungs for oxygen more compliant. As a result of regional
causes problems especially during exercise. differences in intrapleural pressure, upper lung

s.
zones are already in a more expanded position at
Testing diffusion capacity of the lungs: the start of inspiration.
The single breath technique: The subject
inhales a mixture containing 0.01% CO Lung zones:
We know that diffusion rate J = (PAPc) PA = Alveolar pressure;
DA/x Pa = Pressure at the arterial end of the

DA/x
DLCO = 25 ml/min/mm Hg
DLO2 = 25 ml/min/mm Hg
iim
In a nonsmoker, Pc CO = 0 mm Hg and PA is a
constant; thus the diffusion rate depends upon
pulmonary capillary;
Pv = Pulmonary venous pressure;
Pi = Pulmonary interstitial pressure.

Zone Definition
Since CO exchange is diffusion limited, it is Zone 1 PA > Pa > Pv
used for assessing diffusion capacity. Zone 2 Pa > PA > Pv
Zone 3 Pa > Pv > PA
4a
40. Since pulmonary and systemic circulations Zone 4 Pa > Pi > PA
are in series, the flows are identical
Cardiac output = P / R; Zone 1: above the heart; arterial pressure is
(P is mean arterial pressure and R is vascular lower and may be lower than alveolar pressure if
resistance) the alveoli are well expanded. Flow may be
Ps / Pp = Rs/Rp (s and p refer to systemic and minimal. V/Q approaches infinity.
pulmonary vascular beds respectively)
Ps / Pp = 90 / 15 = 6
m

Zone 2: pulmonary arterial pressure is greater


Ratio of systemic and pulmonary vascular than alveolar pressure and flow is determined by
resistance = 6 arterial alveolar pressure difference; however,
Thus, the pulmonary circulation is a low during inspiration when alveolar pressure
resistance, low pressure system. becomes more negative, blood flows from the
arteries into the pulmonary veins. This is called
Ai

At rest, systolic pulmonary artery pressure is the waterfall effect.


about 25 mm Hg, diastolic pulmonary artery
pressure averages 10 mm Hg, mean pulmonary Zone 3: flow is continuous and is driven by
artery pressure is about 15 mm Hg, and pulmonary arterial venous pressure gradient.
pulmonary capillary pressure averages about 10
mm Hg. Zone 4: it occurs in pulmonary edema (always
abnormal). When pulmonary interstitial pressure
41-43. Ventilationperfusion ratio is the ratio is > than alveolar pressure, alveolar collapse
of alveolar ventilation to pulmonary blood flow. (atelectasis) would result.

77
Amount of plasma that would be required to
When a Swan Ganz catheter is wedged in the transport 250 ml of oxygen = 83 liters!
pulmonary capillary to estimate left ventricular This problem illustrates the importance of Hb.
end diastolic pressure (LVEDP) in a patient who
is on a ventilator with positive end expiratory 51. O2 solubility in plasma is limited, hence Hb
pressure (PEEP) added, the measured pressure required for oxygen transport.
would reflect alveolar pressure rather than O2 solubility in plasma = 0.003 ml O2/100 ml
LVEDP; thus, the catheter must be placed in plasma/ mm Hg PO2.
Zone 3 conditions (perhaps by momentarily Greater the PaO2, greater the dissolved oxygen

in
discontinuing PEEP) to reliably estimate (Henrys Law)
LVEDP Calculate the expected PaO2 using the alveolar
gas equation:
Regulation of pulmonary vascular resistance: PAO2 = [(4 760) 47] 1 (40/0.8)
Expected PaO2 is approximately 3000 mm Hg.

s.
Block ventilation to a lung zone Thus, about 9 ml of oxygen will be transported
in 100 ml of plasma.
Perfusion of that lung zone decreases That is, in 4 liters of plasma 360 ml of oxygen
can be transported. Let us say the requirement is
Hypoxic pulmonary vasoconstriction serves to 250 ml O2/min. This meets the demand; this is
shunt blood to well ventilated lung zones and the rationale for use of hyperbaric oxygenation.
iim
optimize oxygen uptake in the lungs. However,
chronic alveolar hypoxia leads to a chronic
increase in pulmonary vascular resistance and
pulmonary hypertension.
However, it is resorted to only when the oxygen
transport capacity of blood cannot be raised
otherwise because oxygen toxicity is also a
significant concern.

44. It contains more CO2 since CO2 is much 52. The presence of higher levels of CO2 and H+
more soluble than O2 in blood. in the capillaries of metabolically active tissue
promotes the release of O2 from hemoglobin.
4a
45. It is 75% saturated with oxygen This is the Bohr effect.

47. HCO3 cannot buffer protons formed from 53. CO is a competitive inhibitor of oxygen
CO2; binding of hemoglobin. Exposure for 1 hour to a
CO2 + H2O = H+ + HCO3 CO concentration of 0.1% in inspired air leads to
H + HCO3 = CO2 (acid); the occupancy by CO of about 50% of heme
Hb is the major buffer of CO2 sites in Hb, a proportion that is frequently fatal.
Apart from being a competitive inhibitor of
m

48. Factors that affect oxygen delivery to the oxygen binding to Hb, CO also inhibits the
tissues include the following (the oxygen dissociation of oxyhemoglobin (prevents
cascade): unloading of oxygen at the tissues). That is, it
Alveolar ventilation shifts the ODC to the left.
Pulmonary blood flow
Ai

Ventilation / perfusion balance 57. Ventilation perfusion imbalance is the


Diffusion capacity of lungs for oxygen commonest cause of hypoxemia. Of course, the
Cardiac output hypoxemia in ARDS is also due to ventilation-
Hb concentration of arterial blood perfusion match; V/Q mismatch is an umbrella
Affinity of Hb for oxygen term that illuminates pathophysiology and is the
Blood flow to each tissue single best answer for this question.

49. Basal oxygen requirement at rest = 250 58. Note the difference between the terms
ml/min hypoxemia (decreased arterial PO2) and hypoxia
O2 carrying capacity of 1L of plasma = 3 ml (inadequate tissue oxygenation). While the

78
PaO2 may be normal in carbon monoxide 77. In chronically hypercapnic patients, CO2
poisoning, circulatory shock, cyanide poisoning depresses rather than stimulates respiration and
and anemia, the tissues are yet hypoxic. Carbon respiration is driven by hypoxia. Administration
monoxide inhibits the dissociation of O2 from of 100% oxygen may remove the hypoxic drive
HbO2. and breathing may stop. Such a patient will
require mechanical ventilation to bring down
64. CO2 in plasma quickly equilibrates across CO2 a level which would stimulate respiration.
the blood brain barrier. There it is quickly
hydrated to form H2CO3. Carbonic acid is 81. The Hering Breuer inflation reflex is an

in
unstable and dissociates at physiologic pH to increase in the duration of duration of expiration
form H and HCO3. The protein content of CSF produced by steady lung inflation and the Hering
is extremely low that it cannot buffer H ions Breuer deflation reflex is a decrease in the
formed from carbon dioxide. Thus H ions in duration of expiration caused by marked
CSF stimulate central chemoreceptor neurons deflation of the lung.

s.
which in turn drive the respiratory neurons.
Central chemoreceptors are chiefly stimulated Nonchemical influences on respiration
by an increase in PaCO2 i.e. an increase in [H+] Reflex Stimulus Response
of CSF. Hering Breuer Lung inflation Increase in the
inflation duration of
67. Increase in plasma [K] as may occur during reflex* expiration
exercise, cyanide. iim
68. In arterial plasma, CO2 gets buffered by Hb
and plasma proteins. Therefore, it is less
effective stimulating receptors in carotid and
Hering Breuer
deflation
reflex*
Lung
deflation
Increase in the
duration of
inspiration

aortic bodies. This would not happen in the CSF Heads Lung inflation Further
because there is very little protein there. Note paradoxical inflation
that the ratio of the concentration of protein in reflex
4a
plasma and CSF is normally about 300. *Hering Breuer reflexes are probably initiated by
considerable changes in lung volume (deep inspiration or
70. Plasma protein concentration = 6000 mg/dL deep expiration).

It may play a role in lung expansion in newborns.
CSF protein concentration = 20 mg/dL
82. Periodic breathing may occur in:
74. If minute ventilation reduces, then PaCO2
Severe congestive heart failure
increases (hypercapnia). Hypercapnia is a
Uremia
PaCO2 > 45 mm Hg. Hypercapnia is almost
m

Sleep
always due to inadequate alveolar ventilation.
Brain stem disease
Conversely, hypocapnia (PaCO2 < 35 mm Hg) is
A normal individual following a bout of
almost always due to hyperventilation, i.e.
voluntary hyperventilation.
minute ventilation in excess of that required to
maintain arterial plasma pH at 7.4
85. PaCO2 changes little because alveolar
Ai

ventilation also increases.


76. Hyperventilation prior to breath holding
increases breath holding time since CO2, the
The impulse rate from the carotid bodies
major stimulus for respiration, is washed out by
increases possibly because of an increase in
overbreathing. The braking point is the point at
body temperature, plasma [K] and [H] formed
which voluntary control of respiration (either
from lactic acid.
breath holding or voluntary hyperventilation) is
overridden by chemical control.
Plasma K increases because K is released from
actively metabolizing skeletal muscle.

79
Normal PO2 and PCO2 levels in blood:
pH of arterial blood is initially maintained but PO2 Arterial 100 mm Hg
eventually decreases and that is when one PO2 Venous 40 mm Hg
fatigues. A substantial increase in hydrogen ion PCO2 Arterial 40 mm Hg
concentration of arterial blood will accelerate PCO2 Venous 46 mm Hg
the onset of fatigue.
Gas content of blood (in ml).
Arteriovenous O2 difference increases because Arterial Venous
oxygen extraction by tissues increases. Dissolved Combined Dissolved Combined

in
O2 0.3 19.5 0.12 15
Saturation of mixed venous blood with oxygen CO2 2.5 45 3 50
decreases because oxygen extraction by
metabolically active tissue increases.
Oxygen transport capacity of blood:
Assuming all Hb is Hb A, PaO2 = 100 mm Hg,

s.
During maximal exercise, O2 consumption may
and that Hb is fully saturated with O2, then:
increase to as high as 3-4 liters/min, i.e. 12-16
Content of oxygen in arterial blood (CaO2) =
times the oxygen consumption at rest.
19.8 ml/dL of blood; of this, about 0.3 ml is
dissolved in plasma and the rest is in
Heart rate may remain elevated for as long as 1
combination with Hb A.
hour after severe exercise because the increase
iim
in sympathetic nerve activity also persists. This
is due to the oxygen debt incurred during
exercise. Until the debt is cleared, minute
ventilation and cardiac output will be higher
compared to preexercise levels.
Hypoxia: The term is often used to denote
inadequate oxygenation of tissues (tissue
hypoxia).

Hypoxemia: a reduction in oxygen tension of


*************************************** arterial blood below 80 mm Hg
Supplement: Gas Transport Classification of hypoxemia (based on
4a
severity):
Oxygen loading in the lungs: Grade PaO2
the reaction of Hb with oxygen Mild 6180 mm Hg
The T (tense) configuration reduces the affinity Moderate 4160 mm Hg
of Hb for O2; the R (relaxed) configuration Severe (fulminant) < 40 mm Hg
increases the affinity for O2
Hb4 + O2 = Hb4O2 Classification of causes of tissue hypoxia:
Type PaO2 Cause Treatment
m

Hb4O2 + O2 = Hb4O4
Hypoxic Reduced High Administer
Hb4O4 + O2 = Hb4O6 hypoxia altitude; oxygen;
Hb4O6 + O2 = Hb4O8 defects in address cause
gas
These are some facts that you must remember: exchange
The solubility of oxygen in plasma at body Anemic Normal Anemia; Raise oxygen
Ai

hypoxia CO carrying
temperature is 0.003 ml / mm Hg PO2 / 100 ml poisoning* capacity of
of blood; plasma /
Assuming PO2 is 100 mm Hg, the amount of blood as
oxygen dissolved in blood / plasma 0.3 ml / dL appropriate;
address cause
However, CO2 is about 20 times more soluble in Hypokinetic Normal or Myocardia Address
blood compared to O2 and there is much more hypoxia, alt. reduced l hypoxia cause;
CO2 in arterial blood compared to oxygen (see stagnant due to
the tables below). hypoxia, coronary
ischemic ischemia;
hypoxia circulatory

80
shock Functions of hemoglobin: Apart from
Histotoxic Normal Cyanide Address transporting oxygen, CO2 and buffering protons,
hypoxia poisoning cause;
hyperbaric
Hb also binds nitric oxide (a vasodilator); this
oxygenation may help match oxygen delivery with increases
in tissue blood flow.
Myoglobin (Mb): an oxygen storage protein;
also a heme protein; single subunit; so there is ***************************************
no quaternary structure; so it does not bind 2,3
BPG; it is abundant in red muscle; 1 mol of Mb

in
binds one mol of O2; its P50 is about 1 mm Hg
(much lesser compared to HbA); Neuroglobin
may have a similar function in brain.

Transport of CO2

s.
1. CO2 from tissues diffuses into RBC
2. It is hydrated by the enzyme carbonic
anhydrase (CA) present in RBC:
CO2 + H2O = H2CO3
The hydration of CO2 in plasma is much

iim
slower because of the absence of CA in
plasma.
3. H2CO3 dissociates into H+ and HCO3
4. H+ is buffered by Hb; HCO3 enters plasma
in exchange for chloride which enters RBC;
this is called chloride shift or Hamburger
phenomenon (it is mediated by the Cl-
HCO3 exchanger)
4a
What happens in the lungs?
The high PO2 in alveolar gas causes Hb to give
up CO2 and take up oxygen.

Haldane effect: The high concentration of O2 in


the alveolar capillaries unloads H+ and CO2 from
hemoglobin; i.e., oxygen dissociation curve of
m

Hb shifts to the left.

Transport of CO2 in 100 ml arterial blood:


Arterial Venous
Dissolved 2.5 ml 3 ml
Bicarbonate 45 ml 47 ml
Ai

Carbamino 2.5 ml 3.5 ml


compounds

How much acid do the lungs excrete per day?


200 ml of CO2 per minute amounts to 288 liters
of CO2 per day which is equivalent to 12500
meq of H+ per day. The kidneys normally
excrete about 50 mEq of H ions per day.

81
9. RENAL PHYSIOLOGY C. 0.3
D. 0.4
Abbreviations used here:
ADH antidiuretic hormone 6. Glomerular efferent arteriolar constriction:
ANG II angiotensin II A. increases GFR
CCD cortical collecting duct B. raises filtration fraction
DCT distal convoluted tubule
GFR glomerular filtration rate C. decreases renal plasma flow
MCD medullary collecting duct
MI medullary interstitium 7. In humans, what percentage of nephrons have

in
PCT proximal convoluted tubule long loops of Henle?
RBF renal blood flow
RPF renal plasma flow
A. 5
TAL thick ascending limb B. 15
TDL thin descending limb C. 25
D. 85

s.
Please examine all answer choices and check all
correct answers. 8. Juxtamedullary nephrons constitute about
______ % of the total nephrons
Functional anatomy, GFR, RPF, A. 10
clearance: B. 15
C. 25

B. Creatinine
C. Urea
D. Mannitol
iim
1. The clearance of which of the following
provides the most accurate estimate of GFR?
A. Inulin
D. 40

9. The macula densa is located in the:


A. proximal convoluted tubule
B. mesangium
C. efferent arteriole
2. In clinical practice, the clearance of which D. beginning of DCT
substance is most frequently used to estimate
4a
GFR? Tubular function:
A. Inulin 10. The brush border is most extensive in cells
B. Creatinine of the:
C. Urea A. PCT
D. Mannitol B. loop of Henle
C. DCT
3. Which of the following substances can not D. collecting duct
m

been used to measure GFR?


A. Inulin 11. Which of the following increases renal
B. Creatinine cortical blood flow?
C. Phenol red A. PGE2
D. Mannitol B. Norepinephrine
C. Angiotensin II
Ai

4. Filtration fraction is equal to: D. Vasopressin


A. RPF / GFR
B. GFR / RPF 12. The PAH clearance of a patient with
C. RPF times GFR elevated levels of blood urea, creatinine and
potassium was found to be 330 ml/min. His
5. What fraction of renal plasma flow is PCV is 25. What is the approximate renal blood
normally filtered in the glomerular capillaries? flow?
A. 0.1 A. 489 ml/min
B. 0.2 B. 673 ml/min

82
C. 742 ml/min 15. Angiotensin converting enzyme is a
D. 1273 ml/min dipeptidyl carboxypeptidase.

Regulation of GFR: 16. Which of the following stimulates the


13. Autoregulation of GFR is achieved through release of aldosterone from the adrenal cortex?
changes in: A. Renin
A. renal perfusion pressure B. Angiotensin III
B. renal blood flow C. plasma [K+]
C. glomerular afferent arteriolar resistance D. ANP

in
D. glomerular efferent arteriolar resistance
Down the nephron:
14. In healthy humans, renal autoregulation of 17. In the nephron, glucose reabsorption occurs
GFR is overwhelmed when mean arterial mainly in the:
pressure falls below: A. proximal convoluted tubule

s.
A. 50 mm Hg B. loop of Henle
B. 60 mm Hg C. distal convoluted tubule
C. 70 mm Hg D. collecting duct
D. 80 mm Hg
18. The following data were obtained on a
15. The rate-limiting step in the biosynthesis of patient: inulin clearance = 125 ml/min; plasma

B. angiotensinogen
C. angiotensin II
iim
angiotensin II is the synthesis of:
A. renin
glucose = 400 mg/dL. If the tubular maximum
for glucose were 350 mg/min, what would the
glucose excretion rate be?
A. 50 mg/min
B. 100 mg/min
All about angiotensin II: C. 150 mg/min
Which of the following statements regarding D. 200 mg/min
angiotensin II are true and which are false?
4a
1. It is a much more potent vasopressor 19. Reabsorption of water in the PCT:
compared to norepinephrine; A. is chiefly driven by osmotic gradients
2. It stimulates aldosterone release from B. occurs through water channels in apical
adrenal cortex; membranes of tubular epithelial cells
3. It inhibits ADH release; C. is facilitated by vasopressin
4. It has a dipsogenic effect; D. all of the above statements are correct
5. It reduces Na reabsorption in PCT;
6. It facilitates norepinephrine release from 20. Principal cells (P cells) in the collecting duct
m

postganglionic sympathetic neurons; are concerned with:


7. It has a greater constrictor effect upon A. acid secretion
glomerular afferent arterioles; B. potassium reabsorption
8. It reduces renal plasma flow; C. ADH-stimulated water reabsorption
9. It causes mesangial cells to contract; D. Na reabsorption
Ai

10. Normally, it serves to maintain GFR;


11. It increases Na and H2O reabsorption by a 21. Which ion is both secreted and absorbed by
direct action on PCT the nephron?
12. It plays an important role in cardiac A. Na
remodeling after a myocardial infarction; B. K
13. Angiotensin III has 100% aldosterone- C. Cl
stimulating activity; D. Ca
14. Angiotensin III is a more potent pressor than
angiotensin II; 22. The water-retaining effect of ADH in the
collecting ducts is mediated mainly by:

83
A. AQP-1
B. AQP-2 30. Water deprivation fails to produce an
C. AQP-3 increase in urine osmolality in:
D. AQP-4 A. neurogenic diabetes insipidus
B. nephrogenic diabetes insipidus
23. The major humoral regulator of the
concentration of sodium in plasma is: 31. The tubular fluid at the end of the PCT is
A. aldosterone ____ with respect to plasma
B. vasopressin A. isotonic

in
C. renin B. hypotonic
D. angiotensin II C. hypertonic

24. Plasma potassium levels are chiefly 32. Which is the concentrating segment of the
regulated by: nephron?

s.
A. aldosterone A. Descending limb of loop of Henle
B. vasopressin B. Thin ascending limb of loop of Henle
C. renin C. Thick ascending limb of loop of Henle
D. ANP D. Collecting ducts

25. In the presence of vasopressin, most of the 33. Which portion of the tubule is impermeable

A. PCT
B. loop of Henle
C. DCT
D. collecting ducts
iim
filtered water is reabsorbed in the: to water?
A. PCT
B. Thin descending limb of LOH
C. Thick ascending limb of LOH
D. Cortical collecting duct

26. In the absence of vasopressin, most of the 34. The diluting segment of the nephron is:
filtered water is reabsorbed in the: A. PCT
4a
A. PCT B. descending limb of loop of Henle
B. loop of Henle C. ascending limb of loop of Henle
C. DCT D. cortical collecting duct
D. collecting ducts
35. In the nephron, free water is formed in:
27. Potassium secretion occurs mainly in: A. descending limb of loop of Henle
A. PCT B. thin ascending limb of loop of Henle
B. DCT C. thick ascending limb of loop of Henle
m

C. collecting ducts D. collecting ducts


D. loop of Henle
36. Na-K-2Cl transporter in the apical
28. ADH secretion is stimulated by: membrane of the thick ascending limb of the
A. water loop of Henle is an example of:
Ai

B. ethanol A. primary active transport


C. ACE inhibitors B. secondary active transport
D. food intake C. passive transport
D. countertransport
29. Which one of the following inhibits ADH
secretion? 37. What fraction of filtered water is reabsorbed
A. Hyperosmolality in the loop of Henle?
B. Exercise A. 15%
C. Angiotensin II B. 25%
D. Ethanol C. 35%

84
D. 5%
46. What contributes most to the osmolality of
38. What fraction of filtered NaCl is reabsorbed the medullary interstitium? Urea / NaCl?
in the loop of Henle?
A. 15% 47. Which segment of the nephron is permeable
B. 25% to urea?
C. 35% A. Descending limb of loop of Henle
D. 5% B. Thin ascending limb of loop of Henle
C. Thick ascending limb of loop of Henle

in
39. The fluid entering the distal tubule is:
A. always isotonic 48. The principal regulator of plasma osmolality
B. always hypotonic is:
C. isotonic / hypotonic but not hypertonic A. plasma [Na]
D. isotonic / hypertonic but not hypotonic B. antidiuretic hormone

s.
C. aldosterone
40. Na-K ATPase content is highest in the: D. angiotensin II
A. proximal convoluted tubule
B. distal convoluted tubule 49. Which is the fundamental mechanism
C. collecting duct generating hypertonicity in the medullary
D. thick ascending limb of loop of Henle interstitium?
iim
41. Loop diuretics (example, frusemide) inhibit:
A. 1Na-1K-2Cl transport in the TAL
B. Na reabsorption in the collecting duct
C. aquaporins in PCT
A. Active transport of NaCl in TAL
B. ADH action in the collecting ducts
C. Passive recirculation of NaCl in MI
D. Urea permeability of collecting ducts

D. carbonic anhydrase in PCT 50. The highest amount of H ions is secreted in:
A. PCT
42. Vasa recta receive blood from the: B. DCT
4a
A. afferent arteriole of cortical nephron C. collecting ducts
B. afferent arteriole of juxtamedullary nephron D. loop of Henle
C. efferent arteriole of cortical nephron
D. efferent arteriole of juxtamedullary nephron 51. Most of the H+ is secreted in the:
A. proximal convoluted tubule
43. The main function of the vasa recta is: B. loop of Henle
A. filtration of fluid into the tubules C. distal convoluted tubule
B. renin secretion D. collecting duct
m

C. to maintain the hypertonicity in the


medullary interstitium 52. The principal mechanism of H+ secretion in
the proximal convoluted tubule is:
44. The system for the formation of concentrated A. NaH exchange
or dilute urine does not include the: B. HATPase
Ai

A. proximal convoluted tubule C. HK ATPase


B. loop of Henle D. HCa exchange
C. collecting ducts
D. vasa recta 53. The principal mechanism for acid secretion
E. medullary interstitium in the nephron is:
A. HATPase
45. Free water formation in the nephron is B. NaH exchanger
essential for production of: C. HCa exchanger
A. dilute urine D. HK ATPase
B. concentrated urine

85
54. HCO3 is chiefly reabsorbed in: C. collecting ducts
A. PCT D. loop of Henle
B. DCT
C. CCD 62. Urinary ratio of NH4+ to titratable acid
D. loop of Henle ranges from:
A. 12.5
55. The most abundant buffer in proximal B. 25
tubular fluid is: C. 0.10.5
A. bicarbonate D. 57

in
B. phosphate
C. ammonia 63. Which of the following is normally not lost
in the urine?
56. The principle site of acidification of tubular A. Na
fluid is: B. K

s.
A. proximal convoluted tubule C. Cl
B. loop of Henle D. HCO3
C. distal portions of nephron
D. urinary bladder 64. The [H+] of arterial plasma is normally
about:
57. In urine, most of the H+ is tied up with: A. 20 nmol/L
A. bicarbonate
B. phosphate
C. ammonia
iim
58. Normally, most of the titratable acidity of
B. 30 nmol/L
C. 40 nmol/L
D. 50 nmol/L

65. The Henderson-Hasselbalch equation states


urine is attributable to acid buffered by: that when a buffer acid is half dissociated the pH
A. bicarbonate of the solution is equal to:
B. phosphate A. pKa
4a
C. ammonia B. 7.0
D. lactate C. 2 pKa
D. 0.5 pKa
59. Which of the following statements is / are
correct regarding the role of kidney in acid-base 66. A buffer is most effective when the pH of
balance? the solution is close to:
A. The kidney can excrete CO2 as free H a. pKa
B. Acid secretion is a passive process b. 2 pKa
m

C. Most acid secreted in the PCT acidifies c. 0.5 pKa


urine
D. Acid secretion in collecting ducts is 67. Quantitatively, the most important buffer of
facilitated by aldosterone carbonic acid in blood is:
A. bicarbonate
Ai

60. The intercalated cells (I cells) in the B. plasma protein


collecting duct are concerned with: C. phosphate
A. acid secretion D. hemoglobin
B. controlled bicarbonate absorption
C. ADH-stimulated water reabsorption 68. HCO3 cannot buffer protons formed from:
D. Na reabsorption A. lactic acid
B. sulfuric acid
61. Most of the NH3 is produced in: C. carbonic acid
A. PCT
B. DCT 69. The anion gap is elevated in:

86
A. metabolic acidosis clinical method to estimate GFR. See p. 258, Ch
B. metabolic alkalosis 47, In: Harrisons Principles of Internal
C. respiratory acidosis Medicine, 14th edition.
D. respiratory alkalosis
E. both (a) and (c) Phenol red is used to estimate renal plasma flow.
It is secreted by tubular cells. Isotonic mannitol
70. The anion gap reflects the plasma could be used for measuring GFR.
concentration of:
A. HCO3 12. PAH clearance = 330 ml/min

in
B. chloride Effective renal plasma flow (ERPF) = 330
C. protein anions ml/min.
D. phosphates True renal plasma flow = ERPF/PAH extraction
ratio = 330/0.9 = 367 ml/min.
Answers: Renal Physiology Renal blood flow (RBF) = RPF (100 / 100

s.
1A 2B 3C 4B 5B Hct).
6ABC 7B 8B 9D 10A In this case renal blood flow is 477 ml/min.
11A 12A 13CD 14A 15C
16B 17A 18C 19AB 20CD 13-14. Renal autoregulation is a mechanism to
21B 22B 23B 24A 25A defend changes in GFR and renal blood flow in
the face of changes in BP. As the name suggests,
26A
31A
36B
41A
46
51A
27C
32A
37A
42D
47B
52A
28D
33C
38B
43C
48B
53B
iim
29D
34C
39B
44A
49A
54A
30AB
35C
40D
45AB
50A
55A
it is essentially independent of extrinsic
innervation, although it is influenced by renal
sympathetic activity. The effect of intense
stimulation of the renal sympathetic nerves is to
reduce GFR.
56C 57C 58B 59AD 60AB
Renal autoregulation of GFR is overwhelmed
61A 62A 63D 64C 65A
when mean arterial pressure falls below 70 mm
66A 67D 68C 69A 70CD
4a
Hg. The increased sympathetic stimulation in
this instance would cause afferent arteriolar
All about ANG II: constriction, decreasing GFR and RPF and renal
1T 2T 3F 4T 5F failure (often called functional renal failure) will
6T 7F 8T 9T 10T ensue.
11T 12T 13T 14F 15T
18. Filtered load = plasma [glucose] GFR
Answer Explanations: Amount of glucose filtered per minute = 500 mg
m

1-3. Clearance is the volume of plasma that is Glucose excretion rate = filtered load Tmax
cleared of a substance per minute for glucose = 500 350 = 150 mg/min.
Clearance of a substance X = UXV/PX
UX: concentration of substance in urine 22. Rapid diffusion of water across cells
PX: concentration of substance in plasma depends upon the presence of water channels
Ai

V: urine flow rate in ml/min called aquaporins. There are several types of
aquaporins. Vasopressin mediates its
Inulin clearance gives the best estimate of GFR antidiuretic effect by increasing the number of
since it is neither reabsorbed nor secreted by the AQP-2 channels in the luminal membrane of
nephron; it is nontoxic, freely filtered and does collecting duct epithelial cells.
not affect hemodynamics; endogenous
creatinine clearance is slightly higher than 23. Changes in water excretion are a commoner
inulin clearance since creatinine is also secreted cause of changes in plasma Na concentration
by the tubules. Despite this shortcoming, rather than changes in Na excretion. ADH being
creatinine clearance remains the most common the hormone that regulates water excretion is an

87
important determinant of plasma Na of the filtered solute is reabsorbed in the loop of
concentration and osmolality. For example, Henle.
diabetes insipidus, which is caused by ADH
deficiency, is characterized by excretion of large 41. The thick ascending limb of the loop of
volumes of dilute urine and plasma [Na] Henle is impermeable to water. It is permeable
increases. only to solute. 25% of the filtered Na is
recovered in the loop of Henle. That is why loop
24. Aldosterone is the hormone regulating diuretics are very efficacious. They inhibit the
secretion of potassium in the collecting ducts, 1Na-1K-2Cl symporter and increase solute

in
and hyperaldosteronism is typically associated delivery to distal portions of the nephron. The
with hypokalemia. unabsorbed solutes in the tubular lumen retain
their water equivalent and a considerable
25-26. 60-70% of the filtered water is diuresis results because the reabsorptive capacity
reabsorbed in the proximal convoluted tubule. of the collecting duct is overwhelmed.

s.
This does not depend upon vasopressin. Even in
the absence of vasopressin, about 65% of the 46. Just a friendly question both NaCl and urea
filtered fluid is reabsorbed in the PCT. contribute substantially (and to a similar extent)
to the tonicity of the medullary interstitium.
When vasopressin is absent, 24-hr urine volume
increases to 21 L. This is about 12% of the 56. Tubular fluid is mainly acidified in the distal
filtered fluid load. iim
30. Normally, water deprivation (for say 7 hr)
produces a significant increase in ADH secretion
Consequently, urine becomes more concentrated
portions of nephron (distal convoluted tubule
and collecting ducts). Since bicarbonate has
been completely reabsorbed by the time fluid
reaches this portion of the tubule, secreted H
acidifies the tubular fluid. The concentration of
and its osmolality exceeds 700 mOsm/kg H2O. phosphate increases however and it buffers some
of the secreted H. This allows more H to be
In neurogenic diabetes insipidus, urine does not secreted without acidifying the urine beneath a
4a
become concentrated because of a deficiency of limiting pH.
ADH.
69, 70. Anion gap is calculated as:
In nephrogenic diabetes insipidus, urine does not {[Na+] + [K+]} {[Cl-] + [HCO-3]}
become concentrated because the kidneys do not All of them are expressed in mmol/L. [K+] may
respond to ADH. This is usually due to defects be omitted from the equation. But if this is done,
in V2 receptors or ADH responsive aquaporin then the normal value will be lower by about 4
(AQP-2) water channels in the collecting ducts. mM.
m

After administration of a vasopressin analog, Here is an easy way to understand the anion gap
urine would become concentrated in neurogenic In plasma, the sum of charges of positive
diabetes insipidus but not in nephrogenic ions is equal to the sum of negative charges
diabetes insipidus. Plasma is thus electroneutral
Ai

Some ions in plasma are routinely measured


34. The ascending limb of the loop of Henle is (measured ions)
impermeable to water and is the diluting Some are not (unmeasured ions)
segment of the nephron. The fluid emerging What are the major cations in plasma?
from the ascending limb of the loop of Henle is They are Na and K
always hypotonic whether the urine excreted is What are the major anions in plasma?
dilute or concentrated. They are chloride and HCO3
Measured cations are Na and K
40. Na-K ATPase content is greatest in the thick Measured anions are Cl and HCO3
ascending limb of the loop of Henle. About 25% Unmeasured cations are Ca and Mg

88
Normally, the unmeasured anions are plasma in life
phosphate, sulfate and protein anions. threatening acidosis
7 100 Pure water
The concentration of unmeasured cations is 7.1 80 Inside RBC
small that it can be ignored 7.33 50 Cerebrospinal fluid
In summary, [measured cations] 7.4 40 Normal arterial plasma
[measured anions] = [unmeasured anions] 7.7 20 Severe alkalosis
8 10 Pancreatic juice
Anion gap = {[Na] + [K]} {[Cl] + [HCO3]} =
The Henderson-Hasselbalch equation reads thus:
[unmeasured anions] pH = pKa + log [salt] / [acid]

in
For example, consider the H2CO3 HCO3 system
When does the anion gap increase and why? The acid is H2CO3
In metabolic acidosis (example, lactic acidosis), The conjugate base is HCO3
pH decreases. The H ions are buffered by
[H2CO3] [H+] + [HCO3-]
HCO3. The concentration of lactate (an

s.
unmeasured anion) increases. Therefore, anion pH = pK + log [HCO3-] / [H2CO3]
gap increases. The anion gap increases only
when there is an increase in the concentration of You can imagine that if buffer acid were half dissociated,
i.e., the concentration of acid and the conjugate base are
an unmeasured anion. This happens only in equal, then, [salt] = [acid], and the pH would be equal to
metabolic acidosis. pK. A buffer system works best when the pH of the
solution is close to the pK value.
Examples of metabolic acidoses and the
unmeasured anions:
Metabolic acidosis
Lactic acidosis
Ethylene glycol
Unmeasured anion
Lactate
Oxalate
iim Buffers in 1 L arterial plasma:
Buffer

HCO3
Hb
Concentration in
plasma (mM)
24
9
poisoning Protein 15
Total buffer base 48
Ketoacidosis Acetoacetate,
betahydroxybutyrate What is the role of HCO3 in plasma?
4a
HCO3 in plasma combines with H liberated by the
*************************************** formation of fixed acids (lactic acid, ketoacids).
Lactic acid Lactate + H
Supplement: H is buffered by HCO3 to form H2CO3
pH = - log [H] in moles/liter
H2CO3 is unstable.
Suppose [H] = 100 nM, pH = - log [10-7]
It dissociates to form CO2 + H2O.
= - 7 [-1] = 7 CO2 is volatile. The lungs excrete CO2.
pH Hydrogen ion concentration The hydrogen ions formed from lactic acid are eliminated
7 100 nM as carbon dioxide.
m

6 1000 nM
5 10000 nM The Henderson-Hasselbalch equation may be modified to
4 100000 nM read thus:

H ion concentration is expressed in logarithmic as well as [H+] in nM = 24 PaCO2 (mm Hg)


linear scale. It is expressed on a log scale so that the values
become convenient to handle (because hydrogen ion [HCO3-] mM
Ai

concentration is of the order of nanomoles). However, it This is the modified Hendersons equation.
must be noted that a unit change in pH say from 6 to 7 Substitute normal values of PaCO2 and plasma HCO3 and
represents a ten-fold change in H ion concentration. The you can easily remember the equation. Normally, [H+]
table above is meant to emphasize this. nmol/L = 40 nmol. This is on the left side of the equation.
40 = 24 (a constant) 40 / 24
pH of various body fluids: On the right side, 40 = normal PaCO2 and 24 = normal
pH [H] concentration Compartment / plasma HCO3. The constant 24 has been derived taking
in nanomoles per condition into consideration the solubility coefficient of carbon
liter dioxide at body temperature, so that PaCO2 can be
1 100 million Gastric juice substituted in the equation instead of carbonic acid.
6.9 120 ICF (muscle); arterial

89
10. CRITICAL CARE 5. What is the approximate dead space of a
normal 70-kg man breathing through a tube that
PHYSIOLOGY has a radius of 5 mm and a length of 100 cm?
A. 150 ml
Please examine all options and select all
B. 180 ml
correct answers. Occasionally, none of the
C. 230 ml
options may be correct.
D. 280 ml
Reproduced from WF Ganong, Review of Medical
1. An 8 year old child is being treated with Physiology, 2005.
mechanical ventilation and other disease-

in
specific therapy for acute respiratory distress 6. An (acute) increase in intracranial tension
syndrome. She is being administered 60% does not occur as a result of:
oxygen. Her PaCO2 and PaO2 were found to be A. breath holding in full inspiration
20 and 100 mm Hg respectively. Which of the B. ventilation with 20 cm H2O PEEP
following statements is correct? C. Valsalva maneuver

s.
A. Gas exchange in the lungs is normal D. decrease in PaCO2
B. There is a defect in pulmonary gas exchange
C. She has alkalosis 7. A 60 yr old man is on mechanical ventilation
for management of acute pulmonary edema.
2. An eight-year old boy was put on a ventilator Arterial plasma pH = 7.5, plasma [HCO3] = 24
iim
after he developed acute respiratory failure. FiO2
was 60%, and PaCO2 was found to be 60 mm
Hg. What is the expected O2 tension of arterial
blood?
A. 340 mm Hg
B. 290 mm Hg
mM; PaCO2 = 30 mm Hg, PaO2 = 50 mm Hg,
and O2 saturation of Hb is 84%.
Ventilator settings are as follows: tidal volume =
1000 ml, respiratory frequency = 12 breaths per
min, and FiO2=1.
Which of the following statements is correct
C. 240 mm Hg with regard to management?
D. 190 mm Hg A. Minute ventilation should be increased
B. PEEP must be instituted
4a
3. During mechanical ventilation, the addition of C. Calcium gluconate must be given
positive end-expiratory pressure (PEEP): D. IV infusion of 20% mannitol is required
A. is indicated in the management of ARDS
B. decreases cardiac output 8. Does any one of the following physiological
C. is associated with increased ADH secretion parameters taken alone indicate that arterial
D. allows FiO2 to be metered down blood oxygenation in a critically ill patient is
satisfactory? If so, which one?
m

4. A 60-yr old man is being mechanically A. (Aa) O2 gradient = 10 mm Hg


ventilated after he developed respiratory arrest. B. PaO2 = 95 mm Hg
The ventilator settings are as follows: VT = 500 C. O2 saturation of Hb > 90%
ml; respiratory rate = 12/min; FiO2 = 60%; and D. Blood [Hb] = 12 g/dL
PEEP = 5 cm H2O. His arterial [H+] is 20 nM;
arterial plasma HCO3 is 24 mM; and PaO2 is 200 9. In ventilated patients, oxygenation of arterial
Ai

mm Hg. Which of the following statements is blood should be monitored by (tick all that
correct? apply):
A. FiO2 will need to be increased A. (Aa) O2 gradient
B. Intracranial tension is elevated B. PaO2
C. Minute ventilation needs to be decreased C. Blood hemoglobin concentration
D. PEEP could be increased D. O2 saturation of Hb

10. The organ that excretes the greatest amount


of acid is:

90
A. lungs Expected PAO2 = 400 mm Hg;
B. kidneys Expected PaO2 = 390 mm Hg;
C. skin Actual PaO2 = 100 mm Hg;
D. GIT (Aa) O2 difference = 290 mm Hg
This represents a considerable defect in gas
11. The most common acid-base disturbance in exchange. In fact, this is the reason why this girl
patients on mechanical ventilators is: was ventilated in the first place; acute
A. metabolic acidosis respiratory distress syndrome is characterized by
B. metabolic alkalosis hypoxemia and normocapnia or even

in
C. respiratory acidosis hypocapnia (type I respiratory failure).
D. respiratory alkalosis Regarding option c, it is impossible to tell
whether she has alkalosis because plasma pH is
12. Sodium bicarbonate should not be not known.
administered to a patient with:

s.
A. arterial blood pH of 7.1 2. Use the alveolar gas equation and first
B. serum [HCO3] of 12 mM determine the oxygen tension of alveolar gas
C. a PaO2 of 50 mm Hg PAO2.
D. a PaCO2 of 68 mm Hg PAO2 = [(PB-PH20) FiO2] [(PACO2) / R].
Here PB = 760 mm Hg, PH20 = 47 mm Hg, FiO2 =
13. Hyperbaric oxygen therapy is useful in the 0.6, R = 0.8].

A. cyanide poisoning
B. carbon monoxide poisoning
C. diabetic leg ulcers
D. gas gangrene
iim
management of (tick all that apply): Since PaCO2 = 60 mm Hg, PACO2 will be about
the same, since CO2 rapidly equilibrates across
the alveolar capillaries. It is much more soluble
than oxygen.

E. decompression sickness Substituting, we get, alveolar PO2 = 350 mm Hg.


Normally, the (A-a) O2 gradient is less than 10
14. Which of the following is not required to mm Hg. So we would expect arterial PaO2 to be
4a
calculate total dead space using Bohrs about 340 mm Hg.
equation?
A. End-tidal CO2 Thus, it is clear that alveolar PO2 is much less
B. PaO2 than PO2 of inspired air. This is because it gets
C. PaCO2 saturated with water vapor and then diluted with
D. Tidal volume carbon dioxide in the alveolar gas.

Answers: Critical Care Physiology The reason for subtracting PACO2 /R is that
m

1B 2A 3all 4CD 5C alveolar PO2 gets diluted by alveolar CO2 which


6D 7B 8 No 9all 10A is equal to arterial CO2 (since CO2 is highly
11D 12D 13all 14B soluble). This is divided by R, the respiratory
exchange ratio; i.e., the ratio of the volumes of
Answer Explanations: CO2 to O2 exchanged across the lungs per unit
Ai

1. The approach: time. R is for practical purposes assumed to be


What is the PAO2? 0.8; i.e., 200 ml CO2 / 250 ml O2. Let us say an
What is the expected PaO2? equal volume of CO2 and O2 were exchanged;
Is there a difference between expected PaO2 then the denominator would be 1. Since only a
and actual PaO2? lesser volume of CO2 is exchanged (i.e. 200 ml)
Alveolararterial O2 gradient is normally 10 and more O2 is taken up (i.e. 250 ml), alveolar
mm Hg oxygen gets even more diluted by CO2. This is
given by R; i.e. VCO2 /VO2).
Solution: Using the alveolar gas equation, we
get:

91
3. In patients who are on mechanical ventilators, Now use the alveolar gas equation and
intrathoracic pressure (ITP) becomes positive determine his PaO2.
during inspiration, and when PEEP is added, it PAO2 = (760 47) 0.6 (20/0.8) = 400 mm Hg
becomes even more positive. The beneficial His PaO2 is expected to be at least 380 mm Hg
effect of PEEP is that it recruits previously whereas his arterial PaO2 is only 200 mm Hg.
atelectatic alveoli and increases the surface area The (A-a) O2 gradient = 180 mm Hg is quiet
and time available for oxygenation; furthermore, high and reflects poor gas exchange (a right-to-
it keeps fluid from entering the pulmonary left shunt).
interstitium. However, the hemodynamic effect

in
of raised ITP would be to reduce venous return Increasing FiO2 alone can never ameliorate
and consequently cardiac output. Note that it is hypoxemia due to a shunt because the oxygen
possible that the beneficial effect of PEEP on administered would not equilibrate with the
oxygenation might be offset by a low cardiac shunt. In this case, the underlying disease
output. To prevent this from happening, fluids process has to be addressed.

s.
must be administered, and cardiac output has to
be monitored. Increasing PEEP in this patient would certainly
improve oxygenation if the poor gas exchange,
Oxygenation could be improved by one or more manifest as high (A a) DO2, were due to a
of the following means: generalized process such as ARDS or
Increasing minute ventilation but this has the cardiogenic pulmonary edema. If that happens,

volumes.
iim
effect of removing CO2; further, there is a
risk of volutrauma with higher tidal

Increasing FiO2 but there is a limit to the


time for which oxygen rich gas could be
FiO2 could then be reduced. However, cardiac
output has to be monitored.

In this patient, it is however important to reduce


minute ventilation since it will be difficult to
administered; i.e., there is a risk of lung wean him at a later stage if his PaCO2 is as low
damage due to high FiO2. as 20 mm Hg. CO2 is the major stimulus for
Administering PEEP; however, this comes respiration. If the PaCO2 is too low, the drive for
4a
with two risks: one, of barotrauma due to spontaneous respiration itself is removed.
overdistended lung zones; and two,
reduction in cardiac output due to a decrease In ventilated patients, it is not uncommon to
in venous return. The risk of barotrauma and vary minute ventilation so that PaCO2 is around
consequent pneumothorax is significant 45 mm Hg (i.e. a little above normal) though this
because even in patients with lung disease, represents a trade off from the possibility of
not all lung zones are affected, and some better oxygenation. This is called permissive
might be even normal. Lung zones that are hypercapnia. This is usually accompanied by
m

normal will be overdistended by the applied hypoxemia (also called permissive hypoxemia).
PEEP and may rupture. This is an attempt to preserve the drive for
spontaneous respiration. This makes it more
4. [H+] = 24 [PaCO2] / [HCO3] likely that, as soon as the underlying disease
Here [H+] = 20 nM, and [HCO3] = 24 mM, so process is fully addressed, the patient could be
Ai

PaCO2 = 20 mm Hg weaned to start breathing on his own.

An increase in PaCO2 raises intracranial tension 5. Radius = 0.5 cm


(ICT) by causing cerebral vasodilation. In this Length = 100 cm
case, ICT cannot be high since [H+] is quite low; Volume of the tube
i.e. it is only 20 nmol/L (normal [H+] of arterial = (radius squared) length
plasma is 40 nmol/L). Also [HCO3] is normal, = (22/7) (0.5)2 100
so the patient has respiratory alkalosis. This is = 80 ml
because of hyperventilation.

92
This adds to the normal anatomical dead space His PaO2 = 50 mm Hg
of about 150 ml to make it 230 ml in all. Problem: moderate hypoxemia
Conclusion: D(A-a)O2 only tells us the
6. PaCO2 will gradually rise during breath efficiency of oxygen exchange
holding and it will raise ICT by causing cerebral
vasodilation. Intrathoracic pressure is increased Case 2: An anemic patient with a PaO2 of 95
during PEEP and Valsalva maneuver and this mm Hg performing mild-moderate exercise
will reduce venous return from the skull and PaO2 = 95 mm Hg
raise ICT. A decrease in PaCO2 per se will Hb = 7.5 g/dL

in
produce cerebral vasoconstriction and this is in Oxygen carrying capacity is reduced by half
itself can not cause ICT to increase. Significant exercise intolerance due to
anemic hypoxia
7. This patient, who has acute pulmonary edema,
has arterial hypoxemia despite administration of Case 3: A patient who is on a ventilator and has

s.
100% O2. The expected PaO2 can be calculated a PaO2 of 95 mm Hg
to be about 650 mm Hg but his arterial PaO2 is Patient is given 60% oxygen
only 50 mm Hg. This represents a formidable His PaCO2 is 40 mm Hg
defect in oxygen exchange. In this situation, Then, expected PaO2 is 400 mm Hg
institution of PEEP would improve oxygenation If actual PaO2 is 95 mm Hg then, this
by inflating atelectatic alveoli. Secondly, PEEP represents a defect in gas exchange.
iim
would reduce the gradient for filtration of fluid
across the pulmonary capillaries. FiO2 can
subsequently be tailored down to an acceptable
value; this is important since prolonged
administration of high concentrations of O2
Case 4: A PaO2 of 90 mm Hg may occur even in
the absence of Hb!

Case 5: A person can have a blood [Hb] of 3


(especially > 24 h) may result in oxygen g/dL and yet the O2 saturation of Hb may be >
toxicity. This patient has respiratory alkalosis, a 90%;
consequence of hyperventilation. Note that CO2
4a
elimination is hardly of any concern even in the The properties of Hb are such that it is 90%
face of severe impairment of oxygen exchange. saturated with O2 even if PaO2 is only 60 mm
This is because CO2 is much more soluble. Hg. While this is a safety characteristic, a
Patients with respiratory alkalosis may develop decline in PaO2 below 60 mm Hg is associated
hypocalcemia but in this instance the alkalosis with a steady decline in O2 saturation of Hb
can be readily corrected by reducing minute (please examine the oxygen dissociation curve
ventilation. The patient also does not require for adult hemoglobin). Further, if O2 saturation
mannitol since there is no evidence from the of Hb is 90% but blood Hb is only 3 g/dL, then
m

data provided that ICT is raised. In fact, PaCO2 he will definitely be hypoxemic!
is lower than normal.
Case 6: Neither does a blood [Hb] of 12 g/dL by
8. None of these parameters taken alone will itself guarantee adequate oxygenation.
give us this information.
Ai

An individual might have a blood [Hb] of 12


To illustrate, consider the following situations: g/dL and still not be able to transport enough
A (A-a) O2 gradient of 10 mm Hg only tells us oxygen to the tissues if the hemoglobin itself is
that diffusion capacity of the lungs for oxygen is not pure. For example, with even small amounts
normal. of carboxy hemoglobin, the oxygen dissociation
curve is shifted to the left. Simple methods of
Case 1: a healthy individual at high altitude blood Hb estimation cannot distinguish between
D (Alveolar arterial) O2 gradient = 10 mm normal and abnormal hemoglobin derivatives.
Hg
His PAO2 = 60 mm Hg

93
See Prakash ES and Madanmohan. What does one mean by arterial 14. Bohrs equation for determining total
blood oxygenation?
http://advan.physiology.org/cgi/content/full/30/1/46 dead space: PECO2 VT = PaCO2 (VTVD)
If PECO2 = 28 mm Hg, PaCO2 = 40 mm Hg, and
9. All of these parameters should be monitored. VT = 500 ml, then, dead space = 150 ml
Furthermore, one should know if hemoglobin
itself is normal Hb A or not. End-tidal carbondioxide is measured by
capnometry. Note that PaCO2 represents gas
12. Injected NaHCO3 dissociates to give Na and equilibrated with perfused alveoli. If there is
HCO3. The HCO3 buffers H ions in ECF and significant V/Q mismatch, PaCO2 will be higher

in
becomes H2CO3 than PACO2.
H2CO3 will dissociate to give CO2 and H2O.
Normally, the lungs will eliminate CO2 readily. VD/VT is determined and used as an index of
However, if the PaCO2 is already as high as 68 dead space ventilation.
mm Hg, this means that minute ventilation is ************************************

s.
already inadequate and such a patient would
require mechanical ventilation since he has
respiratory acidosis (type II respiratory failure).
The administration of NaHCO3 in this instance
will aggravate acidosis. Sodium bicarbonate is

iim
indicated only for the management of metabolic
acidosis. So get the CO2 out down to a level that
will make him breathe. Intubate and ventilate!

13. The principle of hyperbaric oxygenation has


been discussed in the chapter on pulmonary
physiology. 100% oxygen is administered at
pressures exceeding atmospheric (usually 23
atmospheres) for brief periods of time. This
4a
greatly increases arterial oxygen tension if
pulmonary gas exchange is normal.

Hyperbaric oxygenation is resorted to only when


all other means of increasing oxygen carrying
capacity of arterial blood have failed.
m

High concentrations of oxygen might serve to


displace cyanide from cytochrome oxidase; in
carbon monoxide poisoning, high concentrations
of oxygen help overcome competitive inhibition
by carbon monoxide; diabetic leg ulcers due to
obliterative peripheral vascular disease might
Ai

benefit from an increase in the content of


dissolved oxygen content of blood; in gas
gangrene, the causative organisms are anerobic
and will be killed by high oxygen concentration;
in decompression sickness, the rationale is to
recompress as well as oxygenate and this is
achieved by hyperbaric oxygenation, and of
course, this is followed by slow decompression.

94
SELF-SCORERS
Self-Scorer 1: General and cellular basis
of medical physiology

Check all correct answers

1. 2 liters of Ringers lactate is administered to a Fig A Fig B

in
12-year old boy with isotonic dehydration. What
will be the change in ICF volume? Which of the following statements is correct?
A. No change A. B is transported by facilitated diffusion
B. Increases by 2 liters B. Transport of B may be ATP dependent
C. Increase C. There is a transport maximum for A

s.
D. Decreases by 0.5 liter D. Substance A would move in one direction
only
2. Cell volume and pressure is mainly dependent
upon activity of: 7. Which of the following directly mediates ion
A. Na glucose cotransporter transport across epithelia?
A. Gap junctions
B. Na-K pump
C. glucose transporter
D. Na-Ca exchanger
iim
3. Which of the following ions is not transported
across the cell membrane by a primary active
B. Tight junctions
C. Desmosomes
D. Hemidesmosomes

8. Which of the following transport processes is


transport mechanism? active?
A. Na A. Acidification of lysosomes
B. Cl B. Actin-myosin crossbridge cycling
4a
C. K C. Exocytosis
D. Ca D. Micropinocytosis

4. Which of the following substances can be 9. Which of the following transport processes is
used as a marker for the ECF compartment? mediated by a carrier protein in the plasma
A. Nonmetabolizable sugars membrane?
B. Glucose A. Glucose uptake through SGLT-1
B. Na influx through Na channels
m

C. Radio-iodinated albumin
D. D2O C. Na-K ATPase
D. Water fluxes through aquaporins
5. Sodium-glucose cotransport in the intestine
and kidneys is an example of: 10. The volume of RBC placed in a NaCl
A. primary active transport solution with an osmolality of 280 mOsm/Kg
Ai

B. secondary active transport H2O is 100 femtoliters. What will be the steady
C. facilitated diffusion state volume when they are placed in a NaCl
D. passive transport solution with an osmolality of 350 mOsm/Kg
H2O?
6. Figures A and B given below depict the A. 120 fL
relationship between transport rates (in Y-axis, B. 80 fL
in arbitrary units) and concentration gradients C. 50 fL
(in X-axis, in arbitrary units) of two substances D. Data inadequate
A and B respectively.

95
Answers to Self-Scorer 1: 7. Transport from the luminal side to the blood
General and cellular basis of medical side of an epithelium is regulated by tight
physiology junctions that link the apical ends of polarized
1C 2B 3B 4A 5B epithelial cells.
6B 7B 8all 9AC 10B
8. All of them are examples of active processes.
There are proton translocating ATPases in
Notes:
lysosomes.
1. Ringers lactate is an isoosmotic solution;

in
however, as lactate is taken up by liver and other
9. The term carrier-mediated transport
cells and metabolized, the infusion of it adds a
emphasizes the fact that the transported species
definite amount of water to plasma that dilutes
binds to a transport protein. Glucose attaches to
plasma in the steady state. This will, in turn,
the GLUT molecule and a conformational
cause some water to enter cells. It is hard to tell

s.
change in GLUT shuttles glucose to the interior
what the actual increase in ICF volume will be
of the cell. The transport of Na and K by Na-K
but one would expect ICF volume to increase
ATPase is also an example of carrier-mediated
especially when a large volume of Ringers
transport because both ions bind to the
lactate is administered.
transporter.

iim
2. By pumping 3 Na ions out of the cell for
every 2 K ions pumped into the cell, the Na-K
ATPase effectively removes one osmole (and
the corresponding water equivalent) from the
cell. Thus, inhibition of Na-K pump would cause
cells to swell.
In contrast, aquaporins and ion channels are not
carrier proteins; they are channels through which
water or the respective ions flow. The ions
themselves do not bind to ion channels. This is
an example of diffusion facilitated by the
presence of proteins in the cell membrane
(facilitated diffusion). Some authors also call it
3. Chloride transport across the cell membrane
simple diffusion.
occurs by secondary active transport
4a
mechanisms or diffusion but not by a primary
10. The following mathematical relationship
active transport mechanism.
applies to the situation described in the question.
Pi Vi = Pf Vf
4. Examples of substances that distribute
Pi and Pf are initial and final osmolalities and Vi
exclusively in the ECF include inulin, mannitol,
and Vf are initial and final volumes respectively
and sucrose. Isotonic solutions of inulin,
280 100 = 350 Vf
mannitol and sucrose can all be used to
Vf = 80 femtoliters
m

determine ECF volume. Radioiodinated albumin


Thus, there is a decrease in RBC volume in the
may be used for estimating plasma volume.
steady state when it is placed in hypertonic
saline.
5. The complete absorption of glucose from the
intestinal lumen (against a concentration
**************************************
gradient) is driven by the electrochemical
Ai

gradient of sodium. This is a classic example of


secondary active transport.

6. B may be transported either by facilitated


diffusion or active transport. Both are like
enzyme catalyzed reactions and exhibit
saturation kinetics. Diffusion rate is proportional
to the concentration gradient when other factors
affecting diffusion are held constant.

96
Self-Scorer 2: Physiology of nerve D. The neurotransmitter released by neuron A
opens K channels in neuron C
and muscle cells
6. Which of the following nerve fibers is most
Check all correct answers
susceptible to hypoxia?
A. Somatic motor neurons
1. Which one of the following increases B. Nociceptive afferents
excitability of cardiac muscle? C. Preganglionic autonomic neurons
A. Increase in ECF [K+] from 5 to 10 mM D. Fibers transmitting touch sensation
B. Increase in ECF [K+] from 5 to 28 mM

in
C. Decrease in ECF [K+] from 5 to 1.4 mM 7. The term size principle refers to the fact
that:
2. The force of skeletal muscle contraction A. neurons in small motor units conduct slowly
cannot be increased by: B. chronically denervated skeletal muscle
A. increasing the frequency of activation of

s.
decreases in size due to reduced release of
motor units growth factors from motor neurons
B. increasing the number of motor units C. small motor units are recruited for more
activated forceful contractions
C. increasing the amplitude of action potentials D. fast motor units are recruited after slow
in motor neurons motor units
iim
D. recruiting larger motor units

3. Which of the following is correct with regard


to numbers of each of the following in the CNS?
A. neurons >> glia > synapses
B. glia >>> synapses >>> neurons
8. Recruitment and activation of motor units is
investigated by:
A. electromyography
B. electroencepaholgraphy
C. nerve action potential recordings
C. synapses >> glia > neurons D. clinical examination of tendon jerks
D. synapses >> neurons >> glia
4a
Answers to self-scorer 2:
4. Na is not included in the assessment of the
Physiology of nerve and muscle cells
approximate value of the resting membrane
potential of nerve fibers because: 1A 2C 3C 4C 5B
A. extracellular [Na] is higher than ICF [Na] 6C 7D 8A
B. Na-K pump extrudes 3 Na for 2 K pumped
in Notes:
C. permeability of the resting membrane to Na 1. Using the Nernst equation (below) to
m

is low determine the equilibrium potential of K, we get,


D. unexcited cells are equally permeable to Na equilibrium potential for potassium
and K Ek = - 61 log [(K)i / (K)o]
Case a: Ek RMP = - 70 mV
Case b: Ek RMP = - 40 mV
5. Neuron C responds to stimulation of neuron A Case c: Ek RMP = - 122 mV
Ai

with a propagated action potential only when


neuron B (which terminates only on neuron C) is When RMP is 70 mV, the muscle cell is
also simultaneously stimulated. Which of the closest to the firing level which is about 55
following is correct? mV. In contrast, when RMP is 40 mV, it has
A. Neuron C releases glycine crossed the firing level so it would be absolutely
B. Neuron B releases an excitatory refractory or inexcitable. Hypokalemia makes
neurotransmitter the RMP more negative, and because at RMP
C. The synapse between A and C is axosomatic the heart would be filling with blood, it may stop
the heart in diastole. Similarly, intense

97
stimulation of the vagus nerve to the heart stops
the heart in diastole because acetylcholine
activates an outwardly directed potassium
current in SA node; i.e. it hyperpolarizes the SA
node and the AV node.

2. There is no larger action potential; rather,


action potentials are all or none.

in
3. The ratio of synapses to neurons in the brain
is about 1000; glia outnumber neurons by about
50 times. See p. 51 in Ch 2 and p. 85 in Ch 4 in
WFG, 2005.

s.
4. The resting cell membrane is 50 times more
permeable to K than Na. When Na and Cl are
removed from the Goldman Hodgkin equation, it
becomes the much simpler Nernst equation.

5. It is easy if you draw a schematic diagram.


iim
You will see that both neurons A and B have to
release excitatory neurotransmitters in order to
fire C (i.e., by spatial summation of EPSP).
However, repeated stimulation of either A or B
may also elicit an action potential in C by
temporal summation.

6. Type B fibers are most susceptible to hypoxia.


4a
7. Slow motor units are activated by slowly
conducting axons; fast motor units are
innervated by fast conducting axons. Slow units
are recruited first in most movements; fast motor
units, which are easily fatigable, are recruited for
more powerful contractions. Please see p. 76,
WFG, 2005.
m

8. In this technique, compound muscle action


potentials (CMAP) are recorded by placing
electrodes either on the surface of muscles or
within the muscle (when needle electrodes are
Ai

used); it is much like ECG which represents the


compounded potentials generated by heart
recorded by placing electrodes on the body
surface. CMAP is examined at rest, during mild
contraction, and forceful contractions. The
pattern of EMG during these maneuvers is
useful to distinguish between muscle weakness
due to myopathy and neuropathy.

If you would like details, please visit

98
Self-Scorer 3: Functions of the CNS B. labeled-line principle
C. Weber-Fechner law
1. Which of the following cells in the CNS is D. law of projection
primarily involved in the reuptake of excitatory
neurotransmitters released by neurons? 8. The receptor for autogenic inhibition (or the
A. Astroglial cell inverse stretch reflex) is:
B. Microglial cell A. hair cell
C. Oligodendroglial cell B. muscle spindle
D. Ependymal cell C. Golgi tendon organ

in
D. glomus cell
2. Which of the following effects is not
produced by stimulation of opioid receptor? 9. Proprioceptive information reaches
A. Analgesia consciousness through the:
B. Constipation A. spinothalamic system

s.
C. Miosis B. dorsal column medial lemniscal pathway
D. Diuresis C. spinocerebellar pathway
E. Sedation D. olivocerebellar pathway
F. Euphoria
10. If light reflex is present and the
accommodation reflex absent, then the lesion is
A. Bell-Magendie law
B. the law of projection
iim
3. The phantom limb phenomenon exemplifies:

C. Mullers doctrine of specific nerve energies


D. Weber-Fechner law
most likely in the:
A. pretectal nucleus
B. ganglion cells
C. Edinger-Westphal nucleus
D. visual association cortex
4. In which sensory transduction pathway does
hyperpolarization of the receptor cell activate 11. REM behavior disorder is primarily
transmission in the pathway? characterized by:
4a
A. Phototransduction A. lack of REM sleep
B. Auditory signaling B. day-time somnolence
C. Gustation C. absence of hypotonia during REM sleep
D. Olfactory signaling D. absence of PGO spikes during REM sleep

5. The muscle spindle is not innervated by: 12. Which sensory modality is transduced by
A. A fibers free nerve endings?
A. Vision
m

B. A fibers
C. Ia fibers B. Taste
D. II fibers C. Smell
D. Sound
6. The muscle spindle remains capable of
responding to stretch even the muscle is 13. The fifth taste modality is:
Ai

contracting. This is because of: A. umami


A. presence of stretch receptors in the tendon B. imami
B. co-activation of - and -motor neurons C. himami
C. reciprocal innervation D. emami
D. autogenic inhibition E. mami

7. Mullers doctrine of specific nerve energies is 14. Muscle contraction in response to


better known as: maintained stretch is initiated by sensory inputs
A. Bell-Magendie law from:
A. nuclear bag fibers

99
B. nuclear chain fibers D. 7 ml/100 g/min
C. Golgi tendon organ
D. -motor neurons 21. Normally, the brain receives about ___
% of resting cardiac output.
15. Rhodopsin has peak sensitivity to light at a A. 5
wavelength of: B. 10
A. 405 nm C. 15
B. 505 nm D. 20
C. 605 nm

in
D. 705 nm 22. In which sensory system does excitation of
receptor results in hyperpolarization of receptor
16. Which one of the following sensory cells?
modalities does not have a separate neocortical A. Visual pathway
projection? B. Auditory pathway

s.
A. Taste C. Taste pathway
B. Smell D. Olfactory signaling
C. Vision
D. Hearing 23. Which of the following has been suggested
as a possible mediator of analgesia produced by
17. Dark adaptation is nearly maximal in about: stimulation of the raphespinal pathway?
A. 1 minute
B. 5 minutes
C. 20 minutes
D. 40 minutes
iim A. Substance P
B. Glutamate
C. Serotonin
D. GABA

18. Which of the following statements regarding 24. Which area is uniquely concerned with color
the autonomic nervous system is incorrect? vision?
A. Preganglionic sympathetic neurons are more A. V1
4a
sensitive to hypoxia compared to B. V3
postganglionic sympathetic neurons. C. LO
B. Postganglionic fibers are unmyelinated. D. V8
C. Medulla is the highest center for integration
of autonomic reflexes. 25. Which eye movements bring new objects of
D. Stimulation of parasympathetic system interest onto the fovea?
usually elicits discrete responses whereas A. Saccades
stimulation of the sympathetic nervous B. Convergence movements
m

system usually elicits widespread C. Smooth pursuit movements


physiologic responses. D. Vestibular movements

19. Normally, the total blood flow to the brain is 26. Kluver-Bucy animals exhibit:
about: A. hypersexuality
Ai

A. 250 ml/min B. visual agnosia


B. 500 ml/min C. bizarre exploratory behaviour
C. 750 ml/min D. amnesia
D. 1200 ml/min E. all of the above

20. Normally, cerebral metabolic rate for oxygen 27. Symptoms of the Kluver Bucy syndrome
(CMRO2) is about: have been shown to be reproduced by:
A. 2 ml/100 g/min A. amygdalectomy
B. 3.5 ml/100 g/min B. dorsal rhizotomy
C. 5 ml/100 g/min C. decerebration

100
D. lesions in the internal capsule 7. The same sensation is perceived no matter
how a sensory pathway is excited. Each pathway
28. In a majority of lefthanders, the left cerebral is labeled to transmit a specific sensory
hemisphere is the categorical hemisphere. modality. This is the labeled line principle or
True/False. Mullers doctrine of specific nerve energies.

29. The planum temporale is an area in the 8. Autogenic inhibition is a term that is used
superior temporal gyrus concerned with interchangeably with inverse stretch reflex. It
language-related auditory processing. indicates that the relaxation that occurs when

in
True/False. muscle is excessively stretched arises from a
receptor located in the muscle itself. This
30. Lumbar CSF pressure varies from 70-180 receptor is the Golgi tendon organ.
mm CSF. True / False
9. Impulses in the spinothalamic tracts and the

s.
Answers to Self-Scorer 3: Functions of CNS dorsal column-medial lemniscus pathways reach
1A 2D 3B 4A 5A consciousness. Impulses in the spinocerebellar
6B 7B 8C 9AB 10D pathways terminate in the cerebellar cortex and
11C 12C 13A 14B 15B therefore do not reach consciousness.
16A 17C 18C 19C 20B
10. This is what happens in cortical blindness.
21C
26E
22A
27A

Explanations:
23C
28T
24D
29T iim
25A
30T

1. Indeed, reduced reuptake of excitatory


11. In this condition, hypotonia does not occur
during REM sleep and the person may act out
his dreams. It is treated with benzodiazepines.

neurotransmitters such as glutamate by 14. The nuclear bag fiber is activated whenever
astrocytes in ischemic zones is said to contribute the spindle is stretched; i.e. it detects a change in
to excitotoxicity in stroke. length of the muscle. The nuclear chain fiber, a
4a
slowly adapting receptor, fires even during
2. Diuresis is mediated by an action on kappa maintained stretch; i.e. it detects the absolute
receptors. See Table 4-5 on p. 113, Ch 4, WFG, length of the muscle. See p. 131, Ch 6, WFG,
2005. 2005.

3. When light strikes retina, conformational 15. Steady state neurotransmitter release from
changes in rhodopsin activate rod cells occurs in the dark. Light causes
activation of cGMP - phosphodiesterase,
m

phosphodiesterase, decreasing cGMP, causing


closure of Na channels and hyperpolarization of reducing intracellular cGMP, this resulting in
receptor cells and decreased release of closure of Na channels, hyperpolarization of rod
neurotransmitter. This triggers response in other cells and reduced release of neurotransmitter
neural elements of the retina. See Fig 8-19, p. from rod cells.
159 in WFG 22nd ed.
Ai

17. Dark adaptation requires synthesis of more


6. Since - and -motor neurons are coactivated, rhodopsin.
intrafusal fibers also contract whenever
extrafusal fibers contract. Thus, the central 18. The hypothalamus is the highest center for
portion i.e. the spindle is stretched and fires even integration of autonomic reflexes. Charles
while the muscle is contracting; thus the spindle Sherrington said, the hypothalamus is the head
is capable of responding to rate of change of ganglion of the autonomic nervous system.
stretch, as well as maintained stretch.
20. Mass of the brain = 1400 g

101
Cerebral oxygen consumption per minute = 50 Self-Scorer 4: Endocrinology and
ml
CMRO2 (i.e. oxygen consumption per minute
Reproduction
per 100 g of brain tissue) = 3.5 ml/100 g/min
In each of the following questions, choose the
single best response.
21. Resting cardiac output = 5 L/min
Cerebral blood flow = 750 ml/min (this is 15%
1. JAK-STAT pathways mediate the effects of:
of cardiac output)
A. miracullin
B. aquaporins

in
23. Serotonin may act presynaptically on the
C. gusducins
gate in the dorsal horn of the spinal cord and
D. growth hormone
reduce the efficacy of transmission in the pain
pathway. Enkephalins are also implicated in this
2. Men castrated before puberty grow taller
mechanism. See p. 146, WFG, 2005.
because:

s.
A. release of growth hormone is increased
25. Saccades are sudden jerky movements and
B. of androgen resistance
occur as gaze shifts from one object to the other.
C. estrogen levels are reduced
They are programmed in the frontal eye field (in
D. of insensitivity to IGF-1
the cerebral cortex) and the superior colliculus.

iim
26, 27. Kluver & Bucy observed the effects of
bilateral temporal lobectomy. However, it is
now known that these features could be
reproduced by bilateral resection of the
amygdalae.
3. Patients with sexual precocity are apt to be
dwarf because of:
A. increased levels of androgens
B. the estrogen surge during precocious
puberty
C. deficiency of androgen receptors
D. increased levels of thyroxine
***************************************
4. The satiety-producing hormone is:
4a
A. orexin
B. resistin
C. adiponectin
D. leptin

5. Hypophysectomy does not immediately affect


the secretion of:
m

A. zona glomerulosa
B. zona fasciculata
C. zona reticularis

6. Which hormone also upregulates its receptors


in the adrenal?
Ai

A. Epinephrine
B. Angiotensin II
C. Cortisol
D. Aldosterone

7. In glucocorticoid-remediable aldosteronism:
A. angiotensin II has no effect on aldosterone
synthesis

102
B. angiotensin II stimulates whereas A. 17 hydroxylase
angiotensin III inhibits aldosterone synthesis B. testosterone 5 reductase
C. aldosterone synthase is more sensitive to C. 11 hydroxysteroid dehydrogenase
stimulation by ACTH. D. CYP 450scc
D. administration of glucocorticoids increases
ACTH levels 15. The adrenal medulla is not known to secrete:
A. VIP
8. What is true about development of a 44 XX, B. chromogranin A
SRY+ zygote? C. adrenomedullin

in
A. Female internal and external genitalia D. met-enkephalin
develop
B. Both ovaries and testes develop 16. The hormone that is mainly secreted by the
C. Only ovaries develop adrenal medulla is:
D. Only testes develop A. epinephrine

s.
B. norepinephrine
9. Sertoli cells are not known to produce: C. dopamine
A. inhibin B D. adrenomedullin
B. estrogens
C. androgen-binding protein 17. The major stimulator of PTH release from
D. semenogelin the parathyroid gland is:

junctions between:
A. Leydig cells
B. primary spermatocytes
iim
10. The blood-testis barrier is formed by tight
A. vitamin D
B. a fall in plasma ionized calcium
C. calcitonin
D. a fall in plasma phosphate concentration

C. spermatogonia 18. Thyroid hormones increase oxygen


D. Sertoli cells consumption in all of the following tissues
except:
4a
11. Human menopausal gonadotropin consists A. heart
of: B. skeletal muscle
A. -subunits of hCG C. brain
B. FSH & LH D. liver
C. prolactin
D. recombinant GnRH 19. Which of the following hormones is
lactogenic?
12. The hormone that is essential for the A. Chorionic growth hormone
m

formation of male external genitalia is: B. HCG


A. testosterone C. Oxytocin
B. dihydrotestosterone D. Relaxin
C. Mullerian regression factor
D. SOX factor 20. Which of the following inhibits lactation?
Ai

A. Prolactin
13. Testicular feminization syndrome is caused B. Chorionic somatomammotropin
by mutations in genes coding for: C. Estrogens
A. androgen-binding proteins D. Growth hormone
B. testosterone 5 reductase
C. androgen receptor 21. Which of the following inhibits ovulation?
D. inhibin A. Prolactin
B. Human menopausal gonadotropin
14. The penis-at-12 syndrome occurs due to a C. Estrogens
deficiency of: D. Luteinizing hormone releasing hormone

103
Leptin receptors are coded by db gene
22. Which of the following hormonal Leptin deficiency as well as deficiency of
combinations in plasma is associated with functional leptin receptors have been
normal lactation? implicated in the pathogenesis of obesity.
A. Cortisol + prolactin + oxytocin
B. Estrogen (E) + progesterone (P) + prolactin 5. Although ACTH is trophic to all three zones
C. E + P + insulin + cortisol + prolactin in the adrenal cortex, the major stimulator of
D. Prolactin + oxytocin + E + P aldosterone synthesis is angiotensin II. In the
long run, however, the size of the zona

in
23. During normal pregnancy, which hormone(s) glomerulosa would also decrease.
peak during the first trimester? (select all that
apply) 7. The basic defect in glucocorticoid-remediable
A. HCG aldosteronism is a hybrid aldosterone synthase
B. HCS gene extremely sensitive to stimulation by

s.
C. Prolactin ACTH. Thus, hyperaldosteronism in this
D. Estradiol instance is remediable with glucocorticoid
E. Estriol therapy; i.e., administered glucocortiocoids
F. Relaxin inhibit ACTH release and consequently reduce
aldosterone synthesis.
Answers to Self-scorer 4: Endocrinology &
Reproduction
1D
6B
11B
16A
2C
7C
12B
17B
3B
8D
13BC 14B
18C
4D
9D

19A
iim
5A
10D
15A
20C
8. The sex determining region of the Y
chromosome dictates the formation of
embryonic testes and inhibits the development
of ovaries. Mullerian inhibiting polypeptide
from the testes inhibits the growth of Mullerian
21A 22A 23AF duct structures. Testosterone induces the
development of male internal genitalia.
Answer Explanations:
4a
1. Janus tyrosine kinases (JAK) are enzymes that 13. Testicular feminizing syndrome
phosphorylate signal transducers and activators Genotype 44XY
Gonad Testes
of transcription (STAT). STAT proteins are Testosterone Levels normal or increased
transcription factors. Growth hormone, Internal genitalia Male or female
prolactin, and erythropoietin act via JAK-STAT External genitalia Female
pathways. Phenotype Female
Defects Mutations in genes coding for
either the androgen receptor or
m

2. Men castrated before puberty would have


testosterone 5 reductase
little androgens and consequently estrogens that
This condition is a classic example of male
are formed from testicular androgens. Epiphyses
pseudohermaphroditism. Testosterone levels are
would remain open for a longer time and
increased in this condition. When the basic
consequently such men would be taller.
defect is a mutated androgen receptor, there is
resistance to actions of testosterone and
Ai

4. All you need to know about leptin:


dihydrotestosterone and the resulting clinical
Leptin is a satiety signal from the adipocytes
syndrome is called androgen resistance
acting on the hypothalamus to inhibit food
syndrome. The resistance can vary from mild to
intake.
severe. Note that both testosterone and DHT act
It presumably signals the amount of fat to
upon the same androgen receptor. DHT is much
the CNS.
more potent and is essential for formation of
Chemically, it is a polypeptide. It is coded
male external genitalia. To simplify, DHT
by the ob gene
amplifies the actions of testosterone.
Leptin levels increase after a meal

104
Causes of male pseudohermaphroditism: Self-scorer 5: Gastrointestinal
Androgen resistance syndrome
Testicular feminizing syndrome
Physiology
Cholesterol desmolase deficiency (rare)
1. Normally, instillation of HCl into the
17- hydroxylase deficiency (rare)
duodenum:
A. stimulates gastrin release
14. Pathogenesis of the penis-at-12
B. increases output of an enzyme rich
syndrome:
pancreatic juice
It occurs due to a deficiency of the enzyme
C. increases activity of Brunners glands

in
5- reductase
D. relaxes the pyloric sphincter
Testosterone (T) and DHT act on the same
receptor
2. Which of the following statements regarding
DHT is much more potent than T
the actions of gastrointestinal hormones is
However, high levels of T may overcome
incorrect?

s.
requirement for DHT
A. Secretin inhibits gastric emptying
During puberty the surge in LH induces
B. Gastrin stimulates histamine release in the
synthesis of large amounts of T
stomach
Because of DHT deficiency, feedback
C. Somatostatin reduces blood flow to the GIT
inhibition of LH release is also less.
D. GIP inhibits the release of insulin from
Thus, high levels of T may mimic the
iim
actions of DHT and induce enlargement of
the clitoris.

15. Adrenomedullin is a vasodepressor peptide;


it was first isolated from pheochromocytoma
pancreas

3. Which of the following exemplifies the


potentiating effect of one hormone on another?
A. output of HCO3 in pancreatic juice with
secretin
cells; its physiologic role is unsettled.
B. output of enzymes in pancreatic juice with
CCK
18. T4 and T3 increase the oxygen consumption
C. alkalinity of pancreatic juice with CCK
4a
of all metabolically active tissues. The
than without it
exceptions are the adult brain, testes, uterus,
D. enzyme output in pancreatic juice with
lymph nodes, spleen and anterior pituitary.
secretin than without it
20. Estrogens stimulate breast growth but
4. Gastric emptying is slowest after a meal
antagonize the milk producing effect of
containing:
prolactin; lactation commences once the
A. fat
placenta is expelled and there is an abrupt
m

B. carbohydrate
decline in circulating levels of estrogens and
C. protein
progesterone.
D. indigestible fiber
21. The best answer is prolactin. Prolactin
5. Slow waves in the GIT are initiated by:
inhibits the release of FSH and LH from the
A. I cells
Ai

anterior pituitary and also inhibits the actions of


B. K cells
FSH and LH on the ovary.
C. interstitial cells of Cajal
D. S cells
However, oral contraceptive pills which inhibit
ovulation usually contain a combination of
6. The most alkaline exocrine secretion is:
estrogen and progesterone, and the dose is
A. bile
pharmacologic.
B. pancreatic juice
C. intestinal juice
**************************************
D. saliva

105
A. Glucose
7. The odor of feces is partly due to: B. Amino acids
A. stercobilinogen C. Galactose
B. indole, skatole and sulfides D. Fructose
C. primary bile acids
D. secondary bile acids 15. Which of the following cells sample the
antigenic milieu of the gastrointestinal lumen?
8. The normal rate of bile salt synthesis is: A. P cells
A. 0.2 0.4 g/day B. I cells

in
B. 0.05 0.1 g/day C. K cells
C. 0.8 1.2 g/day D. M cells
D. 2 2.5 g/day
16. The defecation reflex is integrated in the:
9. The Oral Rehydration Solution is helpful in A. brain stem

s.
rehydration in diarrheas because: B. pons
A. Na-glucose symporter is unaffected in C. medulla
diarrheas D. spinal cord
B. 1Na-1K-2Cl symporter is located only in the
basolateral membrane Answers to self-scorer 5 in Gastrointestinal
C. cAMP regulated Cl secretion is reduced Physiology
iim
D. toxins inhibit facilitated glucose transport

10. Which of the following enzymes is not


synthesized in the pancreas?
A. DNAase
1C
6B
11A
16D
2D
7B
12B
3C
8A
13C
4A
9A
14D
5C
10D
15D

B. Proelastase Answer Explanations:


C. Aminopeptidases 1. The acidity of duodenum drives the release of
D. Pepsin secretin thereby evoking alkaline secretions
4a
from the liver, pancreas and intestine.
11. The basal acid output /maximum acid output
ratio is normally less than: 2. Secretin and CCK slow gastric emptying by
A. 0.1 contracting the pyloric sphincter. Gastrin from G
B. 0.6 cells stimulates acid secretion by two means:
C. 0.8 one, it stimulates histamine release from mast
D. 0.9 cells in stomach; two, it directly acts on parietal
m

cells to augment acid secretion. Somatostatin


12. There is a net secretion of which ion in the reduces intestinal blood flow and secretion.
colon?
A. Na 3. Secretin and CCK potentiate each others
B. K actions on pancreatic acinar cells. The primary
C. Cl effect of secretin is to increase HCO3 output in
Ai

D. Ca pancreatic juice, bile and intestinal secretion. In


addition, the enzyme secretory response of
13. The rate-limiting step in the metabolism of pancreatic acinar cells to CCK is much better in
bilirubin is: the presence of secretin than its absence.
A. uptake by ligandin
B. conjugation with glucuronic acid 12. The secretion of K in the colon is stimulated
C. secretion into bile by aldosterone. Functional villous adenomas in
the colon may cause hypokalemia. Similarly, the
14. Which of the following is absorbed in the diarrhea of VIPoma may be accompanied by
intestine independent of sodium? hypokalemia.

106
Self-Scorer 6: Cardiovascular
15. M cells (microfold cells) are antigen-
presenting cells in the GI mucosa. They sample
Physiology
the antigenic milieu of the GIT and pass them on
1. What is the primary ionic basis of the
to T-cells in the Peyers patches.
prepotential in the SA node?
A. Ca influx through transient T Ca channels
***************************** B. Inwardly directed long-lasting Ca current
C. Outward Na current
D. Potassium efflux through leak channels

in
2. Normally, the impulse that excites the left
ventricular myocardium originates in the:
A. SA node
B. Purkinje system

s.
C. left bundle branch
D. ventricle

3. Conduction speed is slowest in the:


A. SA node
iim B. atrial pathways
C. bundle of His
D. Purkinje system

4. The propagation of repolarization from the


ventricular epicardium to endocardium is
represented by the
A. QRS complex
B. QT interval
4a
C. T wave
D. TP period

5. T wave inversion occurs when ventricular


repolarization occurs from:
A. endocardium to epicardium
B. epicardium to endocardium
m

C. apex to base of the heart


D. base to apex of the heart

6. The most reliable index of AV nodal delay is:


A. PR interval
B. PR segment
Ai

C. AH interval
D. PA interval

7. Cardiac muscle cannot be tetanized because


of:
A. accommodation
B. its slow rate of repolarization
C. calcium influx during phase II

107
D. voltage inactivation of Na channels at B. Increase in plasma level of renin
membrane potentials < 80 mV C. Increase in plasma level of ANP

8. When you stand up, pressure falls in: 15. Which of the following statements is
A. carotid arteries incorrect?
B. brachial arteries A. Pulse pressure is directly proportional to
C. lower limb arteries stroke volume
D. all arteries B. Pulse pressure is directly proportional to
large artery distensibility

in
9. If QRS deflection is highest and upright in C. Reflected arterial pulse waves normally
lead I and equiphasic (or null) in lead aVF, then serve to increase coronary perfusion during
mean electrical axis of the QRS vector in the diastole
frontal plane is about: D. In the elderly, higher pulse pressure is
A. 30 degrees associated with increased risk of morbidity

s.
B. 0 degrees
C. +45 degrees 16. Which of the following is most likely to
D. +90 degrees happen in a normal healthy adult seated in a
centrifuge and spun at a force equal to 2 times
10. What would be the change in blood flow to a that of gravity?
tissue if radius of the arterioles in that tissue is A. Increase in intracranial tension

A. Increase 8 times
B. Increase 16 times
C. Increase 4 times
D. Decreases 4 times
iim
doubled and perfusion pressure is halved? B. Increase in BP
C. Increase in cardiopulmonary blood volume
D. A doubling of pressure in the veins of the
foot

17. The blood-brain barrier is formed by:


11. Recombinant brain natriuretic peptide A. tight junctions between vascular endothelial
(nesiritide) is currently used in the management cells in the cerebral capillaries
4a
of: B. choroidal epithelial cells
A. bronchial asthma C. ependymal cells
B. congestive heart failure D. foot processes of oligodendroglia
C. bilateral renal artery stenosis
D. ACE inhibitor induced cough Answers to self-scorer 6 in
Cardiovascular Physiology
12. Maximal oxygen consumption (VO2 max) in 1A 2A 3A 4C 5A
healthy active men is about
m

6C 7B 8AB 9B 10A
A. 10 ml/kg/min 11B 12C 13C 14B 15B
B. 20 ml/kg/min 16D 17A
C. 40 ml/kg/min
D. 80 ml/kg/min Answer Explanations:
1. See p 548, Ch 28, WFG, 2005.
Ai

13. The last ditch stand in defense of a falling


blood pressure is the: 2. Normally, it is the impulse that originates in
A. arterial baroreflex mechanism the SA node that excites the ventricular
B. arterial chemoreflex mechanism myocardium. Such a rhythm is called sinus
C. CNS ischemic pressor response rhythm.
D. Bainbridge reflex
3. There are two regions in the heart where the
14. Which of the following does not occur when inhibitory effect of vagal stimulation on
the body is immersed in water up to the neck? conduction speed is profound. These are the SA
A. Increase in central venous pressure

108
and AV nodes. The conduction speed in the SA
and AV nodes is about 0.05 m/s. See p. 549, Ch 12. VO2 max is the maximum amount of oxygen
28, WFG, 2005. Compared to the fast that can be utilized during dynamic exercise; it
conducting Purkinje system, this is about 80 can be increased by training. See p 635, Ch 33,
times slower. The conduction speed is slowed WFG, 2005.
even further in disease conditions such as SA
node exit block and AV nodal block. 13. Arterial BP is affected by numerous neural
and humoral mechanisms. The arterial
4-5. Depolarization and repolarization are baroreflex mechanism buffers BP fluctuations

in
electrically opposite processes. Normally, when mean arterial pressure (MAP) is in the 70-
ventricular depolarization occurs from 150 mm Hg range. But when MAP is lower than
endocardium to epicardium, and repolarization 70 mm Hg, baroreceptors are maximally
occurs from epicardium to endocardium. This is deactivated and the arterial chemoreflex
why the T wave is upright, i.e. in the same mechanism assumes greater importance in

s.
direction as QRS. An inverted T wave indicates defending against a further fall in BP.
that the direction of ventricular repolarization is
reversed, i.e. occuring from endocardium to In severe hypotension such as when mean
epicardium. arterial pressure is < 50 mm Hg), ischemia of the
vasomotor center in the medulla triggers a
6. PA interval (approximately 30 ms) reflects the powerful increase in sympathetic outflow and
iim
time taken for conduction across the atrial
pathways to the AV node. PR segment is a
combined index of AV nodal delay and
conduction in infranodal pathways. PR interval
is a rather poor index of AV nodal delay. AH
MAP. This mechanism, the CNS ischemic
pressor response (called the last ditch stand in
defense of a falling BP), contributes to restoring
cerebral blood flow.

interval, which is obtained by a His bundle Reflex mechanism Mean arterial


electrogram specifically reflects AV nodal pressure (mm Hg)
delay. See p. 553-554, Ch 28, WFG, 2005. range in which it
4a
operates
7. The long duration of action potential (or the Arterial baroreflex 70110
absolute refractory period) in the heart is due to Arterial chemoreflex 4070
slow repolarization. This is a safety features that CNS ischemic < 50
ensures that the ventricles relax to fill with blood response
before contracting again.
14. When a person is immersed in water up to
10. Tissue blood flow = BP / local vascular
m

the neck, this functions as an anti-G suit (anti G


resistance. Blood flow is directly proportional to suits are filled with water or compressed air)
the fourth power of radius. thereby mitigating the effect of gravity. Thus,
venous return and central blood volume
11. ANP is a natriuretic hormone. It increases increase. The rise in central venous pressure
blood flow through the kidneys and glomerular (CVP) may stretch the ventricles and lead to
Ai

filtration rate. ANP levels are increased in heart increased release of ANP, a natriuretic hormone.
failure. The action of ANP in heart failure is to A rise in CVP due to an increase in total blood
increase Na and water excretion by the kidneys. volume is usually accompanied by a reflex
At first sight, this would appear to be a decrease in renin release from the kidneys.
beneficial effect; however, it must be noted that
in heart failure, effective arterial blood volume 15. The stiffer the arteries, i.e., the less
is diminished. Angiotensin II, aldosterone and distensible they are, higher the pulse pressure for
the renal vasoconstrictive effects of elevated a given stroke volume. This is the basis of
activity in the renal sympathetic nerves higher systolic pressure in isolated systolic
counteract the natriuretic effects of ANP. hypertension, the commonest type of

109
hypertension in the elderly.

16. The effect of positive g forces, in an


Self-Scorer 7: Pulmonary
individual standing upright, would be to throw Physiology
blood into the lower part of the body.
Choose the single best answer
17. While foot processes of astrocytes end on
cerebral capillaries and induce the formation of 1. The major area of airway resistance during
tight junctions, the actual anatomic basis of the breathing is located in:

in
blood brain barrier is the tight junctions between A. oropharynx
endothelial cells in cerebral capillaries. In the B. trachea and large bronchi
circumventricular organs such as area postrema, C. intermediate-sized bronchi
subfornical organ, this barrier is absent. D. bronchioles < 2 mm in diameter

2. Pulmonary capillary wedge pressure must be

s.
***************************************
estimated under:
A. zone 1
B. zone 2
C. zone 3
D. zone 4 conditions
iim 3. Cardiogenic pulmonary edema occurs when
pulmonary capillary pressure exceeds:
A. 10 mm Hg
B. 15 mm Hg
C. 20 mm Hg
D. 25 mm Hg
4a
4. Apneusis occurs typically after:
A. damage to phrenic motor neurons
B. damage to pneumotaxic center
C. lesions of pneumotaxic center and vagotomy
D. denervation of peripheral chemoreceptors

5. Which one of the following has been recently


postulated to function as a sensor of oxygen
m

levels and a facilitator of oxygen transport in the


brain?
A. Myoglobin
B. Neuroglobin
C. Nitric oxide
Ai

D. Cytochrome oxidase c

6. Life is impossible without pressurization at an


altitude greater than:
A. 12000 m
B. 14000 m
C. 17000 m
D. 19000 m

110
7. The altitude at which body fluids boil is
about: 14. The amount of oxygen dissolved in plasma
A. 8800 m relates most closely with:
B. 12400 m A. FiO2
C. 14500 m B. PAO2
D. 19000 m C. PaO2
D. Blood [Hb]
8. Which of the following occurs as a
consequence of increased barometric pressure? Answers to Self-scorer 7: Pulmonary

in
A. Rapture of the deep Physiology
B. Black out 1C 2C 3D 4C 5B
C. Bends 6B 7D 8A 9B 10C
D. Air embolism 11B 12C 13D 14C 15C

s.
9. Deep sea divers have been known to offer Answer Explanations:
their mouths to fish! This is because: 1. This is because of the high airflow velocity in
A. of low fat solubility of nitrogen these segments. Bronchioles less than 2 mm in
B. of nitrogen toxicity diameter represent less than 10% of total airway
C. helium impairs intellectual functions resistance. See p 282, NMS Physiology, Bullock
D. of increased work of breathing

A. Compressed room air


B. 100% oxygen
iim
10. Which of the following is used during deep
sea diving?
J et al, 2001.

3. Normally, pulmonary capillary hydrostatic


pressure is about 10 mm Hg. It is considered
elevated when PCWP exceeds 18 mm Hg. The
osmotic pressure of plasma proteins is normally
C. A heliumoxygen mixture about 25 mmHg.
D. 50% N2 and 50% oxygen
4. The effect of afferent vagal input from stretch
4a
11. In which of the following situations does receptors in the lungs is to inhibit inspiration.
decompression sickness not occur? The pneumotaxic center in the pons serves to
A. During ascent in an unpressurized cabin of switch from inspiration to expiration. Thus,
an airplane after vagotomy alone, the depth of respiration
B. At a depth of 100 ft in the ocean would be higher than normal. However, after
C. During rapid ascent from a deep-sea dive damage to both pneumotaxic center as well as
vagotomy, prolonged inspiratory spasms that
12. What is the treatment of choice for air
m

resemble breath holding (apneusis) occur. See p


embolism? 672, Ch 36, WFG, 2005.
A. Continuous positive airway pressure
B. Gradual decompression 6. At an altitude of 14000 meters, consciousness
C. Hyperbaric oxygenation is lost despite administration of 100% oxygen.
D. Positive pressure ventilation This is because the barometric pressure at this
Ai

altitude is far too low to permit adequate


13. Which of the following combination of oxygenation of arterial blood. See p. 689, Ch 37,
arterial blood gas results is most likely in a WFG, 2005.
normal person after a months residence at 4000-
meter altitude? Arterial pH, PaCO2 and HCO3 9. The euphoria is attributed to increased
(mmol/L) respectively would be: solubility of nitrogen in body fluids. This is why
A. 7.4. 40 mmHg, 24 a helium-oxygen mixture is used and while
B. 7.36, 36 mm Hg, 30 increased solubility of helium is associated with
C. 7.6, 20 mm Hg, 40 impairment of manual dexterity, intellectual
D. 7.46, 26 mm Hg, 19

111
function is apparently not significantly impaired.
See p. 694-5, Ch 37, WFG, 2005.
Self-scorer 8: Renal and Acid-Base
10. If a diver breathes compressed air, the Physiology
increased partial pressure of nitrogen can result
in a constellation of neurological symptoms 1. Which of the following substances cannot be
known as rapture of the deep. See p 694-5, Ch used for measuring GFR?
37, WFG, 2005. A. Glucose
B. Sucrose

in
11. At a depth of 100 ft in the ocean, barometric C. Mannitol
pressure is about 4 atmospheres, four times D. Lactose
greater than at mean sea level. The problems that
occur are therefore due to increased solubility of 2. If the glomerular capillary hydrostatic
gases in blood and depend on the gas mixture pressure, osmotic pressure of plasma proteins,
hydrostatic pressure in the Bowmans space and

s.
used for breathing. You may want to remember
this as compression sickness. In contrast, oncotic pressure in the interstitium are
decompression sickness (the exact opposite of respectively 40, 25, 5, and 0 mm Hg
compression sickness) occurs during rapid respectively, what is the net pressure driving
ascent following a deep-sea dive or when an filtration of fluid into the Bowmans space?
airplane cabin loses pressure at high altitude. A. 10 mm Hg
iim
Gases dissolved in body fluids come out of
solution because of the reduction in ambient
pressure, forming bubbles and blocking blood
flow to tissues.
B. 15 mm Hg
C. 20 mm Hg
D. 25 mm Hg

3. In a normal adult human on an average diet,


14. By applying high pressure, solubility of gas which one of the following does not usually
(air emboli in this question) in plasma is appear in the urine?
increased. At a constant temperature, solubility A. Calcium
4a
of a gas in plasma is directly proportional to B. Urea
partial pressure of gas (Henrys law). C. Uric acid
D. HCO3
***************************************
4. Normally the ratio of PaCO2 and plasma
HCO3 is:
A. 1.2
B. 1.4
m

C. 1.6
D. 1.8

5. If PaCO2 is 80 mm Hg and arterial plasma


[H+] is 80 nmol/L, then plasma [HCO3] is:
Ai

A. 40 mmol/L
B. 28 mmol/L
C. 24 mmol/L
D. 10 mmol/L

6. In the above situation (Question 5), what is


the acid-base status?
A. Compensated metabolic alkalosis
B. Uncompensated respiratory acidosis
C. Compensated respiratory acidosis

112
D. Uncompensated metabolic acidosis 9. One must understand that it is possible that
E. Normal acid-base status there might be an acid base disturbance although
the pH of arterial plasma may be 7.4 (normal).
7. In arterial blood gas analysis, which one of For example, there might be intracellular
the following is calculated? acidosis before arterial pH drops. See
A. Arterial pH Balakrishnan et al. What is the ultimate goal in
B. Plasma bicarbonate acid-base regulation?
C. PaCO2 http://advan.physiology.org/cgi/content/full/31/1/51
D. PaO2

in
8. Which of the following is not elevated in
ethylene glycol poisoning?
A. Anion gap
B. Base excess

s.
C. Osmolar gap
D. Serum osmolality

9. The severity of acidosis is related to:


A. anion gap
B. pH of arterial plasma
C. pH of ICF and plasma
D. plasma HCO3

Answers:
1A 2A 3D 4C
iim 5C
6B 7B 8B 9C

Answer Explanations:
4a
1. Glucose is normally not excreted by the
nephron. So it cannot be used to measure GFR.

2. Recall Starlings equation describing fluid


movement across capillaries (see page 10)
In this instance, the pressure gradient for
filtration across the glomeruli is (40 + 0) (25 +
5) = 10 mm Hg.
m

4. Normally, PaCO2 = 40 mm Hg, and plasma


HCO3 = 24 mM and PaCO2 / [HCO3] = 1.6. If
this were the case, then the [H+] of arterial
plasma would be equal to 40 nM (corresponding
Ai

to an arterial pH of 7.4) as predicted by the


modified Henderson equation (see p. 86).

5,6. Using the modified Henderson equation, we


get, plasma HCO3 = 24 mM. This is
uncompensated respiratory acidosis because
there is no compensatory increase in bicarbonate
levels.

113
A BLAST FROM THE PAST: C. Ca
D. Cl
With few exceptions, most of the questions in
this chapter have appeared in PG entrance
6. Amongst plasma proteins, albumin makes the
examinations, but I have made several editorial
greatest contribution to the colloid osmotic
changes (mostly minor) to the stem and the
pressure of plasma proteins because:
answer choices in many of them. Needless to
A. albumin has a very high molar mass, and its
say, it is good to use these questions to guide
concentration in plasma is high
your preparation for future exams.
B. albumin has a very low molar mass, and its

in
concentration in plasma is high
Choose the single best answer
C. albumin has a very low molar mass, and its
concentration in plasma is low
1. A substance injected intravenously was found
D. albumin has a very high molar mass, and its
to be distributed through 30% of total body
concentration in plasma is high
water (TBW). It probably:

s.
A. did not pass through blood capillaries
7. The osmotic pressure of crystalloids is not
B. was distributed evenly throughout body
included in the Starlings equation describing
water
fluid exchange across capillaries because:
C. did not enter cells
A. they were discovered after Starling
D. was excluded from CSF
described the forces

A. 0.4%
B. 7.6%
C. 10.4%
iim
2. What fraction of total body potassium is
present in plasma?
B. crystalloids carry electric charges
C. crystalloids diffuse freely across the
capillary endothelium
D. the quantity of crystalloids in plasma is
negligible compared to plasma proteins
D. 89.5%
8. A pole vaulter fell during pole vaulting and
paralyzed his arm. Which of the following
3. A solution contains 1 gram mole of
4a
investigations is most useful to estimate
magnesium sulfate per liter. Assuming full
likelihood of recovery?
ionization, calculate the osmotic pressure of the
A. Electromyography
solution. (1 mOsm/L exerts an osmotic pressure
B. Muscle biopsy
of 19.3 mm Hg)
C. Strength-duration curve
A. 19.3 mm Hg
D. CPK levels
B. 3.86 mm Hg
C. 19300 mm Hg
9. Synaptic conduction is mostly orthodromic
m

D. 38600 mm Hg
because:
E. 57900 mm Hg
A. dendrites cannot be depolarized
B. once repolarized, an area cannot be
4. If the potassium chloride concentration of a
depolarized.
solution is 39 mg %, then, what is the millimolar
C. the strength of antidromic impulses is less.
concentration of potassium in this solution?
Ai

D. chemical mediator is located only in the


A. 1
presynaptic terminal.
B. 3.9
C. 10
10. The main excitatory neurotransmitter in the
D. 100
CNS is:
A. glycine
5. Emiocytosis requires an increase in the
B. acetylcholine
intracellular concentration of:
C. aspartate
A. Na
D. glutamate
B. K

114
11. A 10C decrease in body temperature 18. Which combination of the following
decreases cerebral metabolic rate (i.e., cerebral statements is correct with reference to hypoxia?
consumption of oxygen) by: 1. When it is severe, causes stimulation of the
A. 10% sympathetic nervous system
B. 30% 2. It leads to the accumulation of hydrogen and
C. 50% lactate ions in tissues
D. 70% 3. It causes a decrease in cerebral blood flow
4. If it is chronic, causes a rightward shift of
12. In spinal anesthesia, which of the following oxyhemoglobin dissociation curve

in
is lost first?
A. Sympathetic tone A. All of the above statements are correct
B. Sensation in the lower limbs B. 2&3
C. Motor function C. 1, 2 & 4
D. Proprioception D. 2, 3 & 4

s.
13. Feedforward inhibition occurs in the: 19. An anterolateral cordotomy relieving pain in
A. basal ganglia the right leg is effective because it interrupts the:
B. thalamus A. left dorsal column
C. cerebellum B. left ventral spinothalamic tract
D. cerebral cortex C. left lateral spinothalamic tract

A. Increase in cGMP
iim
14. Which of the following changes occur in rod
cells when rhodopsin is activated by light?

B. Deactivation of phosphodiesterase
D. right lateral spinothalamic tract

20. A lesion of the ventrolateral part of the


spinal cord will lead to loss (below the level of
the lesion) of:
C. Depolarization of rod cells A. pain sensation on the ipsilateral side
D. Decreased release of neurotransmitter B. proprioception on the contralateral side
C. pain sensation on the contralateral side
4a
15. During the dark phase of the visual cycle, D. proprioception on the ipsilateral side
which form of vitamin A combines with opsin to
make rhodopsin? 21. Which of the following states do beta
A. All trans retinaldehyde waveforms in electroencephalogram designate?
B. All trans retinol A. Deep anesthesia
C. 11-cis retinaldehyde B. Surgical anesthesia
D. 11-cis retinol C. Light anesthesia, eyes closed, relaxed
D. Awake, alert state
m

16. Relative color and luminosity of


photoreceptive input under changing light 22. Which one of the following is least likely to
conditions are regulated and maintained by: aggravate insult in injured brain?
A. Muller cells A. Hypercapnia
B. amacrine cells B. Hypoxia
Ai

C. ganglion cells C. Hypotension


D. retinal astrocytes D. Hypothermia

17. Which of the following phenomena is 23. Cushings triad does not include:
closely associated with slow wave sleep? A. hypertension
A. Dreaming B. bradycardia
B. Atonia C. hypothermia
C. Sleep walking D. irregular respiration
D. Irregular heart rate

115
24. Cell bodies of orexigenic neurons are present D. conduction loss in the left ear
in:
A. locus ceruleus 30. Adrenaline, noradrenaline and dopamine act
B. dorsal raphe nucleus upon membrane receptors that span the
C. lateral hypothalamic area membrane:
D. hippocampus A. 2 times
B. 5 times
25. Exposure to darkness leads to increased C. 7 times
melatonin secretion. It is brought about by: D. 10 times

in
A. decreasing the activity of suprachiasmatic
nucleus 31. In which form does ADH circulate in
B. increasing activity of serotonin N-acyl plasma?
transferase A. Bound to neurophysin I
C. decreasing the hydroxy-indole-O-methyl B. Bound to neurophysin II

s.
transferase activity. C. Bound to plasma albumin
D. blocking the release of norepinephrine from D. Free form
sympathetic nerve terminals.
32. Which of the following is least dependent or
26. Stimulation of sympathetic nerves to the independent of the production of maternal or
pineal gland: fetal glucocorticoids?
A. increases synthesis of melanin

transferase
C. reduces melatonin synthesis
D. increases release of serotonin
iim
B. increases activity of serotonin N-acetyl
A. Induction of thymic involution
B. Production of surfactant
C. Thyroid function
D. Function of hypothalamo-pituitary axis

33. Before the onset of puberty, GnRH neurons


27. Which of the following statements regarding are under the inhibitory control of:
the thermic effect of feeding is incorrect? A. glycine
4a
A. It is most prominent at the time of eating B. glutamate
B. It is synonymous with specific dynamic C. GABA
action of food D. beta-endorphin
C. It is greater following a protein meal
D. It is enhanced by sympathetic neural activity 34. The positive feedback effect of estrogen in
inducing LH surge is associated with one of the
28. The first physiologic response to high following steroid hormone ratios in the
environmental temperature is: peripheral circulation:
m

A. sweating A. High estrogen: low progesterone


B. cutaneous vasodilation B. Low estrogen: high progesterone
C. decreased heat production C. Low estrogen: low progesterone
D. nonshivering thermogenesis D. High estrogen: high progesterone
Ai

29. A 38-year old gentleman reports decreased 35. Sertoli cells have receptors for:
hearing in the right ear for the last 2 years. On A. inhibin
testing with a 512 Hz tuning fork, the Rinnes B. LH
test (without masking) is negative on the right C. FSH
ear and positive on the left side. With the D. Melatonin
Webers test, the tone is perceived as louder in
the left ear. The probable diagnosis is: 36. In a young female of reproductive age with
A. conduction loss in the right ear regular menstrual cycles of 28 days, ovulation
B. sensorineural hearing loss in the right ear occurs around the 14th day. When is the first
C. sensorineural hearing loss in the left ear polar body extruded?

116
A. 24 h prior to ovulation
B. At the time of ovulation 42. The conjugation of bilirubin with glucuronic
C. 48 h after the ovulation acid in the liver:
D. At the time of fertilization A. converts a hydrophilic compound to a
hydrophobic molecule
37. A baby girl presents with bilateral inguinal B. converts a hydrophobic molecule to a
masses, thought to be hernias, but are found to hydrophilic molecule
be testes in the inguinal canals. Which karyotype C. enables bilirubin to cross the cell membrane
would you expect to find in the child? D. is increased during neonatal jaundice

in
A. 46 XX
B. 46 XY 43. Removal of the liver is fatal because:
C. 47 XXY A. blood urea rises
D. 47 XYY B. jaundice develops
C. clotting time is prolonged

s.
38. The enzyme associated with the conversion D. progressive hypoglycemia occurs
of androgen to estrogen in the growing ovarian
follicle is: 44. CD-95 has a major role in:
A. desmolase A. apoptosis
B. isomerase B. cell necrosis
C. aromatase C. interferon activation
D. hydroxylase

A. testosterone
B. dehydropepiandrosterone
iim
39. The biologically most active androgen is:
D. proteolysis

45. Erythropoiesis is inhibited by:


A. ACTH
B. thyroxine
C. androstenedione C. estrogen
D. dihydrotestosterone D. prolactin
4a
40. The laboratory report shows values of 46. Vagal stimulation is least likely to:
gonadotropins and ovarian hormones of the A. increase intestinal secretion
blood sample taken on the 20th day of the B. contract smooth muscle of intestine
menstrual cycle of a young woman. Whether her C. cause bronchodilation
cycle was ovulatory or not may be validly D. lead to a drop in blood pressure
assessed by measuring serum levels of:
A. FSH 47. Saccades are
B. LH A. voluntary, slow eye movements
m

C. estradiol B. involuntary, slow eye movement


D. progesterone C. abrupt, involuntary, slow eye movements
D. abrupt, voluntary, rapid eye movements
41. Apoptosis occurs as a result of changes in
hormone levels during the ovarian cycle. When 48. The consequences of Rh incompatibility are
Ai

the ovum is not fertilized, endometrial cells die not serious during the first pregnancy because:
because: A. antibodies are not able to cross placenta
A. the involution of corpus luteum causes B. antibody titer is very low during primary
estradiol and progesterone levels to fall immune response
dramatically. C. IgG is ineffective against fetal red cells
B. LH levels rise after ovulation. D. massive hemolysis is compensated by
C. estradiol levels are not involved in the LH increased erythropoiesis
surge phenomenon.
D. estradiol inhibits the induction of the
progesterone receptor in the endometrium.

117
49. Although more than 400 blood groups have D. The path of spread of excitation was normal.
been identified, ABO blood group system is
most important in clinical medicine because: 53. A dye ABC has been in use for the
A. it was the first blood group system to be measurement of blood volume and cardiac
discovered. output. This was rivaled by the introduction of
B. it has four different groups viz A, B, AB and XYZ, which crossed the capillaries. What
O difference would the use of XYZ make in this
C. ABO antigens are present in most body context?
tissues and fluids. A. No change in measured cardiac output &

in
D. ABO antibodies are invariably present in blood volume
plasma when the persons RBC lacks the B. Alteration in both values
corresponding antigen. C. Increase in measured blood volume alone
D. Increase in measured cardiac output alone
50. A 55-year-old male accident victim in the

s.
ED urgently requires a transfusion. His blood 54. The most recent technique for noninvasive
group could not be determined as his red cell measurement of cardiac output is:
group and plasma group did not match. A. pulmonary artery catheterization
Emergency transfusion should be done with: B. thermodilution
A. RBC corresponding to his red cell group and C. echocardiography
colloids and crystalloids D. impedance cardiography

group.

crystalloids
D. AB negative blood
iim
B. Whole blood corresponding to his plasma

C. O positive RBC and colloids and


55. A cardiologist asked his lab technician to
determine the systolic time intervals of a 60 yr
old patient with a recent anterior wall infarction.
The technician said that the pulse transducer was
not working. Which of the following could he
51. Stimulation of sympathetic nerves to the then have not determined?
heart decreases: A. QS2
4a
A. heart rate B. Left ventricular ejection time (LVET)
B. force of cardiac contraction C. Pre-ejection period (PEP)
C. speed of conduction D. Both LVET and PEP
D. refractory period
56. Loss of 500 ml of blood over 30 minutes
52. Which of the following statements represent will lead to:
the most correct interpretation of the ECG A. an increase in HR, fall in BP
shown below? B. a fall in BP and HR
m

C. a prominent increase in HR and BP


X beat D. a slight increase in HR

57. Two students, Vineet and Kamlesh were


asked to demonstrate in dogs the role of sinus
Ai

nerve in hypovolemic shock. Vineet severed the


sinus nerve when the mean blood pressure was
85 mm Hg and Kamlesh cut the sinus nerve
when the mean arterial pressure (MAP) was 60
mm Hg. On cutting the sinus nerve:
A. Vineet recorded an in MAP but Kamlesh
A. X beat originated from an atrial focus recorded a
B. X beat reset the cardiac rhythm B. Vineet recorded a in MAP but Kamlesh
C. Both heart sounds would have been present recorded an
at X beat C. both recorded an in MAP

118
D. both recorded a in MAP. A. Brain
B. Heart
58. As part of a space-research program, a C. Viscera
physiologist was asked to investigate the effect D. Skin
of flight-induced stress on blood pressure.
Accordingly, the blood pressures of the 63. Statement: Blood is a non-Newtonian fluid.
cosmonauts were to be measured twice: once Reason: Its viscosity is constant at any flow
before take-off and once after the spacecraft velocity.
entered the designated orbit around the earth. A. Both (S) and (R) are false

in
For a proper comparison, the pre-flight blood B. (S) is true and (R) is false
pressure should be recorded in (the): C. Both (S) and (R) are true but (R) does not
A. lying down position explain (S)
B. sitting position D. Both (S) and (R) are true and (R) explains
C. standing position (S)

s.
D. any position as long as the post-flight
recording is made in the same position. 64. While introducing the Swan Ganz catheter,
its placement in the pulmonary artery (PA) is
59. During exercise, increase in O2 uptake by identified by the following pressure tracing:
exercising muscles occurs due to all of the A. Diastolic pressure is lower in PA than in
following mechanisms except: right ventricle

the left
iim
A. oxyhemoglobin dissociation curve shifts to

B. decrease in vascular resistance in exercising


skeletal muscle
C. O2 extraction
B. Diastolic pressure is higher in PA than right
ventricle
C. PA pressure tracing has dicrotic notch from
closure of pulmonic valve
D. Right ventricular pressure tracing for plateau
D. blood flow and sharp drop in early diastole.

60. In hemorrhaged dogs with marked 65. Which of the following is usually associated
4a
hypotension (mean arterial pressure < 50 mm with turbulence in blood flow?
Hg), denervation of arterial chemoreceptors A. Reynolds number less than 2000
would: B. Decrease in blood flow velocity
A. increase BP since chemoreceptors reduce C. Decrease in density of blood
sympathetic outflow D. Increase in diameter of blood vessel
B. produce no change in BP since
chemoreceptors do not influence 66. Filtration at the arterial end of capillary
sympathetic outflow occurs mainly due to:
m

C. result in a further fall in BP since the arterial A. hydrostatic pressure in capillaries


chemoreflex is sympathoexcitatory B. hydrostatic pressure in interstitium,
D. depend on whether arterial baroreceptors are C. oncotic pressure in capillaries
reset or not D. oncotic pressure in interstitium
Ai

61. Patients with acute cardiac failure may not 67. Intrapleural pressure is normally negative
have edema if: because:
A. oncotic pressure of plasma proteins is high A. intrapulmonary pressure is always negative
B. renal compensation occurs B. chest wall and lungs recoil in opposite
C. cardiac output is decreased directions
D. there is a fall in systemic capillary C. surfactant prevents lung collapse
hydrostatic pressure D. transpulmonary pressure determines the
negativity
62. In which of the following organs is the flow
least under sympathetic control?

119
68. Fetal hemoglobin has all of the following D. 100 mm Hg
characteristics except that:
A. it has strong affinity for 2,3BPG 75. A traveling nerve impulse does not
B. its ODC is shifted to left depolarize the area immediately behind it
C. at low PO2, it gives up more oxygen to because:
tissues than adult hemoglobin A. it is hyperpolarized
B. it is refractory
69. The normal value of P50 on the C. it is not self-propagating
oxyhemoglobin dissociation curve in an adult is: D. the conduction is always orthodromic

in
A. 1.8 kPa
B. 2.7 kPa 76. The renal plasma flow (RPF) of a patient
C. 3.6 kPa was to be estimated through the measurement of
D. 4.5 kPa para-amino hippuric acid (PAH) clearance. The
technician observed the procedures correctly but

s.
70. Which of the following conditions leads to due to an error in weighing inadvertently used
tissue hypoxia without alteration of oxygen thrice the recommended dose of PAH. The RPF
content of blood? estimated thus is likely to be:
A. Carbon monoxide poisoning A. falsely high
B. Methemoglobinemia B. falsely low
C. Cyanide poisoning C. high or low depending on GFR
D. Respiratory acidosis iim
71. The arterial blood gas values, pH 7.58, PCO2
23 mm Hg, PO2 300 mm Hg and oxygen
saturation of hemoglobin 60% are most
D. correct and is unaffected by the overdose of
PAH

77. Several hormones regulate the tubular


reabsorption of water and electrolytes at
consistent with a diagnosis of: different sites in the nephron. Which of the
A. carbon monoxide poisoning following combinations is correct?
B. ventilatory failure A. Angiotensin II acts in the DCT
4a
C. voluntary hyperventilation B. Aldosterone acts in the collecting ducts
D. methyl alcohol poisoning C. ADH acts in the PCT
D. ANP acts in the loop of Henle
72. Intense stimulation of J receptors results in:
A. tachypnea 78. A modified neuroepithelial cell is not the
B. hypoxemia sensory receptor in the:
C. pulmonary edema A. visual pathway
D. apnea followed by tachypnea B. auditory pathway
m

C. gustatory pathway
73. Inflation of the lungs induces further D. olfactory pathway
inflation. This is the:
A. Hering-Breuer inflation reflex 79. The hyperkinetic features of the
B. Hering-Breuer deflation reflex Huntington's disease are due to the loss of
Ai

C. Heads paradoxical reflex neurons in the:


D. J-reflex A. nigrostriatal dopaminergic system
B. intrastriatal cholinergic system
74. One intern calculated the concentration of O2 C. GABAergic and dopaminergic system
in blood as 0.0025 ml/ml of blood. Considering D. intrastriatal GABAergic and cholinergic
atmospheric pressure as 760 mm Hg, what is the system
approximate oxygen tension of arterial blood?
A. 40 mm Hg 80. Epiphysial closure in humans is caused by:
B. 60 mm Hg A. androgens
C. 80 mm Hg B. estrogens

120
C. growth hormone 88. Rod cell excitation by photons results in the
D. thyroxine conversion of:
A. 11-cis-retinal to 11-trans-retinal
81. A genetic male fetus with functional testes B. 11-cis-retinal to all trans-retinal
does not develop male external genitalia. The C. all trans retinal to all cis-retinal
most likely reason for this is: D. 11-trans-retinal to 11-cis-retinal
A. mutations in SRY
B. deficiency of testosterone 89. Which of the following is characterized by a
C. deficiency of Mullerian inhibiting peptide hypercontractile, shrunken bladder, increased

in
D. deficiency of testosterone 5 reductase frequency of urination and incontinence?
A. Deafferented bladder
82. Of those mentioned below, the least likely B. Denervated bladder
change in a normal pregnancy is a/an: C. Spastic neurogenic bladder
A. increase in HR D. Bladder in acute paraparesis

s.
B. increase in cardiac output
C. increase in blood volume 90. The smooth muscle relaxing effects of
D. decrease in SBP endothelium derived relaxing factor nitric oxide
are mediated by an increase in intracellular
83. The SI unit of pressure is: levels of
A. mm Hg A. cAMP
B. cm H2O
C. Pascal
D. Torr
iim
84. Which method is dependable for measuring
B. cGMP
C. calcium
D. endothelin

91. Calculate the osmolality of plasma given that


cardiac output when cardiac output is low? plasma glucose is 108 mg/dL, plasma [Na] is
A. Fick principle 125 mM, and blood urea nitrogen is 140 mg/dL.
B. Thermodilution method A. 300 mOsm/Kg H2O
4a
C. Indicator dilution method B. 306 mOsm/Kg H2O
C. 312 mOsm/Kg H2O
85. Patients with restrictive lung disease breathe: D. 318 mOsm/Kg H2O
A. slow and deep
B. slow and shallow 92. Which of the following methods is not used
C. rapid and deep for measurement of body fluid volumes?
D. rapid and shallow A. Antipyrine for total body water
B. Inulin for ECF volume
m

86. The mitotic spindle is made up of a protein C. Evans blue for plasma volume
called: D. I125 albumin for blood volume
A. tubulin
B. caveolin 93. Which of the following transport processes
C. connexin does not exhibit 'saturation kinetics'?
Ai

D. spindlin A. facilitated diffusion


B. Na+-Ca2+ exchanger
87. When a heavy object in hand is lowered, the C. simple diffusion
extension at the elbow is brought about by: D. Na+ coupled active transport
A. active shortening of the extensors
B. passive shortening of the extensors 94. Insulin dependent glucose uptake into
C. active lengthening of the flexors skeletal muscle and adipose tissue is mainly
D. active shortening of the flexors mediated by:
A. GLUT 1
B. GLUT 2

121
C. GLUT 3
D. GLUT 4 101. Physiologic dead space ventilation is
decreased by:
95. Insulin secretion from beta cells of pancreas A. upright position
in response to a glucose load is mediated by: B. positive pressure ventilation
A. GLUT 1 C. neck flexion
B. GLUT 2 D. emphysema
C. GLUT 3
D. GLUT 4 102. What does the macula densa sense?

in
A. Na concentration of fluid delivered to DCT
96. Which of the following is not true about B. Cl concentration of the fluid delivered to
cerebrospinal fluid? DCT
A. Its pH is less than that of plasma C. NaCl concentration of fluid delivered to
B. It is formed in arachnoid villi DCT

s.
C. It normally does not contain neutrophils D. Volume of fluid delivered to DCT
D. Leakage of CSF during dural tap causes E. Combination of the above
headache
103. When does a switch over from fetal to adult
97. Which mechanism is not currently hemoglobin synthesis begin?
implicated in learning and memory? A. 14 weeks gestation
iim
A. Modulation of release of neurotransmitters
in postsynaptic neurons in response to
repeated firing of presynaptic neurons
B. Modulation of neurotransmitter receptor
synthesis in postsynaptic neurons
B. 30 weeks gestation
C. 36 weeks gestation
D. 7-10 days postnatally

104. The following values were obtained from a


C. Formation of new neurons patient diagnosed to have diabetic ketoacidosis:
D. Spatial organization of association areas PaO2 = 90 mm Hg, PaCO2 = 30 mm Hg, plasma
[HCO3] = 10 mM, the pH of CSF in this patient
4a
98. The first reflex response to reappear during would be about:
recovery from spinal shock is the A. 7.2
A. tympanic reflex B. 7.25
B. withdrawal reflex C. 7.33
C. neck righting reflex D. 7.40
D. labyrinthine reflex
105. A 10 days old neonate is posted for pyloric
99. The concentration of sodium in normal stenosis surgery. Serum calcium is 6 mg/dL
m

human plasma varies from (normally 8.5 10.5 mg/dL). What information
A. 280 295 mOsm/Kg H2O would you like to have before you supplement
B. 135 145 mOsm/Kg H2O calcium to this neonate?
C. 240 255 mOsm/Kg H2O A. Blood glucose
D. 95 110 mOsm/Kg H2O B. Serum protein
Ai

C. Serum bilirubin
100. Arterial blood gas of a 5 year old child D. Oxygen saturation
done at sea level gives the following results: pH
7.41, PaO2100 mmHg, and PaCO2 40mm Hg. Answers: Blast from the Past
The child is being ventilated with 80% oxygen. 1C 2A 3D 4C 5C
What is the (A-a) PO2? 6D 7C 8A 9D 10D
A. 420 mm Hg 11D 12A 13C 14D 15C
B. 470 mm Hg 16C 17C 18C 19C 20C
C. 520 mm Hg 21D 22D 23C 24C 25B
D. 570 mm Hg

122
26B 27A 28B 29B 30C 8. While the strength duration curve can tell us if
31D 32 33C 34A 35C a nerve is functioning or not or what is the
36B 37B 38C 39D 40D degree of impairment at a point in time, of late,
41A 42B 43C 44A 45C it has been superseded by electromyography
46C 47D 48B 49D 50C which has this capability plus a number of others
51D 52C 53B 54D 55D (this technique has a wide range of uses). That is
56D 57A 58A 59A 60C why, perhaps, little mention is made about SD
61D 62A 63B 64C 65D curve these days in textbooks. The only
66A 67B 68A 69C 70C advantage of the SD curve is that it is

in
71A 72D 73C 74C 75B noninvasive. EMG can be recorded with surface
as well as needle electrodes. The occurrence of
76B 77C 78D 79D 80B
reinnervation potentials or denervation
81D 82D 83C 84A 85D
potentials (in an EMG) can be used for
86A 87C 88B 89C 90B
prognostication.
91B 92D 93C 94D 95B

s.
96B 97C 98B 99B 100A 11. A 1C decrease in temperature reduces
101C 102? 103NS 104D 105B cerebral metabolic rate for oxygen by 7%.
Answer Explanations: 12. Type B fibers are more susceptible to
1. Volume of distribution of this substance = blockade by anesthetics compared to large
iim
30% of total body water (TBW). In a healthy
adult male weighing about 70 kg, the volumes of
ICF and ECF are 28 and 14 L respectively; i.e.,
1/3rd of TBW is outside cells. Thus, the
substance in question distributed exclusively in
ECF.
diameter A fibers. Sympathetic preganglionic
neurons, which originate in the intermediolateral
column of the thoracolumbar segments of the
spinal cord are type B.

15. In the dark, the prosthetic group retinene in


retinaldehyde is in the 11-cis configuration. The
2. TBW = ICF + ECF volumes = 28 + 14 = 42 L action of light is to change the shape of this to
4a
Concentration of potassium in ICF = 140 mM the all-trans isomer.
Concentration of potassium in ECF = 5 mM
Plasma volume = 3.5 L 17. Sleep walking, bed wetting and night terrors
Volumes of all other fluids combined = 38.5 L occur during slow wave sleep or, more
Amount of K contained in plasma is about 20 specifically during arousal from slow wave
mM. sleep. They are not associated with REM sleep
Amount of K in all other fluids = 3980 mM See p 200-1, Ch 11, WFG, 2005.
Thus, about 0.4% of potassium is in plasma
m

18. Cerebral blood flow is regulated by local


3. 1 gram mole of MgSO4 = 1000 mM MgSO4 metabolites and hypoxia would cause
Each molecule of MgSO4 dissociates to give 2 vasodilation and thereby slightly increase flow.
ions viz., Mg and sulphate. Similarly, hypercapnia produces cerebral
Thus, 1000 mM = 2000 mOsm/L vasodilation and an increase in cerebral blood
1 mOsm/L exerts a pressure of 19.3 mm Hg
Ai

flow.
Therefore, 2000 mOsm/L will exert an osmotic
pressure = 19.3 2000 = 38600 mm Hg 23. Cushings triad, which occurs in patients
with raised intracranial tension, includes
4. Concentration of KCl = 39 mg % = 39 mg/dL hypertension, bradycardia and irregular
= 390 mg/L = 0.390 g/L respiration but it is uncommon to have all signs
Molar mass of K = 39 g in a patient.
1 mole of K = 39 g/L http://www.hawaii.edu/medicine/pediatrics/pemxray/v5c06.html;
Thus, 0.39 g of K = 10 mM of K last accessed 15 November 2006.

123
24. Neurotransmitters that increase food intake Furthermore, Webers test was localized to the
are said to be orexigenic. Two neurotransmitters left side. Taken together, this is indicative of
with orexigenic effects are the polypeptides sensorineural deafness on the right side.
orexin A and orexin B. Orexins are synthesized
in neurons located in the lateral hypothalamus. Absolute bone conduction of a test subject could
Recall, the feeding center is located in the lateral also be compared with a reference subject whose
hypothalamus. Ghrelin and neuropeptide Y also hearing is known to be normal. The results of
stimulate food intake. In contrast, leptin from this test (Schwabachs test) are useful in
adipose tissue acts on the hypothalamus to detecting early sensorineural deafness.

in
inhibit food intake (anti-orexigenic effect).
31. ADH is stored in the posterior pituitary
27. Metabolic rate starts increasing 30 min after bound to neurophysins. However, ADH
a meal, stays high for about 2 hours and declines circulates free in plasma.
to the resting level within 4 hours.

s.
33. GABAergic inhibitory inputs to the
28. First of all, to be very specific, the term hypothalamus are implicated (Ch 10.1, Sec 10,
cutaneous vasodilation should have been used Reproduction, p. 570 In: Understanding Medical
rather than vasodilation which refers to all Physiology by RL Bijlani, 3rd Ed. Opioid
vascular beds. This is important because at a peptides such as the enkephalins and the beta-
time when blood vessels in the skin dilate in endorphin reduce the frequency of GnRH pulses
iim
response to neural signals from the
hypothalamus, vasoconstriction could occur in
other vascular beds (example, GIT and skeletal
muscle)
See p 449, Ch 23, WFG, 21st edition.

36. The 46 XX primary oocyte undergoes


meiosis (a halving of chromosome number),
which consists of two phases.
Sweating is an active process; logically, an
increase in flow should precede a significant Phase 1: Meiosis (reduction division) results in
increase in sweating. Indeed cutaneous the formation of secondary oocyte (with a large
4a
vasodilation and sweating go together; however, amount of cytoplasm) and the first polar body
cutaneous vasodilation per se could contribute to (with little cytoplasm). The first polar body is
heat loss even if it were not accompanied by extruded at the end of the first phase of meiotic
increased sweating. division. This is immediately followed by
ovulation. See WF Ganong, 21st Ed, Ch 23, Gonads:
29. The purpose of this question is to check if development and function of the reproductive system, pp
we understand the importance of masking the 438. I think that the best answer for the question
untested ear when performing the Rinnes test. is at the time of ovulation. Ovulation is the
m

For example, if we were testing the right ear, release of the secondary oocyte from the ovary.
how sure are we that the person hears through
the right ear and not on the left? To ensure this Phase 2: The 23 X secondary oocyte
does not happen, the untested ear is masked immediately begins a mitotic division but this is
using a Baranys noise box. Alternately, we arrested in metaphase until fertilization occurs.
Ai

could ask the patient to insert his finger in the After fertilization, the mitotic division is
left ear and make quick noisy movements during completed and the second polar body is
the Rinnes test. extruded.

Second, the results of Rinnes and Webers test 37. The phenotypic sex is female (it is a baby
must be interpreted together and not in isolation. girl); gonadal sex is male (because the surgeons
are telling that the inguinal masses are testes).
In this case, the Rinnes test was negative (called We are to predict chromosomal sex (genetic sex)
false negative) because there is apparently no based on this information. Since testes are
defect in ossicular conduction on the right side. present, a Y chromosome should be present.

124
originated in an atrial focus since it has not reset
In the embryonic stage, the testes could have the sinus rhythm. An atrial premature beat is not
been functional or otherwise. If they functioned followed by a compensatory pause; it is
normally, male internal genitalia would have conducted to the SA node and it resets sinus
developed and Mullerian duct structures would rhythm. In contrast, a ventricular premature beat
have regressed as a result of actions of is usually not conducted retrograde through the
testosterone and Mullerian-inhibiting substance AV node to the SA node, so it does not reset
respectively. On the other hand, nonfunctional normal sinus rhythm.
embryonic testes could not have prevented

in
female internal and external genitalia from The duration of premature ventricular
developing. depolarization is longer since it is conducted
much slowly through the myocardium rather
Gonadal sex is male (so the genetic sex has to be than through the normal conducting pathway.
46 XY) and phenotypic sex is female this is Thus the resulting ventricular contraction is

s.
testicular feminizing syndrome, caused by unlikely to have been really premature. So
mutations in androgen receptors or a deficiency both heart sounds would have been present at
of testosterone 5 alpha reductase. beat X. This is an illustration that a premature
ventricular depolarization does not always result
46. Androgens stimulate erythropoiesis. in a premature beat.
Estrogens have been noted to have an inhibitory
iim
effect on erythropoiesis (Reference: RL Bijlani,
Understanding Medical Physiology, Third edition, Jaypee
Publishers). This may be one reason why anemia
occurs in hyperestrogenic states; perhaps, the
anemia is relative, i.e., due to an increase in
53. For correct estimation of blood volume as
well as cardiac output, the dye must remain in
the blood stream.

55. Without the carotid pulse transducer, LVET


plasma volume rather than inhibition of cannot be determined. Since PEP = QS2-LVET,
erythropoiesis. High levels of ACTH and PEP cannot also be determined.
cortisol stimulate erythropoiesis and
4a
polycythemia is a striking feature of Cushing 56. Note that blood loss occurs over a 30-minute
syndrome. Thyroxine and prolactin also period (a common example is venesection of a
stimulate erythropoiesis. blood donor). There occurs only a slight increase
in HR because of a reduction in central blood
48. Sensitization of the mothers immune system volume. BP would be maintained in the steady
with fetal Rh antigens results in the production state.
of anti Rh immunoglobulin. First, IgM is formed
but this is too large to cross the placental barrier. 57. When MAP is in the normal range, i.e.,
m

between 70 and 110 mm Hg, an increase in BP


The major antibody in the primary immune results in an increase in discharge rate from the
response is Ig M. However, over a time period carotid sinus. Activity in the buffer nerves
that varies from individual to individual, IgG is inhibits the tonic vasoconstrictor discharge from
also produced as part of this response, although the medulla. Therefore, sectioning of the buffer
Ai

the titer of IgG is usually not sufficient to evoke nerves when MAP is normal would result in
significant hemolysis in the first pregnancy. On acute elevation of BP. This is called neurogenic
the other hand, in a woman who has been hypertension.
previously sensitized with Rh antigen and anti D
Ig G titers were significant, significant A MAP less than 70 mm Hg is a hypotensive
hemolysis could occur even during the first state. When MAP is as low as 60 mm Hg,
pregnancy. activity in the baroreceptors is maximally
inhibited. See Figure 2 at
50. You can see that the depolarization is from a http://www.cvphysiology.com/Blood%20Pressure/BP012.htm
focus in the ventricle. The beat could not have

125
Thus, sectioning of afferents from arterial coronary vasospasm and a severe reduction in
baroreceptors would not be expected to have any coronary blood flow.
greater sympathoexcitatory effects.
Cutaneous blood flow is most often varied in
On the other hand, when MAP is as low as 60 response to neural signals from the
mm Hg, blood flow through the arterial hypothalamus. It is the vascular bed that is most
chemoreceptors near the carotid sinus would be and consistently under neural control.
reduced. A reduction in either PaO2 and or a
decrease in flow through the chemoreceptors 65. Reynolds number Re = DV/

in
leads to a reflex increase (via activity in the is the density of the fluid;
buffer nerves) in sympathetic outflow that serves D is the diameter of the vessel;
to bring BP back into the normal range. Thus, V is the blood flow velocity; and
when the buffer nerves are sectioned at a time is the viscosity of fluid.
when MAP is lower than normal, BP would fall Flow is turbulent when Re exceeds 3000;

s.
further. See p. 628, Ch 33, WFG, 2005 See p. 583, Ch 30, WFG, 2005.
Summary: When MAP is reduced below 70 mm
Hg, arterial baroreceptors are maximally 66. The major driving force is the capillary
deactivated and sympathetic outflow is hydrostatic pressure which is normally about 40
disinhibited as a result of this; apart from this, mm Hg at the arterial end of the capillary in a
activation of the arterial chemoreflex contributes capillary that is at the level of the heart.
significantly to increasing BP.

forward heart failure?


iim
61. Why does edema not occur in early stage

1. Acute cardiac failure (forward failure)


67. At functional residual capacity, lungs and the
chest wall recoil in opposite directions causing
the intrapleural pressure to be negative.

2. Cardiac output (Q) reduces 68. Hb F binds 2,3 BPG less avidly compared to
3. Blood flow to tissues reduces Hb A. Other statements are true.
4. Accumulation of metabolites causes a
4a
drop in peripheral resistance (TPR) 69. Here is a professor with a passion for SI
5. BP = Q TPR units, and a you miss I hit kind of a
6. BP falls (i.e. systemic capillary question! The question is designed to test
hydrostatic pressure falls) whether we know how to convert pressure in
7. Reduced filtration at the capillaries > no mm Hg to Pascals. Here it is:
edema 1 atm = 760 mm Hg = 100 kPa
If edema does not occur, then this is the 7.6 mm Hg = 1 kPa
explanation. Ref: Oxford Handbook of Clinical Medicine, 5th Ed,
m

Longmore M et al. p 717.


62. All organs are capable of autoregulating
their blood flows. Although cerebral vessels 70. Methemoglobin cannot transport oxygen.
have noradrenergic innervation, cerebral blood Less than 1% of Hb is in the Fe3+ state.
flow itself is not chronically under neural Methemoglobin cannot also unload oxygen
much like carboxyhemoglobin. Thus, there is
Ai

control. Cerebral blood flow is excellently


autoregulated in the steady state when mean also a leftward shift of the oxyhemoglobin
arterial pressure is between 65 and 140 mm Hg. dissociation curve. Taken together, the oxygen
content of blood is reduced in
Although the heart is also capable of excellent methemoglobinemia and the oxygen unloading
autoregulation, it is possible that under certain capacity of blood is reduced even more.
circumstances, neural regulatory mechanisms
override local regulatory mechanisms. For Carbon monoxide poisoning reduces oxygen
example, sudden excessive discharge in the content of arterial blood by competing with
sympathetic fibers to the heart can cause severe

126
oxygen to bind heme. Respiratory acidosis is precocity are apt to be dwarfed because of
associated with severe hypoxemia. premature epiphysial closure.

Methemoglobin binds oxygen but is not capable 82. Pregnancy is a classic example of a
of unloading oxygen until it is reduced to hyperdynamic circulatory state in which total
ferrohemoglobin. peripheral resistance is lowered as a result of
estrogens and the opening of a new vascular bed
Clinically, respiratory acidosis is always in the systemic circulation. Blood volume, heart
accompanied by arterial hypoxemia (type II rate and cardiac output are increased. The

in
respiratory failure). diastolic pressure drops and pulse pressure
increases; MAP is maintained. A decrease in
However, in cyanide poisoning, oxygen content SBP is not a feature of pregnancy.
of blood is normal, yet utilization of oxygen in
the mitochondria is inhibited because cyanide 84. This is because when flow is reduced, flow

s.
inhibits cytochrome oxidase, the final donor of velocity is reduced and temperature is quickly
electrons to molecular oxygen. dissipated thereby rendering an accurate
measurement of pulmonary blood flow difficult.
71. The fact that oxygen saturation of Hb is as
low as 60% although PaO2 is as high as 300 mm On the other hand, Ficks principle is
Hg suggests a diagnosis of carbon monoxide dependable when cardiac output is low since
poisoning straightaway. iim
72. This reflex helps a newborn child inflate its
liquid filled lungs.
arteriovenous oxygen difference is quite high
and oxygen uptake across the lungs reduces.

Note that cardiac output (or pulmonary blood


flow) equals oxygen uptake across the lungs
73. O2 carrying capacity of blood: divided by whole-body arteriovenous oxygen
0.003 ml O2 per mm Hg O2 tension per deciliter difference.
of blood
4a
i.e., 3 10-3 ml O2 per mm Hg O2tension per Also note that when there are intracardiac
deciliter of blood shunts, Ficks method will either overestimate or
= 3 10-5 ml of O2 per mm Hg O2 tension per underestimate cardiac output depending on
ml of blood whether the shunt is left-to-right or right-to-left.
Here, O2 carrying capacity = 0.0025 ml.
Oxygen tension should have been 2.5 10-3 Furthermore, when there is a ventricular septal
Divided by 3 10-5 defect, mixed venous blood must be taken from
= 80 mm Hg the right atrium rather than the pulmonary
m

artery, i.e. venous blood must be taken proximal


76. PAH clearance gives effective RPF to the shunt. See Harrisons Principle of
Clearance of PAH depends upon its plasma Internal Medicine, 16th ed, vol. 2, p 1329-30.
concentration. Secretory and reabsorptive
processes have a rate maximum i.e., they can be 87. Take a rather heavy object in your hand and
Ai

saturated. When a high dose is used, PAH try lowering it toward the ground. When you do
secretory capacity is overwhelmed and clearance so, you will feel the tension in your biceps, yet
of PAH reflects this secretory capacity rather the elbow extends; that is, although biceps
than RPF. So UPAH V/ PPAH i.e. PAH clearance develops tension, it lengthens. One must
will be low, and estimated RPF will be falsely understand the difference between contraction of
low. sarcomeres which leads to development of
tension, and change in muscle length that
80. Estrogens ultimately terminate linear growth depends upon load. Sarcomeres contract and
by causing epiphyses to fuse to the long bones develop force but the muscle can shorten or
(epiphysial closure). Patients with sexual lengthen depending upon the load. If a muscle

127
lengthens while developing force, external work In the case of positive pressure ventilation, total
done is negative. dead space increases depending on:
the volume of the tubing used;
91. Plasma osmolality (mosm/L) = regional differences in lung compliance in
2 [Na+] + [glucose] / 20 + [BUN] 18/50 patients with lung disease
(mmol/L) (mg/dL) (mg/dL)
102. I am not sure.
97. I am not really sure about option C, although
I would choose it over the other options for 103. Really tough; the synthesis of Hb A

in
which there is evidence. However, the belief that begins in the 8th month (28-32 wk) of fetal life.
we do not add brain cells after birth is probably See Champe PC: Lippincotts Illustrated
incorrect. New neurons arise from stem cells in Reviews. Biochemistry. 2nd edition. My
the olfactory bulb and hippocampus and since understanding is that the switch over to HbA
formation of long-term memories occurs in the synthesis is completed over a period of time and

s.
hippocampus, it is possible that the two are not at one point. Of the options given, 30 weeks
related; this hypothesis is currently intensely seems the best to me. That is my opinion.
researched (example, see
http://www.jneurosci.org/cgi/content/full/22/3/635 104. Using Henderson equation, [H+] of blood
Also See p 270, Ch 16, WFG, 2005. can be calculated to be 72 nM. That is, the
individual has metabolic acidosis. CSF [H+] is
98. Easy question; in the first place, the iim
tympanic reflex, labyrinthine reflex and neck
righting reflex are not lost in paraplegic humans.

100. PAO2
directly proportional to PaCO2. Since PaCO2 is
30 mm Hg (10 mm Hg lower than it normally
is), CSF pH must be higher than normal.
Normally, CSF pH is 7.33. From the options
given, 7.4 is the only answer one can choose.
= [(PB-PH2O) FiO2] [(PACO2) / RER].
This is the alveolar gas equation that is used to 105. Normally, total serum calcium ranges from
estimate the oxygen tension of alveolar gas 8-10 mg/dL. Normally, the ratio of protein
4a
In this problem, bound calcium to ionized calcium is 1:1. In this
PB = 760 mm Hg, FiO2 = 0.8; PaCO2 = PACO2 case, total calcium = 6 mg/dL. A decrease in
= 40 mm Hg, and RER is assumed to be 0.8 total calcium could be due to:
Substituting we get, PAO2 = 520 mm Hg Decrease in serum protein
Arterial PO2 = 100 mm Hg Decrease in ionized calcium
Thus, D (A-a) O2 = 420 mm Hg.
This represents a considerable defect in gas Thus, it is appropriate to seek to determine if
exchange as normally the difference between hypoalbuminemia is the cause of hypocalcemia.
m

alveolar and arterial oxygen tension is not ***************************************


greater than 10 mmHg.

The learning point here is that a PaO2 of 100


mmHg is not necessarily reflective of gas
Ai

exchange function of lungs as it is influenced by


the oxygen concentration of inspired gas.

Rather, D (A-a) O2 is a measure of oxygen


exchange function of lungs.

101. Extension of the neck, positive pressure


ventilation, and emphysema increase ventilation
of dead space.

128
More Questions for Self-Study 6. In adults, ECF volume is about _____% of
body weight
A. 10%
GENERAL PHYSIOLOGY B. 20%
C. 30%
Unless otherwise indicated, choose the single D. 40%
best answer.
7. In a healthy adult male weighing 70 kg, the
1. The philosophy that all vital mechanisms are total volume of fluid present in the transcellular

in
directed toward maintaining constancy of compartment is about:
composition of the internal environment and that A. 1L
this is necessary for the sustenance of life was B. 2L
first propounded by: C. 3L
A. Claude Bernard D. 4L

s.
B. Walter B Cannon
C. William Harvey 8. Which of the following statements regarding
D. Alan Hodgkin body fluid compartments is incorrect?
In health,
2. The term homeostasis was coined by: A. blood volume is about 70-80 ml/kg body
A. Claude Bernard weight
B. Walter B Cannon
C. Homer Smith
D. William Harvey
iim
3. Who is regarded as the Father of
B. the ratio of ICF to ECF volume is about 2
C. ICF volume accounts for about 60% of body
weight.

9. The body fluid compartment that contains


Experimental Physiology? more osmotically active particles (in relation to
A. Claude Bernard other fluid compartments in the same individual)
B. Walter B Cannon is:
4a
C. Ernest H. Starling A. intracellular fluid
D. William Bayliss B. plasma
C. interstitial fluid
4. The core body temperature of an experimental
animal is raised from 98F to 106F by passive 10. Radioiodinated fibrinogen is a marker that
heating. Eventually, it dropped back to 99F. can be used to measure the volume of:
What is the gain of the temperature regulation A. plasma
system in this instance? B. interstitial fluid
m

A. Zero C. ECF
B. One D. red blood cells
C. -7
D. Infinity 11. Which of the following is not isosmotic with
normal human plasma?
Ai

5. Which of the following is not a feature of A. 5% dextrose


negative feedback control systems? B. 0.85% NaCl
A. Output is one of the inputs to the system C. 20% mannitol
B. It is based on a set-point for the controlled
variable. 12. In the steady state, the value of which of the
C. The system corrects errors following variables is the same in ICF and ECF?
D. The set point of the system cannot be A. pH
changed by inputs other than the controlled B. osmolality
variable C. concentration of proteins
D. number of osmoles

129
C. cadherin
13. ICF volume does not change when D. calcineurin
dehydration is:
A. isotonic 20. The mitochondrial genome is absent from:
B. hypertonic A. sperm cells
C. hypotonic B. ovum
C. mature red blood cells
14. Which of the following modes of transport is D. white blood cells
quantitatively more important?

in
A. Diffusion Answers for Practice Questions in General
B. Filtration Physiology
C. Vesicular transport 1A 2B 3A 4C 5D
6B 7A 8C 9A 10A
15. Which of the following is an example of 11C 12B 13A 14A 15E

s.
primary active transport? 16B 17AD 18A 19A 20C
A. Ca extrusion from cells by the Na-Ca
exchanger Answer explanations:
B. Glucose entry into cells through glucose 4. Gain = correction / error; in this example, the
transporter 2 (GLUT-2) correction is 7 degrees i.e. from 106F down to

aquaporins iim
C. H2O flux across cell membranes through

D. Glucose uptake into intestinal epithelial cells


by Na-glucose cotransporter 1 (SGLT-1)
E. Ca sequestration in sarcoplasmic reticulum
by Ca-ATPase
99 F, and the error (deviation from the original
value of 98F) is 1F. The gain of a negative
feedback control system is negative. If the error
is zero, gain is infinite. Guyton and colleagues
suggested that the renal pressure-natriuresis
mechanism for controlling body fluid volumes
has infinite gain.
16. Sodium-glucose cotransport in the intestine
and kidney is an example of: 5. As far as biological systems are concerned,
4a
A. primary active transport the application of control system theory is a
B. secondary active transport means to understanding these are concepts and
C. facilitated diffusion means of describing behavior of physiologic
D. passive transport systems, not necessarily truths.

17. Which of the following is / are active You can view a simple schematic of a negative
transport processes? (Tick all that apply) feedback control system at
m

A. Extrusion of calcium into ECF http://esprakash.wordpress.com/mcqsmedphy


B. Efflux of K during repolarization Click on Negative Feedback Control System
C. Entry of chloride into neurons
D. Transcytosis (vesicular transport) Take temperature regulation as an example. The
controller is the hypothalamus. The fact that a
18. The most abundant protein in mammalian rise in body temperature leads to a fall in
Ai

cells is: temperature back toward 98.6F suggests that


A. actin temperature is the controlled variable, and this
B. collagen operates as a negative feedback control system.
C. titin This means that there must be a sensor for
D. dystrophin temperature. The hypothalamus is in fact able to
sense core body temperature.
19. Gap junctions are made up of a protein
called: When there is an infection, often the release of
A. connexin cytokines as part of the immunologic response
B. clathrin act on the brain to raise the set-point of the

130
temperature regulation system to a higher level 13. In isotonic dehydration (example, that due to
so that a higher than normal body temperature is loss of blood), an equal amount of solute and
maintained and results in fever. The control water is lost from ECF, and there is no change in
system is then said to be reset to a higher ECF osmolality. There is thus no drive for
operating level. However, once the infection and movement of water between ICF and ECF, and
the immune response resolve and the therefore there is no change in ICF volume.
concentration of cytokines and prostaglandins Therapy is aimed at replenishing ECF volume,
that reset the temperature set point upward and typically 0.9% NaCl (a solution that is
resolve, the set point returns to 98.6F indicating isotonic with normal human plasma) is used. In

in
that resetting of a control system is not hypertonic dehydration, steady state ECF
necessarily permanent. osmolality increases; thus, it leads to
A classic negative feedback loop is animated intracellular dehydration as well. In hypotonic
here at http://tiny.cc/6q0v1 the full URL is dehydration, steady state osmolality of ECF is
http://highered.mcgraw- decreased and some water will enter cells.
hill.com/sites/0072495855/student_view0/chapter20/animation__h

s.
ormonal_communication.html
[accessed 10 May 2010] 15, 16. Please review the Supplement on
Transport across cell membranes (p. 10)
At this link http://tiny.cc/lrlzj you can see
positive feedback as well as negative feedback 17. Proteins are translocated from capillaries
into the interstitium by transcytosis (vesicular
axis illustrated. The full URL is
http://highered.mcgraw-
iim
interaction in the hypothalamo-pituitary-ovarian

hill.com/sites/0072495855/student_view0/chapter20/animation__p
ositive_and_negative_feedback__quiz_1_.html
May 2010]
[accessed 10
transport). This involves endocytosis of the
protein molecules into vesicles in endothelial
cells and exocytosis of these vesicles into the
interstitium.

18. Actin is present in virtually all cells although


8. ICF volume constitutes 2/3rd of total body most abundant in muscle. Collagen is the most
water (not body weight). In a healthy adult abundant protein in extracellular matrix. Titin is
weighing about 70 kg, ICF volume (28 liters) is
4a
the largest protein in the human body.
about 40% of total body weight. Dystrophin is a glycoprotein in the muscle cell
membrane; it is involved in transmitting the
9. Since the volume of ICF compartment is tension generated by shortening of sarcomeres
twice as large as ECF, one can readily guess that eventually to the tendon.
ICF would contain more osmotically active
particles compared to ECF. The steady state 19. Clathrin is a protein involved in receptor-
osmolality (concentration of osmotically active mediated endocytosis; it is the protein that coats
m

particles) of ICF and ECF is identical. pits that are the sites of aggregation of cell-
surface receptors involved in receptor-mediated
10. Fibrinogen is a plasma protein. Thus, endocytosis. Cadherin is a cell adhesion
normally, one would expect radiolabeled molecule. Calcineurin, like calmodulin, is a
fibrinogen to distribute only in plasma. calcium binding protein found in cells.
Ai

12. ICF is much more acidic than ECF. For 20. Mature RBCs do not have mitochondria and
example, in muscle cells the pH is about 6.8. they are anerobically respiring.
The steady state osmolality (i.e., concentration
of osmotically active particles) of all body fluid Although sperm cells contain mitochondria,
compartments must be the same. The fact that, note, however, that mitochondria from sperm
in a healthy adult, ICF volume is twice as large cells do not enter the secondary oocyte during
as ECF volume should indicate that the absolute fertilization; i.e., mitochondrial DNA in the
number of osmoles is much greater in the ICF. zygote is exclusively of maternal origin.
**************************************

131
NERVE-MUSCLE PHYSIOLOGY 7. All of the following are true about slow
skeletal muscle fibers (in contrast to fast
In each of the following questions, select the skeletal muscle fibers) except that slow fibers:
single best answer. A. have a greater number of mitochondria
B. have higher myoglobin content
1. The RMP of some neurons is equal to the C. typically demonstrate oxidative metabolism
equilibrium potential of: D. have higher myosin ATPase activity
A. Na
B. K 8. Which of the following is not observed in

in
C. Cl cardiac muscle?
D. Ca A. Contraction of the ventricle is an all-or-none
phenomenon
2. The equilibrium potential of chloride in B. Staircase phenomenon
mammalian spinal motor neurons is about: C. Complete summation of successive

s.
A. + 20 mV contractions
B. - 40 mV D. Long refractory period
C. - 70 mV
D. - 90 mV (inside negative) 9. Which of the following muscles consists
predominantly of fast muscle fibers?
A. Soleus
A. humans
B. whales
C. squid
D. ostriches
iim
3. The largest known axons are found in:
B. Diaphragm
C. Orbicularis oculi
D. Detrusor

10. Which of the following proteins is absent


4. The Nernst potential (also called the from smooth muscle?
Equilibrium potential) is positive for: A. Actin
A. Na and Cl B. Myosin
4a
B. Na and K C. Troponin
C. Na and Ca D. Tropomyosin
D. K and Cl
11. Which of the following statements about
5. Which of the following statements regarding visceral smooth muscle is incorrect?
nerve fibers is incorrect? A. Neighboring cells are electrically coupled by
A. Large diameter fibers conduct faster than means of gap junctions
B. It can be tetanized
m

small diameter fibers.


B. Type C fibers are more susceptible to C. Force is graded by varying intracellular
inhibition by local anesthetics. calcium
C. Nerves innervating skeletal muscle contain D. Stretch of smooth muscle consistently elicits
only one type of nerve fiber. active contraction
Ai

6. Excitation and contraction of muscle are 12. Oxidative capacity is highest in:
coupled by: A. type I muscle fibers
A. ATP B. type II muscle fibers
B. myosin C. type F muscle fibers
C. calcium
D. ryanodine receptor 13. Which of the following statements about
gastrointestinal smooth muscle is incorrect?
A. Contraction can occur in the absence of
extrinsic neural innervation.
B. Within limits, stretch increases tone.

132
C. It always exhibits linear length-tension A. tetanus
relationships. B. tetany
D. Twitch duration is longer when compared to C. staircase phenomenon
skeletal muscle. D. summation

14. The calcium-binding protein that plays a key Answers: Self-Study in Nerve-Muscle
role in the regulation of smooth muscle cell Physiology
contraction is: 1C 2C 3C 4C 5C
A. dystrophin 6C 7D 8C 9C 10C

in
B. calmodulin 11D 12A 13C 14B 15C
C. troponin C 16A 17B 18D 19B 20C
D. calcineurin
Answer Explanations:
15. An example for multi-unit smooth muscle is: 8. Summation of the type seen in skeletal muscle

s.
A. stomach cannot occur in cardiac muscle. That is, a second
B. myometrium contraction cannot begin before the previous
C. iris contraction (systole) is completed and diastole
D. vascular smooth muscle (filling) has commenced. However, a premature
depolarization and premature contraction can
16. Actin is tethered to Z-lines in the sarcomere
by:
A. actinin
B. titin
C. nebulin
D. dystrophin
iim occur when the ventricle is just filling with
blood.
**************************************

CENTRAL NERVOUS SYSTEM

17. A stronger than normal stimulus can cause Some quizzical questions
excitation of nerve or muscle during:
4a
A. absolute refractory period Whatzit:
B. relative refractory period 1. The photoreceptor protein in the rod cells:
C. spike potential 2. The organ of hearing:
D. overshoot 3. The fluid present in the scala media:
4. The unit of sound pressure:
18. Under basal conditions, which is the 5. The fluid present in the bony labyrinth:
predominant energy substrate utilized by cardiac 6. The muscles subserving the tympanic reflex:
muscle?
m

A. Carbohydrate Answers to Whatzit:


B. Amino acids 1. Rhodopsin
C. Ketones 2. Organ of Corti
D. Fatty acids 3. Endolymph
4. Decibel
Ai

19. The gene coding for which of the following 5. Perilymph


proteins is the largest? 6. Stapedius and tensor tympani
A. Titin
B. Dystrophin What is the
C. Gigantin 1. major excitatory neurotransmitter in the
D. Nebulin brain
2. main inhibitory neurotransmitter in the brain
20. At a given stimulus intensity, the increase in 3. neurotransmitter released by small intensely
the force of muscle contraction with increasing fluorescent (SIF) cells in autonomic ganglia
frequency of stimulation is referred to as:

133
4. neurotransmitter released by Golgi bottle 9. Increased gamma efferent discharge
neurons increases the sensitivity of the stretch reflex.
5. neurotransmitter released by Renshaw cells
6. major neurotransmitter released by 10. If gamma motor neurons innervating a
sympathetic postganglionic neurons skeletal muscle were cut, tone of that muscle
7. cotransmitter released by sympathetic would decrease.
postganglionic neurons
8. major neurotransmitter released by 11. If Ia afferents from skeletal muscle were cut,
parasympathetic postganglionic neurons muscle tone would decrease.

in
9. cotransmitter released by some
parasympathetic postganglionic neurons 12. Botulinum toxin inhibits the release of
10. neurotransmitter released by all autonomic acetylcholine at the skeletal muscle
preganglionic neurons neuromuscular junction.

s.
Answers: 13. Strychnine produces convulsions by
1 Glutamate 6 Norepinephrine blocking glycine receptors on alpha-motor
2 GABA 7 Neuropeptide Y neurons.
3 Dopamine 8 Acetylcholine
4 Glycine 9 Vasoactive 14. Tetanospasmin produces spastic paralysis by
preventing the release of glycine from

5 Glycine 10
iim
intestinal
polypeptide
Acetylcholine

State whether the following statements are


true or false.
Renshaw cells (inhibitory interneurons) in
the spinal cord.

Answers: All statements 1-14 are true.

In each of the following questions, match


1. There are no photoreceptors overlying the items in list A with those in list B.
optic disk.
4a
Question 1
2. Renshaw cell is an inhibitory interneuron. List A (Functional areas)
1. Satiety center
3. Olfactory receptors are free nerve endings. 2. Feeding center
3. Osmoreceptors
4. Sign of Babinski is an unequivocal sign of a 4. Thirst center
corticospinal tract lesion. 5. Chemoreceptor trigger zone
6. Micturition center
m

5. The muscle spindle is basically a stretch 7. Deglutition center


receptor that responds to changes in muscle
length. List B (Location)
A. Ventromedial hypothalamus
6. The muscle spindle is part of a feedback B. Anterior hypothalamus
Ai

circuit regulating muscle length. C. Subfornical organ


D. Lateral hypothalamus
7. The Golgi tendon organ is part of a feedback E. Area postrema
circuit regulating muscle force. F. Medulla oblongata
G. Pons
8. Extrafusal muscle fibres can be made to
contract by stimulation of alpha motor Question 2
neurons. List A (Brodmanns area)
1. Areas 3,1,2
2. Area 41

134
3. Area 17
4. Area 18 and 19 Question 6
List A (Circumventricular organ)
List B (Function) 1. Median eminence
A. Visual association area 2. Organum vasculosum of lamina terminalis
B. Primary auditory area (OVLT)
C. Primary visual cortex 3. Subfornical organ (SFO)
D. Somatosensory cortex 4. Area postrema (AP)
E. Primary motor cortex

in
List B (Function)
Question 3 A. Thirst center
List A (Glia) B. Chemoreceptor trigger zone
1. Oligodendroglia C. Hypothalamo-hypophysial portal vessels
2. Microglia

s.
3. Astrocytes Question 7
4. Schwann cells List A (Cortical Area)
1. Posterior parietal cortex
List B (Functions) 2. Fusiform gyrus
A. Blood-brain barrier 3. Angular gyrus
B. Myelin synthesis 4. Posterior end of superior temporal gyrus
C. Scavenger cells
D. Neurotransmitter reuptake

Question 4
List A (Neurons)
iim List B (Effect of lesion)
A. Prosopagnosia
B. Anomic aphasia
C. Wernickes aphasia
1. Sympathetic postganglionic neurons D. Astereognosis
2. Parasympathetic postganglionic neurons
3. Preganglionic autonomic neurons Answers:
4a
4. SIF cells in autonomic ganglia 1 1A 2D 3B 4C 5E 6G 7F
5. Renshaw cells 2 1D 2B 3C 4A
6. Ia afferent neurons 3 1B 2C 3DA 4B
4 1B 2D 3D 4A 5F 6E
List B (Neurotransmitters) 5 1A 2B 3C
A. Dopamine 6 1C 2A 3A 4B
B. Norepinephrine 7 1D 2A 3B 4C
C. Epinephrine
m

D. Acetylcholine State if the following statements are true or


E. Glutamate false.
F. Glycine
1. The dominant EEG rhythm in an adult
Question 5 human at rest with eyes closed and the mind
Ai

List A wandering is beta rhythm.


1. Eye movements, stance, and gait
2. Truncal and proximal limb movements 2. REM sleep is called paradoxical sleep
3. Coordination of movements of distal parts of because it is characterized by slow waves.
the limbs
3. The alpha rhythm in EEG is replaced by a
List B fast irregular low voltage activity when a
A. Vestibulocerebellum person does mental arithmetic.
B. Spinocerebellum
C. Neocerebellum

135
4. Impulses in the ascending reticular Temperature regulating mechanisms:
activating system desynchronize cortical
activity producing arousal. Classify items 1-8 into:
Mechanisms that serve to conserve heat
5. Typically, in adults, REM sleep comprises Mechanisms that serve to dissipate heat
25% of total sleep time. Mechanisms that serve to generate heat
Mechanisms that reduce heat production
6. REM sleep deprivation produces weight loss
despite adequate caloric intake. 1. Panting

in
2. Sweating
7. There is more REM sleep in the early hours 3. Curling up in a ball
of morning. 4. Shivering
5. Rigors
8. Sleep spindles and K complexes are seen in 6. Specific dynamic action of food

s.
REM sleep. 7. Horripilation
8. The its too hot to move reaction
9. The mechanism that triggers REM sleep is
located in the pons. Heat loss mechanisms:
Panting and sweating
10. Hyperventilation elicits latent EEG

flow.

Answers:
1F 2F 3T
iim
abnormalities by decreasing cerebral blood

4T 5T


Mechanisms that conserve heat:
Curling up in a ball, horripilation

Mechanisms that generate heat:


Shivering, rigors, specific dynamic action of
6T 7T 8F 9T 10T food, muscular exercise

Notes on True-False Statements: Mechanisms that prevent excessive heat


4a
1. Alpha rhythm production:
The its too hot to move reaction
2. REM sleep is called paradoxical sleep
because it is characterized by high frequency ***********************************
EEG, similar to that observed in the awake state. ENDOCRINOLOGY &
3. Alpha rhythm is blocked and is replaced by a
REPRODUCTION
high frequency, low voltage beta rhythm.
m

Instruction: Some questions have more than


4. This is the function of the ascending reticular one correct answer. Please select all correct
activating system. answers.

5. The proportion of REM sleep decreases with 1. The term hormone was first used to describe
Ai

age. the actions of:


A. secretin
8. They are seen in Stage II sleep. B. insulin
C. epinephrine
10. The decrease in PaCO2 as a result of D. GH
hyperventilation results in cerebral
vasoconstriction; the decrease in cerebral blood 2. Which of the following is not a glycoprotein?
flow presumably reduces the seizure threshold. A. Erythropoietin
B. TRH
C. FSH

136
D. hCG 10. Sertoli cells do not produce:
A. estrogens
3. Which of the following hypophysiotropic B. androgen-binding protein
hormones is a tripeptide? C. inhibin
A. TRH D. androgens
B. Somatostatin E. Mullerian inhibiting substance
C. Dopamine
D. CRH 11. The thyroid gland produces:
A. CGRP

in
4. The three glycoprotein hormones produced by B. PTHRP
the pituitary are: C. TSH
A. FSH, LH and TRH D. calcitonin
B. TRH, ACTH and LH
C. TSH, LH and FSH 12. The hormone that lowers plasma calcium is:

s.
D. GH, TSH and LH A. calcineurin
B. calcitonin
5. Histamine released from mast cells in the C. PTH
stomach stimulates the secretion of HCl by the D. PTHRP
parietal cells. This is an example of _________
signaling. 13. TSH secretion is inhibited by:
A. endocrine
B. paracrine
C. autocrine
D. juxtacrine
iim A. dopamine
B. TRH
C. somatostatin
D. free T3/T4

6. Which of the following has the longest 14. The RDA of iodine is:
biologic half-life? A. 20 micrograms
A. Insulin B. 50 micrograms
4a
B. Angiotensin II C. 150 micrograms
C. Glucagon D. 300 micrograms
D. Thyroxine
15. The thyroid gland predominantly secretes:
7. The commonest cell type in the anterior A. thyroxine
pituitary is: B. triiodothyronine
A. somatotropes
B. lactotropes 16. Most of the T3 is formed from T4 by the
m

C. corticotropes action of:


D. thyrotropes A. deiodinase I
E. gonadotropes B. deiodinase II
C. deiodinase III
8. Growth hormone stimulates the secretion of: D. deiodinase IV
Ai

A. somatostatin
B. gonadotropin 17. Iodine is concentrated in thyroid cells by
C. somatomedins A. primary active transport
B. secondary active transport
9. Who is regarded as the father of C. facilitated diffusion
Endocrinology?
A. Hans Selye 18. Most of the circulating T4 is bound to:
B. Maxwell Wintrobe A. prealbumin
C. Charles Banting B. albumin
C. thyroxine-binding globulin

137
C. zona reticularis
19. Thyroxine does not increase O2 consumption
in the: 27. New cortical cells are formed in the:
A. heart A. zona glomerulosa
B. adult brain B. zona fasciculata
C. brown adipose tissue C. zona reticularis
D. skeletal muscle
28. Which one of the following is not a
20. The normal basal metabolic rate in a catecholamine?

in
euthyroid adult male is (BSA is body surface A. Epinephrine
area): B. Norepinephrine
A. 10-15 kcal/m2 BSA/hr C. Dopamine
B. 20-25 kcal/m2 BSA/hr D. Acetylcholine
C. 35-40 kcal/m2 BSA/hr
D. 45-60 kcal/m2 BSA/hr

s.
29. Androgens are predominantly secreted by
the:
21. TSH secretion is stimulated by: A. zona glomerulosa
A. dopamine B. zona fasciculata
B. TRH C. zona reticularis
C. somatostatin
D. free T3/T4

incorrect?
iim
22. Which of the following statements is

A. Thyroxine is biologically more active than


30. The hormones produced by the ovary
include:
A. estrogens
B. progesterone
C. relaxin
triiodothyronine. D. inhibin
B. Thyroid gland predominantly secretes T4.
C. Most T3 is formed in the circulation by the 31. Norepinephrine has greater affinity for:
4a
action of deiodinase I. A. adrenergic receptors
B. 1 adrenergic receptor
23. Insulin: C. 2 adrenergic receptor
A. decreases K+ uptake by skeletal muscle
B. increases protein catabolism in muscle 32. The lipolytic effect of catecholamines is
C. decreases cell growth mediated by:
D. increases the utilization of ketone bodies A. 1 adrenergic receptors
B. 2 adrenergic receptors
m

24. Somatostatin is found in: C. 3 adrenergic receptors


A. pancreas D. 1 & 3 adrenergic receptors
B. GIT
C. hypothalamus 33. Most pheochromocytomas secrete:
A. Epinephrine
Ai

25. Administration of ACTH reduces absolute B. Norepinephrine


eosinophil count. This is the basis of the: C. Dopamine
A. Thorns test D. Adrenomedullin
B. Fischers test
C. Mann-Whitneys test 34. Congenital lipoid adrenal hyperplasia occurs
D. Andrews test due to a lack of:
A. CYP450scc
26. The secretion of aldosterone is limited to: B. 3-hydroxysteroid dehydrogenase
A. zona glomerulosa C. 17 hydroxylase
B. zona fasciculata D. Steroidogenic acute regulatory protein

138
35. The most abundant cell type in the islets of 42. The insulin response to an intravenous
Langerhans is: infusion of glucose is _____ the response to a
A. A cells comparable oral glucose load.
B. B cells A. greater than
C. D cells B. equal to
D. F cells C. lesser than

36. The glucose sensor in B-cells of the islets of 43. Glucagon facilitates all of the following

in
Langerhans is: except:
A. GLUT-1 A. lipolysis
B. GLUT-2 B. gluconeogenesis
C. GLUT-3 C. hepatic glycogenolysis
D. GLUT-4 D. muscle glycogenolysis

s.
37. Insulin-dependent glucose uptake in skeletal 44. Which of the following is a potent stimulator
muscle occurs mainly through: of insulin secretion?
A. GLUT-1 A. Somatostatin
B. GLUT-2 B. VIP
C. GLUT-3 C. Glucagon like polypeptide
D. GLUT-4

glucose uptake?
A. Red blood cells
iim
38. Which tissues do not require insulin for
D. Leptin

45. Hypoglycemia does not stimulate the


secretion of:
A. epinephrine
B. Brain B. cortisol
C. Adipose tissue C. growth hormone
D. Skeletal muscle D. insulin
4a
E. glucagon
39. Which one of the following is not an energy
releasing hormone? 46. The hormone of energy storage is:
A. Growth hormone A. growth hormone
B. Cortisol B. thyroxine
C. Epinephrine C. insulin
D. Insulin D. glucagon
E. Glucagon E. epinephrine
m

40. Which of the following hormones is/ are 47. Which of the following increase insulin
diabetogenic? resistance? (Tick all that apply)
A. Epinephrine A. Leptin
B. Cortisol B. TNF-
Ai

C. Growth hormone C. Resistin


D. Glucagon D. Adiponectin
E. Thyroid hormones E. Growth hormone
F. Epinephrine
41. Insulin secretion from B-cells in pancreas is
inhibited by: 48. The two drugs that are commonly used to
A. acetylcholine induce diabetes in rats are:
B. epinephrine A. streptozotocin
C. glucagon B. alloxan
D. somatostatin C. forskolin

139
D. capsaicin
56. Chronic androgen abuse by male athletes is
49. Which of the following hormones inhibits least likely to result in:
ovulation? A. increased erythropoiesis
A. Prolactin B. increased sperm count
B. FSH C. increased respiratory endurance
C. LH D. increased muscle mass
D. hMG
57. Which of the following statements is

in
50. The maternal growth hormone of pregnancy incorrect?
is: A. Somatopause is attributed to a decline in the
A. growth hormone activity of GH-IGF1 axis
B. hCS B. Andropause is characterized by an increase
C. estriol in LH and FSH.

s.
D. hCG C. Menopause is characterized by decreased
ovarian responsiveness to pituitary
51. Which of the following is not detectable in gonadotropins.
the blood of a nonpregnant female? D. Adrenopause is characterized by decreases
A. Oxytocin in plasma DHEA.
B. Relaxin
C. hCS
D. Inhibin
iim
52. Which of the following hormones is not
detectable in females?
58. The hormone of darkness is:
A. leptin
B. melatonin
C. serotonin
D. galanin
A. Androstenedione
B. Testosterone 59. The adrenal medulla does not secrete:
C. Dihydrotestosterone A. dopamine
4a
D. Dehydroepiandrosterone B. adrenomedullin
C. enkephalins
53. Which of the following hormones is not D. VIP
detectable in males?
A. Oxytocin 60. Which hormone stimulates the release of
B. Estrogen adrenal androgens?
C. Progesterone A. LH
D. LH B. FSH
m

C. ACTH
54. Which of the following genotypes is not D. GnRH
associated with any characteristic abnormalities?
A. XXX 61. The biologically most active androgen is:
B. XXY A. testosterone
Ai

C. XO B. dehydroepiandrosterone
D. YO C. androstenedione
D. dihydrotestosterone
55. Which of the following genotypes is lethal?
A. XXX 62. The transient loss of the ability to
B. XXY discriminate between blue and green following
C. XO the use of sildenafil is due to inhibition of:
D. YO A. cGMP-PDE3
B. cGMP-PDE4
C. cGMP-PDE5

140
D. cGMP-PDE6 70. The average number of spermatids formed
from a single spermatogonium is:
63. Which of the following drugs produces A. 16
therapeutic effects by downregulating hormone B. 128
receptors in target cells? C. 512
A. FSH
B. GnRH analogs 71. The formation of mature sperm cells from a
C. Inhibin primitive germ cell takes an average of:
D. Finasteride A. 2 days

in
B. 1 week
64. HCG acts upon the: C. 74 days
A. LH receptor D. 3 months
B. PRL receptor
C. GH receptor 72. Sperm cells first acquire the ability to move

s.
D. FSH receptor forward (progressive motility) in the:
A. seminiferous tubules
65. The testis determining gene is located in: B. epididymis
A. chromosome 6 C. female genital tract
B. chromosome 12
C. short arm of chromosome 22 73. Sildenafil citrate (Viagra) enhances penile
D. Y chromosome

synthesized starting from:


A. tyrosine
iim
66. The hormone secreted by the pineal is
erection by:
A. activating adenylyl cyclase
B. blocking nitric oxide synthase
C. activating guanylyl cyclase
D. inhibiting cGMP-phosphodiesterase
B. tryptophan
C. phenylalanine 74. Sildenafil is most active against:
D. lysine A. cGMP-PDE3
4a
B. cGMP-PDE4
67. In normal healthy adults on a daytime work C. cGMP-PDE5
shift, at what time does pineal hormone peak in D. cGMP-PDE6
blood?
A. 2 am Answers: Self-study in Endocrinology &
B. 8 am Reproduction
C. 2 pm 1A 2B 3A 4C 5B
D. 8 pm 6D 7A 8C 9A 10D
m

11D 12B 13ACD 14C 15A


68. Which of the following statements about the 16A 17B 18C 19B 20C
pineal gland in human beings is correct? 21B 22A 23D 24ABC 25A
A. It directly responds to light 26A 27A 28D 29C 30all
B. It is part of the nervous system 31A 32D 33B 34D 35B
Ai

C. It is controlled by the hypothalamus 36B 37D 38AB 39D 40all


D. It is rudimentary 41D 42C 43D 44C 45D
46C 47BCEF 48AB 49A 50B
69. Which of the following cells undergo 51C 52none 53none 54A 55D
meiotic division?
56B 57none 58B 59D 60C
A. Primordial germ cells
61D 62D 63B 64A 65D
B. Primary spermatocytes
66B 67A 68C 69B 70C
C. Secondary spermatocytes
D. Spermatids 71C 72B 73D 74C

141
Answer Explanations: 38. Red blood cells and brain do not require
insulin for glucose uptake.
2. TRH is a tripeptide.
40. Diabetogenic hormones: epinephrine,
7. 50% of cells in the anterior pituitary secrete norepinephrine, glucagon, growth hormone,
growth hormone. See Table 22-1, p.397, WFG, cortisol, and thyroxine. All of them oppose the
2005. blood glucose lowering effects of insulin.

8. The somatomedins are insulin-like growth 41. When acting via alpha 2 adrenergic

in
factors 1 (IGF-1) and IGF-2. receptors, epinephrine inhibits insulin secretion;
however, when acting via beta adrenergic
15. The human thyroid releases about 80 receptors, it stimulates insulin secretion. See p.
micrograms of T4 and 4 micrograms of T3 per 345, Ch 19, WFG, 2005.
day.

s.
42. The insulin response to an oral load of
16. One third of circulating T4 is normally glucose is greater because of the release of
converted to T3 in humans by the action of 5- glucose-dependent insulinotropic polypeptide
deiodinase in the liver and kidneys. See p. 322, and other hormones from the GIT, which in turn
Ch 18, WFG, 2005. stimulate insulin secretion. This effect, called
the incretin effect, is absent when glucose is
18. See p. 321, Ch 18, WFG, 2005.iim
19. T4 and T3 increase the oxygen consumption
of all metabolically active tissues. The
exceptions are the adult brain, testes, uterus,
administered intravenously.

44. GLP-1 (7-36) is glicentin-like polypeptide, a


gastrointestinal hormone; it is a potent
stimulator of insulin secretion (incretin). See p.
lymph nodes, spleen and anterior pituitary. See 348, Ch 19, WFG, 2005.
p. 324, Ch 18, WFG, 2005.
47. Leptin and adiponectin are adipokines (i.e.,
4a
23. Insulin increases K uptake by muscle hormones from adipocytes) that increase insulin
probably by stimulating Na-K ATPase. It sensitivity. In contrast, TNF- and resistin
increases the oxidation of ketone bodies; that is decrease insulin sensitivity. See p.347-8, Ch 19,
why it is used in the treatment of diabetic WFG, 2005.
ketoacidosis.
51. Human chorionic somatomammotropin is a
25. With radioimmunoassays now being placental hormone.
available for most hormones, the Thorns test is
m

rarely if ever used now to assess adrenocortical 56. Chronic use of pharmacologic doses of
function. androgens suppresses FSH release from the
pituitary thereby inhibiting spermatogenesis.
31. Norepinephrine has a greater affinity for
alpha-adrenergic receptors and epinephrine has a 57. The term adrenopause is used to refer to a
Ai

greater affinity for beta-adrenergic receptors. decline in serum DHEA. Since DHEA is the
For example, when norepinephrine is most abundant adrenocortical hormone, a fall in
administered intravenously the blood pressure serum DHEA indicates a decline in functional
increases enough to trigger a baroreflex- adrenal cortical mass.
mediated decrease in heart rate although
norepinephrine has a direct chronotropic action 63. Long acting GnRH analogs suppress FSH
on the heart. and LH release by downregulating GnRH
receptors in the pituitary, and thereby inhibit
36, 37. See p. 338, Table 19-5, WFG, 2005. ovulation.

142
****************************** 7. The adverse gastrointestinal effects of
erythromycin are probably due to its agonist
GASTROINTESTINAL actions on:
PHYSIOLOGY A. histamine receptors
B. motilin receptors
Some questions have more than one correct C. CCK receptors
answer, as suggested by the stem of the D. somatostatin receptors
question. Check all correct answers.
8. GIP is an acronym for:

in
1. The term hormone was coined by Ernest H. A. gastric inhibitory peptide
Starling to describe the actions of: B. glucagon inhibitory peptide
A. secretin C. galanin inhibitory peptide
B. insulin D. glucose-dependent insulinotropic
C. growth hormone polypeptide
D. glucagon

s.
9. Trypsinogen, a pancreatic proenzyme, is
2. All of the following stimulate gastric acid activated by:
secretion except: A. enterokinase
A. pentagastrin B. HCl
B. hypoglycemia C. HCO3
C. acetylcholine
D. PGE2
iim
3. Which amongst the following are the most
potent inhibitors of gastric acid secretion?
D. chloride ions

10. Most pancreatic zymogens are activated in


the duodenal lumen by:
A. enteropeptidase
A. H+K+ ATPase inhibitors B. trypsin
B. H2 receptor blockers C. trypsin activator protein
C. M2 receptor blockers D. enterokinase
4a
D. Gastrin receptor blockers
11. Trypsin inhibitor:
4. Vagally mediated increases in gastric acid A. inhibits the action of pancreatic enzymes in
secretion is partly mediated by: the lumen of the duodenum
A. gastrin releasing peptide (GRP) B. inhibits activation of trypsin in the pancreas
B. gastric inhibitory peptide (GIP) C. deficiency is implicated in acute pancreatitis
C. vasoactive intestinal polypeptide (VIP) D. is produced by enterocytes
D. somatostatin
m

12. In the GIT, which of the following cell types


5. The optimum pH for the activity of pepsin is: secrete mucus?
A. less than 1 A. M cells
B. between 1.6 and 3.2 B. Goblet cells
C. between 3 and 5 C. Surface epithelial cells
Ai

D. neutral pH D. Brunners glands

6. Which of the following statements about CCK 13. Which of the following is a primary bile
is incorrect? acid?
A. It causes gall bladder contraction A. Sodium taurocholate
B. It relaxes the sphincter of Oddi B. Chenodeoxycholic acid
C. It relaxes the pyloric sphincter C. Deoxycholic acid
D. It stimulates the secretion of an enzyme-rich D. Lithocholic acid
pancreatic juice
14. The most abundant bile acid is:

143
A. cholic acid 22. Quantitatively, the most important enzyme
B. chenodeoxycholic acid in the digestion of fat is:
C. deoxycholic acid A. lingual lipase
D. lithocholic acid B. gastric lipase
C. pancreatic lipase
15. Reabsorption of bile acids occurs mainly in D. lipoprotein lipase
the:
A. duodenum 23. Which one of the following is an
B. jejunum endopeptidase?

in
C. terminal ileum A. Carboxypeptidase A
D. colon B. Deoxyribonuclease
C. Trypsin
16. Bile acids are synthesized starting from: D. Dipeptidase
A. cholesterol

s.
B. fatty acids 24. Which one of the following enzymes is
C. lecithin produced by enterocytes?
D. bile salts A. Lactase
B. Colipase
17. Vitamin B12 is mainly absorbed in the: C. Cholesterol ester hydrolase
A. terminal ileum D. Lingual lipase
B. upper jejunum
C. duodenum
D. stomach
iim
18. Calcium absorption from the intestine:
25. Disaccharidases are present in:
A. pancreatic acinar cells
B. brush border of enterocytes
C. D cells of pancreas
A. is facilitated by vitamin D D. Brunners glands
B. is decreased by phytates and oxalates
C. is regulated in accordance with plasma 26. The mechanism that ensures complete
4a
calcium absorption of glucose from the intestinal lumen
D. occurs mainly in the upper small intestine is:
A. simple diffusion
19. The protein that ferries iron in the plasma is: B. facilitated diffusion
A. mobiliferrin C. primary active transport
B. transferrin D. secondary active transport
C. ferritin
D. hemosiderin 27. The maximal rate of glucose absorption from
m

the intestine is:


20. The principal storage form of iron in the A. 10 g/hour
tissues is: B. 40 g/hour
A. ferritin C. 120 g/hour
B. transferrin D. 180 g/hour
Ai

C. mobiliferrin
D. hemosiderin 28. The least oxygenated zone in the hepatic
acinus is:
21. Most of the iron in the body is present in: A. zone I (periportal zone)
A. hemoglobin B. zone II
B. myoglobin C. zone III (perivenular zone)
C. ferritin
D. transferrin 29. The first part of a test meal reaches the
cecum in about:
A. 4 hours

144
B. 9 hours pancreatic proenzymes are activated only when
C. 12 hours they reach the intestinal lumen.
D. 6 hours
13. Cholic and chenodeoxycholic acids are
30. The main function of the colon is absorption primary bile acids; deoxycholic acid and
of: lithocholic acids are formed in the intestine
A. Na and H2O (secondary bile acids) by the action of bacteria
B. fat on primary bile acids.
C. bile acids

in
D. iron 21. 70% of iron in the body is present in Hb.
Under abnormal circumstances, a large amount
Answers: Self-Study in GI Physiology of iron is present in hemosiderin.
1A 2D 3A 4A 5B
6C 7B 8AD 9A 10B 22. This is evident from the fact that chronic

s.
11BC 12all 13B 14A 15C pancreatic exocrine insufficiency results in
16A 17A 18all 19B 20A steatorrhea. In contrast, fat digestion and
21A 22C 23C 24A 25B absorption is not significantly affected by
26D 27C 28C 29A 30A deficiency of the other lipases.

26. Glucose could enter enterocytes from the


Answer Explanations:
iim
3. This is because the H-K ATPase is the final
common pathway for acid secretion in the
parietal cell. New proton pumps need to be
synthesized before parietal cells can overcome
the inhibitory effects of H-K ATPase inhibitors.
intestinal lumen by diffusion through GLUT;
however, for complete absorption of glucose
against a concentration gradient, the active
transport mechanism provided by the sodium-
glucose cotransporter is required.

4. Some vagal neurons ending on G cells release 28. See


gastrin releasing peptide. Gastrin from G cells http://en.wikipedia.org/wiki/Hepatic_lobule
4a
stimulates acid secretion from parietal cells.
http://tiny.cc/zvez5 the full URL is
http://www.vivo.colostate.edu/hbooks/pathphys/digestion/liver/hist
6. CCK contracts the pyloric sphincter thereby o_acinus.html
inhibiting gastric emptying.
Because the perivenular zone (Zone III) is
7. Erythromycin is a motilin receptor agonist. It farthest from hepatic arterial inflow, it is the
is used in the management of diabetic most vulnerable to ischemic injury.
m

gastroparesis. ***************************************
RENAL PHYSIOLOGY
8. Glucose-dependent insulinotropic polypeptide
was previously called gastric inhibitory peptide.
1. ECF volume is determined by:
However, now it is clear that the physiologic
A. plasma [Na]
action of GIP is stimulation of insulin secretion.
Ai

B. plasma protein concentration


Its inhibitory effects on gastric secretion occur
C. the amount of sodium in the ECF
only at pharmacologic doses.
2. The chief determinant of plasma osmolality is
9, 10. Enteropeptidase (enterokinase) is an
A. plasma [Na]
intestinal brush border enzyme that catalyzes the
B. plasma [glucose]
proteolytic activation of trypsinogen to trypsin.
C. blood [urea]
Trypsin is the common activator of all
D. plasma [albumin]
pancreatic proenzymes. This ensures that
3. The substrate for renin is:

145
A. angiotensinogen A. low blood pressure
B. angiotensin I B. hyperkalemia
C. angiotensin II C. metabolic acidosis
D. bradykinin D. a decrease in plasma renin activity

4. Angiotensin converting enzyme is a / an: 12. Which of the following substances has a
A. dipeptidyl carboxypeptidase clearance ratio of 1.2?
B. aminopeptidase A. Inulin
C. aspartyl protease B. Creatinine

in
C. Urea
5. The kidneys are not known to secrete: D. PAH
A. calcitriol
B. rennin 13. Which one of the following substances has a
C. erythropoietin clearance ratio close to 1?

s.
D. PGE2 A. Creatinine
B. Mannitol
6. Renin is released mainly from: C. Urea
A. macula densa D. Glucose
B. juxtaglomerular cells
C. type I medullary interstitial cells 14. Which of the following substances is cleared
D. peritubular capillaries

cells secrete:
A. renin
iim
7. In the kidneys, type I medullary interstitial
the most by the kidneys?
A. Inulin
B. PAH
C. Creatinine
D. Urea
B. PGE2
C. aquaporins 15. Arrange the following substances in the
D. aldosterone descending order of their clearance. [Cr:
4a
creatinine]
8. Mesangial cells are similar to: A. PAH > Inulin > Urea > Cr > Glucose
A. pericytes B. Inulin > PAH > Cr > Urea > Glucose
B. fibrocytes C. PAH > Cr > Inulin > Urea > Glucose
C. mast cells
D. ependymal cells 16. The tubular transport maximum for glucose
(Tm) in men is about:
9. The stimulators of aldosterone production by A. 100 mg/min
m

the zona glomerulosa cells are (tick all that B. 325 mg/min
apply): C. 375 mg/min
A. a rise in plasma [K+]
B. angiotensin II 17. When urine is maximally concentrated, the
C. angiotensin III ratio of osmolality of urine to plasma is about:
Ai

D. ACTH A. 3
B. 4.2
10. The actions of aldosterone are antagonized C. 4.7
by: D. 6.7
A. angiotensin III
B. arginine vasopressin (AVP) 18. High protein diets increase the likelihood of:
C. ANP A. metabolic acidosis
B. metabolic alkalosis
11. Chronic licorice ingestion is characterized C. respiratory acidosis
by: D. respiratory alkalosis

146
Answers: Self-Study in Renal Physiology
19. Which of the following observations on a 1C 2A 3A 4A 5B
urine sample clearly suggests that urine is 6B 7B 8A 9all 10C
hypotonic? 11D 12B 13B 14B 15C
A. A high Uosm / Posm ratio 16C 17C 18A 19C 20B
B. A urinary pH of 7 21C 22A 23C 24B 25AB
C. Positive free water clearance
Whatzit
20. The first urge to void urine is felt at a 1. The most abundant cation in ECF

in
bladder volume of about: 2. The most abundant anion in ECF
A. 50 ml 3. The most abundant cation in ICF
B. 150 ml 4. The most abundant protein in blood
C. 250 ml 5. The only organic acid to be both secreted
D. 400 ml and reabsorbed by the kidney

s.
6. The ion that is absorbed as well as secreted
21. The volume of urine in the bladder that by the kidney
initiates reflex contraction of the bladder is
about: Answers (to Whatzit):
A. 100200 ml 1. Sodium
B. 200300 ml
C. 300400 ml iim
22. Which of the following is not a feature of
tabes dorsalis?
A. Bladder wall hypertrophy
2. Chloride
3. Potassium
4. Hemoglobin
5. Uric acid
6. Potassium

B. Distended bladder True or false:


C. Urinary retention 1. Urine volume increases to as much as 23 L
D. Incontinence / day in the complete absence of
4a
vasopressin.
23. Arrange the following substances in the 2. 75 % of potassium is secreted in the distal
descending order of their concentrations in nephron (i.e., DCT + collecting ducts)
intracellular fluid. 3. 65% of filtered water is reabsorbed in the
A. K+ > Ca2+ > Na+ > H+ PCT
B. H+ > K+ > Na+ > Ca2+ 4. Stimulation of renal sympathetic nerves
C. K+ > Na+ > Ca2+ > H+ inactivates the renin angiotensin system.
D. K+ > Ca2+ > H+ > Na+
m

Answers: statement 4 is false; others are true.


24. Which of the following statements about
control of micturition is incorrect? Answer Explanations:
A. The micturition center is located in the pons. 1, 2. Children and adults have the same
B. In adults, the first urge to void is felt at a concentration of sodium in plasma. Yet their
Ai

bladder volume of about 400 ml. ECF volumes are greatly different. Thus, ECF
C. Decentralization of bladder function results volume is proportional to the amount of sodium
in urinary retention. in ECF. The concentration of sodium in ECF is
an important determinant of plasma osmolality.
25. What is the immediate effect of spinal cord
transection on bladder function? 7. One of the effects of renal prostaglandins is
A. Urinary retention mediating redistribution of blood flows within
B. Overflow incontinence the kidneys.
C. Anuria

147
8. When mesangial cells contract, surface area concentrated (i.e. hypertonic compared to
available for filtration reduces. plasma), free water clearance is negative. When
water is lost in excess of osmoles cleared, free
9. Factors stimulating aldosterone release water clearance is positive - as in diabetes
from the adrenal cortex: Angiotensin II is the insipidus.
most important stimulator of aldosterone release.
Even a small increase in plasma [K] stimulates 20. See p 726, Ch 38, WFG, 2005.
aldosterone release. In turn, aldosterone
regulates plasma [K] by promoting K secretion 21. Laplaces law states that pressure (P) in a

in
in the distal portions of nephron (distal hollow viscus is directly proportional to wall
convoluted tubule and collecting ducts). tension (T) and inversely with radius (r).
Angiotensin III has 100% aldosterone releasing P = T/r
activity but only 40% of the pressor activity of As the bladder is distended with urine, the radius
aldosterone. ACTH as well as a decrease in as well as passive tension increases

s.
plasma [Na] stimulate aldosterone release. proportionately so there is little change in
intravesical pressure. However, at an intravesical
11. Licorice contains glycyrrhyzinic acid, a volume of about 300-400 ml, any further stretch
substance with mineralocorticoid activity. results in active contraction of the smooth
Chronic licorice ingestion is associated with muscle of the bladder and a sharp increase in
symptoms of mineralocorticoid excess. Thus, the intravesical pressure. This results in emptying of
iim
manifestations are those of primary
hyperaldosteronism hypertension,
hypokalemia and metabolic alkalosis. The
decrease in plasma renin activity is a result of
suppression of renin secretion caused by
the bladder. See p 727, Ch 38, WFG, 2005.

22. Tabes dorsalis predominantly affects the


dorsal roots; thus, the effects observed are those
of deafferentation: the bladder becomes
increased blood volume. distended, thin walled and hypotonic. See p 728,
Ch 38, WFG, 2005.
12. Clearance ratio is the ratio of the clearance
4a
of a substance by the kidneys to the clearance of 24. In chronic paraplegics, the voiding reflex
inulin. The clearance of substances secreted by returns and it may be hyperactive. The
the nephron is greater than inulin clearance. In hyperactive reflex may be because of release of
contrast, the clearance of substances (such as the spinal mechanism for micturition from the
glucose) that are reabsorbed by the nephron is inhibitory effects of higher control. The bladder
less than inulin clearance. wall becomes hypertrophied and bladder
capacity is reduced. This condition is called
14. The clearance of a substance that is actively spastic neurogenic bladder. See p 728, Ch 38,
m

secreted by the nephron (example, PAH) is WFG, 2005.


greater than the clearance of a substance that is
excreted by filtration alone (example, inulin). 25. The immediate effect of spinal cord
transection is urinary retention. This is, of
17. Maximum urine osmolality = 1400 course, followed by overflow incontinence; i.e.,
Ai

mOsm/kg H2O. Plasma osmolality = 300 excessive stretch of the urinary bladder, and
mOsm/kg H2O. When urine is maximally local reflex contractions of the detrusor that is
concentrated, Uosm/Posm = 4.7. not under voluntary control. See p 728, Ch 38,
WFG, 2005.
18. The metabolism of sulfur containing amino
acids generates hydrogen ions (fixed acid) that
only the kidneys can excrete.

19. Free water clearance is the amount of


solute-free water excreted. When urine is

148

Вам также может понравиться